You are on page 1of 189

PIPE REVIEWER

Find the work posses for a Helium gas at 200C


*A. 609 KJ/kg B. 168 KJ/kg C. 229 KJ/kg D. 339 KJ/kg
Solution:
W = m R T = m (8.314 / M) T
For helium, M = 4
W/m = (8.314/4)(20 + 273) = 609 KJ/kg

Two kilogram of gas is confined in a 1 m3 tank at 200 kpa and 880C. What type of gas is
in the tank?
A. Helium *B. Ethane C. Methane D. Ethene
Solution:
PV=mRT
200 (1) = 2 (8.314/M)(88+273)
M = 30
Therefore: the gas is Ethane (C2 H8)

Find the enthalpy of Helium if its internal energy is 200 KJ/kg


144 KJ/kg B. 223.42 KJ/kg *C. 333.42 KJ/kg D. 168 KJ/kg
Solution:
R = 8.314/4 = 2.0785
K = 1.667 for helium
Cp = k R/(k - 1) = 1.667(2.0785)/(1.667 1) = 5.195 KJ/kg-K
Cv = R/(k 1) = 2.0785/(1.667 1) = 3.116 KJ/kg K
h/U = Cp/Cv
h/200 = 5.195/3.116
h = 333.42 KJ/kg

Compute the mass of a 2 m3 propane at 280 kpa and 40C.


6.47 kg B. 5.1 kg C. 10.20 kg *D. 9.47 kg
Solution:
Propane is C3 H3--------------M = 12(3) + 8(1) = 44
PV = m R T
280(2) = m (8.314/44)(40 + 273)
m = 9.47 kg

Compute the air flow in ft3/min of mechanical ventilation required to exhaust an


accumulation of refrigerant due to leaks of the system capable of revolving air from the
machinery room for a mass of 4 lbs refrigerant.
*A. 200 B. 210 C. 220 D. 230
Solution:
Q = 100 x G0.5 ft3/min
Q = 100 x (4)0.5 = 200 ft3/min
Compute the free-aperture cross section in m2 for the ventilation of a machinery room if
the mass of refrigerant is 9 kg.
0.314 *B. 0.414 C. 0.514 D. 0.614
Solution:
F = 0.138 G0.5 m2
F = 0.138 (9)0.5 = 0.414 m2

A 29.53 x 39.37 pressure vessel contains ammonia with f = 0.041. Compute the
minimum required discharge capacity of the relief device in kg/hr.
106.71 kg/hr B. 108.71 kg/hr *C. 110.71 kg/hr D. 112.71 kg/hr
Solution:
C = f D L, kg/s
C = 0.041(29.53/39.37)(39.37/39.37) = 0.03075 kg/s (3600) = 110.71 kg/hr

Compute the maximum length of the discharge pipe installed on the outlet of a
pressure-relief device in feet for internal pipe diameter of 0.5 inch and rated discharge
capacity is 8 lb/min of air. The rated pressure of relief valve is 16 psig.
*A. 0.286 ft B. 0.386 ft C. 0.486 ft D. 0.586 ft
Solution:
P = Pg + Patm = 16 x 1.1 + 14.7 = 32.3 psia
L = 9P2d5/16Cr2 = 9(32.3)2(0.5)5/16(8)2 = 0.286 ft

A thermal power plant has a heat rate of 11,363 Btu/kw-hr. Find the thermal efficiency of
the plant.
28% *B. 30% C. 34% D. 40%
Solution:
eth = 3412/Heat rate = 3412/11,363 = 30%

What is the hydraulic gradient of a 1 mile, 17 inches inside diameter pipe when 3300
gal/min of water flow with f = 0.03.
*A. 0.00714 B. 0.00614 C. 0.00234 D. 0.0018
Solution:
v = (3300/7.481)/(/4)(17/12)2(60) = 4.66 ft/s
L = 1 mile = 5280 ft
hL = fLv2/2_D = 0.03(5280)(4.66)2/2(32.2)(17/12) = 37.7 ft
Hydraulic gradient = 37.7/5280 = 0.007.14

Find the loss of head in the pipe entrance if speed of flow is 10 m/s.
5.10 m B. 10.2 m C. 17.4 m *D. 2.55 m
Solution:
Loss at entrance = 0.5 (v2/2g) = 0.5 [102 / 2(9.81)] = 2.55 m

Wet material, containing 220% moisture (dry basis) is to be dried at the rate of 1.5 kg/s
in a continuous dryer to give a product containing 10% (dry basis) . Find the moisture
removed, kg/hr
*A. 3543.75 kg/hr B. 3513.75 kg/hr C. 3563.75 kg/hr D. 3593.75 kg/hr
Solution:
Solid in wet feed = solid in dried product
[1/(1 + 2.2)](1.5) = [1/(1 + 0.1)](x)
x = 0.5156 kg/s (total dried product)
Moisture removed = 1.5 0.5156 = 0.984 kg/s = 3543.75 kg/hr

Copra enters a dryer containing 70% moisture and leaves at 7% moisture. Find the
moisture removed on each pound on solid in final product.
6.258 lb B. 1.258 lb C. 4.258 lb *D. 2.258 lb
Solution:
Solid in wet feed = solid in dried product
0.3x = 1
x = 3.333 lbs
1 = 0.93y
y = 1.07527 lb
Moisture removed = x y = 3.333 1.07527 = 2.258 lb

A 1 m x 1.5 m cylindrical tank is full of oil with SG = 0.92. Find the force acting at the
bottom of the tank in dynes.
106.33 x 103 dynes B. 106.33 x 104 dynes C. 106.33 x 105 dynes *D. 106.33 x
106 dynes
Solution:
P = w h = (0.92 x 9.81) (1.5) = 13.5378 kpa
F = PA = 13.5378(/4 x 12) = 10.632 KN = 10,632.56 N x 10,000 dynes/N
F = 106.33 x 106 dynes

Find the pressure at the 100 fathom depth of water in kpag.


*A. 1,793.96 kpag B. 1,893.96 kpag C. 1,993.96 kpag D. 1,693.96
kpag
Solution:
H = 100 fathom x 6 = 600 ft
P = w h = (600/3.281)(9.81) = 1,793.96 kpag

Find the depth in furlong of the ocean (SG = 1.03) if the pressure at the sea bed is
2,032.56 kpag.
*A. 1 B. 2 C. 3 D. 4
Solution:
P=wh
2,032.56 = (1.03 x 9.81) h
h = 201.158 m x 3.281 ft/m x 1 yd/3ft x 1 furlong/220yd = 1 furlong

Find the mass of 10 quartz of water.


10.46 kg *B. 9.46 kg C. 11.46 kg D. 8.46 kg
Solution:
V = 10 quartz x 1gal/4quartz x 3.785li/1gal x 1m 3/1000li
V = 0.0094625 x 10-3m3
w = m/V
1000 = m/0.0094625 x 10-3
m = 9.46 kg

Find the mass of carbon dioxide having a pressure of 20 psia at 200F with 10 ft 3
volume.
1.04 lbs B. 1.14 lbs *C. 1.24 lbs D. 1.34 lbs
Solution:
PV = m R T
(20 x 144)(10) = m (1545/44)(200 + 460)
m = 1.24 lbs

Find the heat needed to raise the temperature of water from 30C to 100C with 60%
quality. Consider an atmospheric pressure of 101.325 kpa. Use the approximate
enthalpy formula of liquid.
293.09 KJ/kg B. 1,772.90 KJ/kg C. 1,547.90 KJ/kg *D. 1,647.29
KJ/kg
Solution:
At 100C ht = cp t = 4.187 (100) = 418.7 KJ/kg
htg = 2257 KJ/kg
h2 = h1 + x htg = 418.7 + 0.60(2257) = 1,772.9 KJ/kg
Q = 1(4.187)(100 30) + 1(1772.9 418.7) = 1,647.29 KJ/kg

Find the enthalpy of water at 212F and 14.7 psi if the dryness factor is 30%. Use the
approximate enthalpy formula of liquid.
461 Btu/lb *B. 471 Btu/lb C. 481 Btu/lb D. 491 Btu/lb
Solution:
ht = (F 32) = (212 32) = 180 Btu/lb
htg = 970 Btu/lb
h = ht + x htg
h = 180 + 0.3(970) = 471 Btu/lb

An air compressor consumed 1200 kw-hr per day of energy. The electric motor driving
the compressor has an efficiency of 80%. If indicated power of the compressor is 34 kw,
find the mechanical efficiency of the compressor.
117.65 % B. 75 % *C. 85 % D. 90 %
Solution:
P/m = 1200kw-hr/24 hrs = 50 kw
BP = 50(0.80) = 40 kw
em = 34/40 = 85 %

A refrigeration system consumed 28,000 kw-hr per month of energy. There are 20 % of
energy is lost due to cooling system of compressor and motor efficiency is 90 %. If COP
of the system is 6, find the tons of refrigeration of the system.
43.15 TR B. 46.15 TR *C. 49.15 TR D. 41.15 TR
Solution:
P/m = 28,800/(24 x 30) = 40 kw
BP = 40(0.90) = 36 kw
Wc = 36(1 0.20) = 28.80 kw
COP = RE/Wc
6 = RE/28.80
RE = 172.8/3.516 = 49.15 TR

A 23 tons refrigeration system has a heat rejected of 100 kw. Find the energy efficiency
ratio of the system.
13.42 *B. 14.42 C. 15.42 D. 16.42
Solution:
QR = RE + Wc
100 = 23(3.516) + W c
Wc = 19.132 kw
COP = RE/Wc = (23 x 3.516) / 19.132 = 4.32
EER = 3.412 COP = 3.412(4.23) = 14.42

A 200 mm x 250 mm, 8-cylinder, 4-stroke diesel engine has a brake power of 150 kw.
The mechanical efficiency is 80%. If two of the cylinders were accidentally cut off, what
will be the new friction power?
A. 31.50 kw B. 33.50 kw C. 35.50 kw *D. 37.50 kw
Solution:
em = BP/IP
0.8 = 150/IP
IP = 187.5 kw
FP1 = IP BP = 187.5 150 = 37.50 kw
FP1 = FP2 = 37.50 kw

If the energy efficiency ratio of the refrigeration system is 12.6, what is the COP of the
system?
*A. 3.69 B. 4.23 C. 5.92 D. 6.83
Solution:
EER = 3.412 COP
12.6 = 3.412 COP
COP = 3.69

An air compressor has a power of 40 kw at 4 % clearance. If clearance will increase to


70 %, what is the new power?
70 kw *B. 40 kw C. 53kw D. 60 kw
Solution:
The power of compressor will not be affected with the changes in clearance. Therefore
the power will still be 40 kw.

What is the approximate value of temperature of water having enthalpy of 208 Btu/lb?
138.67 C *B. 115.55 C C. 258.67 C D. 68.67 C
Solution:
h = F 32

Find the work posses for a Helium gas at 20C.


*A. 609 KJ/Kg B. 168 KJ/Kg C. 229 KJ/Kg D. 339 KJ/Kg
Solution:
W = m R T = m (8.314/M) T
For Helium, M = 4
W/m = (8.314/4)(20 + 273) = 609 KJ/Kg

Two kilogram of gas is confined in a 1 m3 tank at 200 kpa and 88C. What type of gas is
in the tank?
Helium *B. Ethane C. Methane D. Ethene
Solution:
PV=mRT
200 (1) = 2 (8.314/m)(88 + 273)
M = 30
Therefore: the gas is Ethane (C2H6)

Find the enthalpy of Helium if its internal energy is 200 KJ/kg.


144 KJ/kg B. 223.42 KJ/kg *C. 333.42 KJ/kg D. 168 KJ/kg
Solution:
R = 8.314/4 = 2.0785
K = 1.667 for helium
cp = k R/(k 1) = 1.667(2.0785)/(1.667 1) = 5.195 KJ/kg-K
cv = R/(k 1) = 2.0785/(1.667 1) = 3.116 KJ/kg-K
h/U = cp/cv
h/200 = 5.195/3.116
h = 333.42 KJ/kg

Compute the mass of a 2 m3 propane at 280 kpa and 40C.


6.47 kg B. 5.1 kg C. 10.20 kg *D. 9.47 kg
Solution:
Propane is C3H6------------------------M = 12(3) + 8(1) = 44
PV = m R T
280(2) = m (8.314/44)(40 + 273)
m = 9.47 kg

Compute the air flow in ft3/min of mechanical ventilation required to exhaust an


accumulation of refrigerant due to leaks of the system capable of revolving air from the
machinery room for a mass of 4 lbs refrigerant.
*A. 200 B. 210 C. 220 D. 230
Solution:
Q = 100 x G0.5 ft3/min
Q = 100 x (4)0.5 = 200 ft3/min
Compute the free-aperture cross section in m2 for the ventilation of a machinery room if
the mass of refrigerant is 9 kg.
0.314 *B. 0.414 C. 0.514 D. 0.613
Solution:
F = 0.138 G0.5 , m2
F = 0.138(9)0.5 = 0.414 m2

A 29.53 x 39.37 pressure vessel contains ammonia with f = 0.041. Compute the
minimum required discharge capacity of the relief device in kg/hr.
106.71 kg/hr B. 108.71 kg/hr *C. 110.71 kg/hr D. 112.71
kg/hr
Solution:
C = f D L, kg/s
C = 0.041(29.53/39.37)(39.37/39.37) = 0.03075 kg/s (3600) = 110.71 kg/hr

Compute the maximum length of the discharge pipe installed on the outlet of a
pressure-relief device in feet for internal pipe diameter of 0.5 inch and rated discharge
capacity is 8lb/min of air. The rated pressure of relief valve is 16 psig.
*A. 0.286 ft B. 0.386 ft C. 0.486 ft D. 0.586 ft
Solution:
P = Pg + Patm = 16 x 1.1 + 14.7 = 32.3 psia
L = 9P2d5/16Cr2 = 9(32.3)2(0.5)5/16(8)2 = 0.286 ft

A thermal power plant has a heat rate of 11,363 Btu/kw-hr. Find the thermal efficiency of
the plant.
28 % *B. 30 % C. 34 % D. 40 %
Solution:
eth = 3412/heat rate = 3412/11,363 = 30%

What is the hydraulic gradient of a 1 mile, 17 inches inside diameter pipe when 3300
gal/min of water flow with f. 0.03
*A. 0.00714 B. 0.00614 C. 0.00234 D. 0.00187
Solution:
v = (3300/7.481)/(/4)(17/12)2(60) = 4.66 ft/s
L = 1 mile = 5280 ft
hL =fLv2/2gD = 0.03(5280)(4.66)2/2(32.2)(17/12) = 37.7 ft
Hydraulic gradient = 37.7/5280 = 0.00714

Find the loss of head in the pipe entrance if speed of flow is 10 m/s.
5.10 m B. 10.2 m C. 17.4 m *D. 2.55 m
Solution:
Loss at entrance = 0.5 (v2/2g) = 0.5[102/ 2(9.81)] = 2.55 m

Wet material, containing 220 % moisture (dry basis) is to be dried at the rate of 1.5 kg/s
in a continuous dryer to give a product containing 10% (dry basis). Find the moisture
removed, kg/hr.
*A. 3543.75 kg/hr B. 3513.75 kg/hr C. 3563.75 kg/hr D.
3593.75 kg/hr
Solution:
Solid in wet feed = solid in dried product
[1/(1 + 2.2)](1.5) = [1/(1 + 0.1)](x)
x = 0.5156 kg/s (total dried product)
Moisture removed = 1.5 0.5156 = 0.984 kg/s = 3543.75 kg/hr

Copra enters a dryer containing 70% moisture and leaves at 7% moisture. Find the
moisture removed on each pound of solid in final product.
6.258 lb B. 1.258 lb C. 4.258 lb *D. 2.258 lb
Solution:
Solid in wet feed = solid in dried product
0.3x = 1
x = 3.333 lbs
1 = 0.93y
y = 1.07527 lb
Moisture removed = x y = 3.333 1.07527 = 2.258 lbs

A 1 m x 1.5 m cylindrical tank is full of oil with SG = 0.92. Find the force acting at the
bottom of the tank in dynes.
106. 33 x 103 dynes B. 106.33 x 104 dynes C. 106.33 x 105 dynes
6
*D. 106.33 x 10 dynes
Solution:
P = w h = (0.92 x 9.81)(1.5) = 13.5378 kpa
F = PA = 13.5378(/4 x 12) = 10.632 KN = 10,632.56 N x 10,000 dynes/N
F = 106.33 x 106 dynes

Find the pressure at the 100 fathom depth of water in kpag.


*A. 1,793.96 kpag B. 1,893.96 kpag C. 1,993.96 kpag D.
1,693.96 kpag
Solution:
H = 100 fathom x 6 = 600 ft
P = w h = (600/3.281)(9.81) = 1,793.96 kpag

Find the depth in furlong of the ocean (SG = 1.03) if the pressure at the sea bed is
2,032.56 kpag.
*A. 1 B. 2 C. 3 D. 4
Solution:
P=wh
2,032.56 = (1.03 x 9.81) h
h = 201.158 m x 3.281 ft/m x 1 yd/3ft x 1 furlong/220yd = 1 furlong

Find the mass of 10 quartz of water.


10.46 kg *B. 9.46 kg C. 11.46 kg D. 8.46 kg
Solution:
V = 10 quartz x 1gal/4quartz x 3,785 li/1gal x 1m3/1000 li
V = 0.0094625 x 10-3m3
w = m/V
1000 = m/0.0094625 x 10-3
m = 9.46 kg

Find the mass of carbon dioxide having a pressure of 20 psia at 200F with 10 ft 3
volume.
A. 1.04 lbs B. 1.14 lbs *C. 1.24 lbs D. 1.34 lbs
Solution:
PV = m R T
(20 x 144)(10) = m (1545/44)(200 + 460)
m = 1.24 lbs

Find the heat needed to raise the temperature of water from 30C to 100C with 60%
quality. Consider and atmospheric pressure of 101.325 kpa. Use the approximate
enthalpy formula of liquid.
A. 293.09 KJ/kg B. 1,772.90 KJ/kg C. 1,547.90 KJ/kg *D.
1,647.29 KJ/kg
Solution:
At 100C
hf = Cp t = 4.187(100) 418.7 KJ/kg
hfg = 2257 KJ.kg
h2 = hf + xhfg = 418.7 + 0.60(2257) = 1,772.9 KJ/kg
Q = 1(4.187)(100-30) + 1(1772.9 418.7) = 1,647.20 KJ/kg

Find the enthalpy of water at 212F and 14.7 psi if the dryness factor is 30%. Use the
approximate enthalpy formula of liquid.
461 Btu/lb *B. 471 Btu/lb C. 481 Btu/lb D. 491 Btu/lb
Solution:
hf = (F 32) = (212 32) = 180 Btu/lb
hfg = 970 Btu/lb
h = hf + x hfg
h = 180 + 0.3(970) = 471 Btu/lb

An air compressor consumed 1200 kw-hr per day of energy. The electric motor driving
the compressor has an efficiency of 80 %. If indicated power of the compressor is 34
kw, find the mechanical efficiency of the compressor.
117.65 % B. 75% *C. 85% D. 90%
Solution:
Pim = 1200kw-hr/24 hrs = 50 kw
BP = 50(0.80) = 40 kw
em = 34/40 = 85%
A refrigeration system consumed 28,800 kw-hr per month of energy. There are 20 % of
energy is lost due to cooling system of compressor and motor efficiency is 90%. If COP
of the system is 6, find the tons of refrigeration of the system.
43.15 TR B. 46.15 TR *C. 49.15 TR D. 41.15 TR
Solution:
Pim = 28,800/(24 x 30) = 40 kw
BP = 40 (0.90) = 36 kw
Wc = 36(1 0.20) = 28.80 kw
COP = RE/Wc
6 = RE/28.80
RE = 172.8/3.516 = 49.15 TR

A 23 tons refrigeration system has a heat rejected of 100 kw. Find the energy efficiency
ratio of the system.
13.42 *B. 14.42 C. 15.42 D.16.42
Solution:
QR = RE + Wc
100 = 23(3.516) + W c
Wc = 19.132 kw
COP = RE/Wc = (23 x 3.516) / 19.132 = 4.23
EER = 3.412 COP = 3.412(4.23) = 14.42

A 200 mm x 250 mm, 8-cylinder, 4-stroke diesel engine has a brake power of 150 kw.
The mechanical efficiency is 80 %. If two of the cylinders were accidentally cut off, what
will be the new friction power?
31.50 kw B. 33.50 kw C. 35.50 kw *D. 37.50 kw
Solution:
em = BP/IP
0.8 = 150/IP
IP = 187.5 kw
FP1 = IP BP = 187.5 150 = 37.50 kw
FP1 = FP2 = 37.50 kw

If the energy efficiency ratio of the refrigeration system is 12.6, what is the COP of the
system?
*A. 3.69 B. 4.23 C. 5.92 D. 60 kw
Solution:
EER = 3.412 COP
12.6 = 3.412 COP
COP = 3.69

An air compressor has a power of 40 kw at 4% clearance. If clearance will increase to


7%, what is the new power?
70 kw *B. 40 kw C. 53 kw D. 60 kw
Solution:
The power of compressor will not be affected with the changes in clearance. Therefore
power will still be 40 kw.

What is the approximate value of temperature of water having enthalpy of 208 Btu/lb?
138.67C *B. 115.67C C. 258.67C D. 68.67C
Solution:
h = F 32
208 = F 32
F = 240 F = 115.55 C

Convert 750R to K
390.33 K B. 395.33 K C. 410.33 K *D. 416.33 K
Solution:
R = 1.8 K
750 = 1.8 K
K = 416.667

An otto cycle has a compression ratio of 8. Find the pressure ratio during compression.
*A. 18.38 B. 16.38 C. 14.38 D. 12.38
Solution:
P1V1k = P2V22
(V1/V2)k = (P2/P1)
rkk = rp
rp = (8)1.4 = 18.38

A diesel cycle has a cut off ratio of 2.5 and expansion ratio of 4. Find the clearance of
the cycle.
9.11 % B. 5.55 % *C. 11.11 % D. 15.15 %
Solution:
rk = rc re
rk = 2.5(4) = 10
rk = (1 + c)/c
10 = (1 + c)/c
c = 11.11 %

A dual cycle has an initial temperature of 30 C. The compression ratio is 6 and the heat
addition at constant volume process is 600 KJ/kg. If cut-off ratio is 2.5, find the
maximum temperature of the cycle.
3638.50 C *B. 3365.50 C C. 3565.50 C D. 3965.50 C
Solution:
T2 = T1 rkk-1 = (30 +273)(6)1.4-1 = 620.44 K
QAV = m cv (T3 T2)
600 = 1(0.7186)(T3 620.44)
T3 = 1455.396 K
rc = T4/T3
2.5 = T4/1455.396
T4 = 3638.49 K = 3365.50 C

A three stages air compressor compresses air from 100 kpa to 1000 kpa. Find the
intercooler pressure between the first and second stage.
505.44 kpa B. 108.44 kpa C. 316.23 kpa
*D. 215.44 kpa
Solution:
Px = (P12P2)1/3
Px = [(100)2(1000)]1/3 = 215.44 kpa

A 10-stages air compressor compresses air from 100 kpa to 800 kpa. Find the
intercooler pressure between 1st and 2nd stage.
282.84 kpa B. 113.21 kpa *C. 123.11 kpa D.
333.51 kpa
Solution:
Px = (P1s-1P2)1/s
Px = [(100)10-1(1000)]1/10 = 123.11 kpa

A 3-stages air compressor compresses air from 100 kpa to 700 kpa. Find the intercooler
pressure between the 2nd and 3rd stage.
*A. 365.88 kpa B. 375.88 kpa C. 385.88 kpa
D. 395.88 kpa
Solution:
Px = (P12P2)1/3
Px = [(100)2(700)]1/3 = 191.28 kpa
Px/P1 = Py/Px
Py = Px2/P1 = (191.28)2/100 = 365.88 kpa

Carnot cycle A, B and C are connected in series so that the heat rejected from A will be
the heat added to B and heat rejected from B will be added to C, each cycle operates
between 30 C and 400 C. If heat added to A is 1000 kw, find the work output of C.
*A. 111.44 kw B. 549.78 kw C. 247.53 kw D. 141.89 kw
Solution:
e1 = e2 = e3 = (400 30)/(400 + 273) = 54.98 %
e1 = W 1/QA1 = (QA1 QR1)/QA1
0.5498 = (1000 QR1)/1000
QR1 = 450.22 = QA2
0.5498 = (450.22 QR2)/450.22
By heat balance:
Qgain = Qloss
mw cp (tb- ta) = mg cpg (t2-t1)
(0.30)(4.187)( tb 15) = (0.5)(1.0717)(150 80)
tb = 44.86 oC

A 350 mm X 450 mm steam engine running at 280 rpm has an entrance steam
condition of 2 Mpa and 230 oC and exit at 0.1 Mpa. The steam consumption is 2,000
kg/hr and mechanical efficiency is 85%. If indicated mean effective pressure is 600
Kpa, determine brake thermal efficiency.
At 2 Mpa and 230 oC (Table 3): h1 = 2849.6 s1 = 6. 4423
At 0.1 Mpa: sf = 1.3026 hf = 417.46 sfg = 6.0568 hfg = 2258 hf2 = 417.46
KJ/kg
A. 23.34% *B. 15.25% C. 14.16% D. 27.34%
SOLUTION:
VD = 2[3.1416/4 (0.35) 2 (0.45)(280/60)]= 0.4041 m3/sec
Indicated Power = Pmi x VD = 600 x 0.4041 =242.45 KW
Brake Power = IP (em) = 242.45 (0.85) = 206.08KW
Brake Power 206.08 = 15.25%
etb = ms (h1-hf2) = (2000/3600)(2849.6 417.46)

A steam turbine receives 5,000 kg/hr of steam at 5 Mpa and 4000oc and velocity of 30
m/sec. It leaves the turbine at 0.06 Mpa and 85% quality and velocity of 15 m/sec.
Radiation loss is 10,000 KJ/hr. Find he KW developed.
At 5 Mpa and 400oC: h1 = 3195.7 KJ/kg s1 =6.6459
At 0.006 Mpa: hf = 151.53 hfg = 2415.9
A. 1273.29 B. 2173.29 *C. 1373.60 D. 7231.29
SOLUTION:
h2 = hf + xhfg = 151.53 + 0.85(2415.9) = 2205.045 KJ/ kg
KE1 = m v2 = (5,000/3600)(30)2 = 625 W = 0.625 KW
KE2 = m v2 = (5,000/3600)(15)2 = 156.25 W = 0.15625 KW
By energy balance:
KE1 + mh1 = KE2 + mh2 + Q + W
W = (KE1 KE2) + m(h1-h2) Q
5000 10,000
W = (0.625 0.156) + (3600)(3195.7 2205.045) 3600 = 1373.60 KW

A steam turbine with 85% stage efficiency receives steam at 7 Mpa and 550oC and
exhausts as 20 Kpa. Determine the turbine work.
At 7 Mpa and 550oC: h1 = 3530.9 Kj/kg s1 = 6.9486
At 20 Kpa (0.020 Mpa): sf = 0.8320 hf = 251.4 sfg = 7.0766 hfg = 2358.3
1,117 KJ/kg B. 1,132 KJ/ kg C. 1,123.34 KJ/ kg *D. 1,054.95 KJ/kg
SOLUTION:
s1 = s2 = sf + xsfg
6.9486 = 0.8320 + x(7.0766)
x = 0.8643
h2 = 251.40 + 0.8643(2358.3) = 2289.78 KJ/kg
h1 - h2a
eST = h1 h2
0.85 =3530.9 h2a
3530.9 2289.78
h2a = 2475.95 KJ/kg
WT = h1 = h2a = 3530.9 2475.95 = 1,054.95 KJ/kg
A steam turbine with 80% stage efficiency receives steam at 7 Mpa and 550oC and
exhaust as 20 Kpa. Determine the quality at exhaust.
At 7 Mpa and 550oC: h1 = 3530.9 Kj/kg s1 = 6.9486
At 20 Kpa (0.020 Mpa): sf = 0.8320 hf = 251.4
*A. 96.96% B. 76.34% C. 82.34% D. 91.69%
SOLUTION:
sfg = 7.0766 hfg = 2358.3
s1 = s2 = sf + sfg
6.9486 = 0.8320 + x(7.0766)
x = 0.8643
h2 = 251.40 + 0.8643(2358.3) = 2289.78 KJ/kg
nST = h1 h2a
h1 h2
0.80 = 3530.9 h2a
3530.9 2289.78
h2a = 2538.004 KJ/kg
h2a = hf + x hfg
2538.004 = 251.40 + x (2358.3)
x = 96.96%

A 16,000KW geothermal plant has a generator efficiency and turbine efficiency of 90%
and 80%., respectively if the quality after throttling is 20% and each well discharges
400, 000 kg/hr, determine the number of wells are required to produce if the charge of
enthalpy if the change of enthalpy at entrance and exit of turbine is 500KJ/kg.
4 wells *B. 2 wells C. 6 wells D. 8 wells
SOLUTION:
WT = ms(h3 h4)
16,000 = ms (500)
0.9(0.8)
ms = 44.44 kg/sec
ms = 160,000 kg/hr
160,000 = 0.20 mg
mg = 800,000 kg/hr
No. of wells = 800,000/400,000 = 2 wells
A liquid dominated geothermal plant with a single flash separator receives water at
204oC. The separator pressure is 1.04 Mpa. A direct contact condenser operates at
0.034 Mpa. The turbine has a polytropic efficiency of 0.75. For a cycle output of 60 MW,
what is the mass flow rate of the well-water in kg/s?
At 204oC: hf = 870.51 KJ/kg
At 1.04 Mpa: hf = 770.38 hfg = 2009.2 hg = 2779.6 sg = 6.5729
At 0.034 MPa: hf = 301.40 hfg = 2328.8 sf = 0.9793 sfg = 6.7463
*A. 2,933 B. 2,100 C. 1,860 D. 2,444
SOLUTION:
h3 = hg at 1.04 MPa = 2779.6 KJ/kg
Solving for h4:
s3 = s4 = sf + xsfg
6.5729 = 0.9793 + x4(6.7463)
x4 = 0.829
h4 = 301.4 + 0.829(2328.8) = 2232.3 KJ/kg
WT = ms (h3 h4)
60,000 = ms (2779.6 2232.3) 0.75
ms = 146.17 kg/sec
Solving for x2: (h1 = h2)
h1 = h2 = hf + xhfg
870.51 = 770.38 + x2(2009.2)
x2 = 0..049836
ms = x mg
146.17 = 0.049836 mg
mg = 2,933.06 kg/sec

An engine-generator rated 9000 KVA at 80% power factor, 3 phase, 4160 V has an
efficiency of 90%. If overall plant efficiency is 28%, what is the heat generated by the
fuel.
18,800 KW B. 28,800 KW C. 7500 KW *D. 25,714 KW
SOLUTION:
Gen. Output = pf x KVA = 0.8 x 9000 = 7200 KW
eoverall= Gen. Output
Qg

0.28 = 7200/Qg
Qg = 25,714.28 KW

The indicated thermal efficiency of a two stroke diesel engine is 60%. If friction power is
15% of heat generated, determine the brake thermal efficiency of the engine.
43% *B. 45 % C. 36% D. 37%
SOLUTION:
ne = IP/ Qg
0.60 = IP/Qg
IP = 0.60 Qg
BP = IP- FP = 0.60Qg 0.15Qg = 0.45Qg
etb = BP/Qg = 0.45Qg/Qg = 45%

A 305 mm x 457 mm four stroke single acting diesel engine is rated at 150 KW at 260
rpm. Fuel consumption at rated load is 0.56 kg/KW-hr with a heating value of 43,912
KJ/kg. Calculate brake thermal efficiency
10.53% B. 27.45% *C. 14.64% D. 18.23%
SOLUTION:
mf = 0.56 kg/KW-hr x 150 KW = 84 kg/hr = 0.0233 kg/sec
Brake thermal efficiency =

A waste heat recovery boiler produces 4.8 Mpa(dry saturated) steam from 104C
feedwater. The boiler receives energy from 7 kg/sec of 954C dry air. After passing
through a waste heat boiler, the temperature of the air is has been reduce to 343C.
How much steam in kg is produced per second? Note: At 4.80 Mpa dry saturated, h =
2796.
1.30 B. 0.92 *C. 1.81 D. 3.43
SOLUTION:
hf = approximate enthalpy of feedwater
hf = Cpt
hf = 4.187(104)
hf = 435.45 KJ/kg
Heat loss = Heat gain
m gc p(t 1 - t 2) = m s(h - h f)
7(1.0)(954 343) = ms(2796.0 436.45)
m s = 1.81 kg/sec

A diesel electric plant supplies energy for Meralco. During a 24-hour period, the plant
consumed 240 gallons of fuel at 28C and produced 3930 KW-hr. Industrial fuel used is
28API and was purchased at P30 per liter at 15.6C. What is the cost of the fuel be to
produce one KW-hr?
*A. P6.87 B. P1.10 C. P41.07 D. P5.00
SOLUTION:
SG 15.6C = 141.5/(131.5 + 28) = 0.887
Density at 15.6C = 0.887(1kg/li) = 0.887 kg/li
SG 28C = 0.887[1-.0007(1 15.6)] = .879
Density at 28C = 0.879(1 kg/li) = 0.879 kg/li
V28C / V15.6C = SG15.6C / SG28C
240 / V15.6C = 0.887 / 0.879
V15.6C = 237.835 gallons x 3.785 li/gal = 900.21 li
Cost = [(30)(900.21)] / 3930 = P6.87/KW-hr

In a gas turbine unit, air enters the combustion chamber at 550 kpa, 277C and 43 m/s.
The products of combustion leave the combustor at 511 kpa, 1004C and 180 m/s.
Liquid fuel enters with a heating value of 43,000 KJ/kg. For fuel-air ratio of 0.0229, what
is the combustor efficiency of the unit in percent?
70.38% B. 79.385% *C. 75.38% D. 82.38%
SOLUTION:
Heat supplied by fuel = mfQh = 0.0229(43,000) = 984.7 KJ/kg air
Q = heat absorbed by fuel
Q/m = Cp(T2 T1) + (V22 V12)
Q/m = (1.0)(1004 277) + [(180) 2 (43) 2]/1000 =742.28 KJ/kg air
Combustor Efficiency = = 75.38%

The specific speed of turbine is 85 rpm and running at 450 rpm. If the head is 20 m and
generator efficiency is 90%, what is the maximum power delivered by the generator.
450.51 KW B. 354.52 KW C. 650.53 KW *D. 835.57
KW
SOLUTION:
NS = (NHP)/h5/4
85 = (450HP)/(20 x 3.281) 5/4
Hp = 1244.52
Generator Output = (1244.52 x 0.746)(0.9) = 835.57 KW

In Francis turbine, the pressure gage leading to the turbine casing reads 380 Kpa. The
velocity of water entering the turbine is 8 m/sec, if net head of the turbine is 45 m, find
the distance from center of spiral casing to the tailrace.
*A. 3.0 m B. 3.5 m C. 4.0 m D. 4.5m
SOLUTION :

h = V2/2g
45 = (380/9.81) + z + [82/(2 x 9.81)]
z=3m

A turbine has a mechanical efficiency of 93%, volumetric efficiency of 95% and total
efficiency of 82%. If effective head is 40 m, find the total head.
48.72 m B. 40.72 m *C. 36.22 m D. 34.72 m
SOLUTION:
eT = emehev
0.8 = 0.93(eh)(.95)
h = 0.9055
Total head = h eh = (40)(0.9055) = 36.22 m

A Pelton type turbine has 25 m head friction loss of 4.5 m. The coefficient of friction
head loss (from Moorse) is 0.00093 and penstock length of 80 m. What is the penstock
diameter?
*A. 1,355.73 mm B. 3,476.12 mm C. 6771.23 mm D.
1686.73 mm
SOLUTION:
h =25- 4.5 = 20.5
v = (2gh) = [(2 x 9.81 x 20.5)1/2] = 20.55 m/sec
hL = (2fLv2)/gD
4.5 = (2)(0.00093)(80)(20.055)2 / 9.81D
D = 1,355,730 m = 1,355.73 mm

In an 9,000 KW hydro-electric plant the over-all efficiency is 88% and the actual power
received by the customer is 110,000 KW-hrs for that day. What is the secondary power
could this plant deliver during the entire day?
58,960 KW-hrs *B. 80,080 KW-hrs C. 65,960 KW-hrs D. 70,960 KW-hrs
SOLUTION:
Plant Capacity = 9,000(0.88)(24) = 190,080 KW-hrs
Secondary Power = 190,080 110,000 = 80,080 KW-hrs

A Pelton type turbine was installed 30 m below the gate of the penstock. The head loss
due to friction is 12 percent of the given elevation. The length of penstock is 100 m and
coefficient of friction is 0.00093. Determine the power output in KW. ( Use Moorse
equation)
22,273 B. 23,234 C. 32,345 *D. 34,452
SOLUTION:
hL = 0.12(30) = 3.6 m
h = 30 3.6 = 26.40 m
v = (2gh)1/2 = [(2)(9.81)(26.4)]1/2 = 22.759 m/sec
hL= (2fLv2)/gD
3.6 = (2 x .00093 x 100 x 22.759) / (9.81D)
D = 2.728 m
Q = A x v = [2](22.759) = 133.03 m3/sec
Power = w Q h = 9.81(133.03)(26.4) = 34,452 KW

Water flows steadily with a velocity of 3.05 m/s in a horizontal pipe having a diameter of
25.4 cm. At one section of the pipe, the temperature and pressure of the water are 21C
and 689.3 Kpa, respectively. At a distance of 304.8 m downstream

A hydro electric plant having 30 sq. km reservoir area and 100 m head is used to
generate power. The energy utilized by the consumers whose load is connected to the
power plant during a five-hour period is 13.5 x 106 kwh. The overall generation
efficiency is 75%. Find the fall in the height of water in the reservoir after the 5-hour
period.
5.13 m B. 1.32 m C. 3.21 *D. 2.20 m
SOLUTION
Energy Output = Power x time = (w Q h) x time
13.5 x 106 = 9.81(Q)(100)(0.75)(5)
Q = 3669.725 m3/s
Volume after 5 hrs = 3669.725(5 x 3600) = 66,055,050 m 3
Volume = A x height
66,055,050 = (30 x 106) h
H =2.202 m

The gas density of chimney is 0.75 kg/m3 and air density of 1.15 kg/m3. Find the driving
pressure if the height of chimney is 63.71 m.
0.15 kpa *B. 0.25 kpa C. 0.35 kpa D. 0.45 kpa
SOLUTION:
hw = H(da dg) = 63.71(1.15 0.75) (0.00981) = 0.25 kpa
The actual velocity of gas entering in a chimney is 8 m/sec. The gas temperature is 25C
with a gas constant of 0.287 KJ/kg-K. Determine the gas pressure for a mass of gas is
50,000 kg/hr and chimney diameter of 1.39m.
95 kpa *B. 98 kpa C. 101 kpa D. 92 kpa
SOLUTION:
Vg = A x v = / 4 (1.39)2(8) = 12.139 m3/s
PgVg = mgRgTg
P(12.139) = (50,000/3600)(.287)(25 +273)
P = 97.85 kpa
A steam generator with economizer and air heater has an overall draft loss of 25.78 cm
of water. If the stack gases are at 177C and if the atmosphere is at 101.3 Kpa and 26C,
what theoretical height of stack in meters is needed when no draft fan are used?
Assume that the gas constant for the flue gases is the same as that for air.
A 611.10 B. 631.10 *C.651.10 D.671.10
SOLUTION:
w = P/RT
da = (101.325)/[(.287)(26 + 273)] = 1.180 kg/m3
dg = (101.3)/[(0.287)(177 +273)] = 0.784 kg/m3
Draft = (0.2578)(1000) = 257.80 kg/m3
Draft = H(da dw)
257.80 = H(1.18 0.784)
H = 651.10 m

A foundation measures 12 ft x 14 ft x16 ft. Find the number of sacks of cement needed
for 1:2:4 mixture.
302 B. 404 C. 356 *D. 598
SOLUTION:
V = 12 X 14 X 16 = 2,688 ft3 (1 yd3 / 33 ft3) = 99.55 yd3 of concrete
For every 1 yd3 concrete, it needs 6 sacks of cement
Therefore:
No. of sacks = 6(99.55) = 597.33 sacks or 598 sacks
A rectangular foundation cross-section has a bed plate dimension of 8 ft x 10 ft. The
uniform clearance on each side is 1 ft. The height of foundation is 4.5 ft. If the weight of
the steel bar reinforcements needed is 1/2% of weight of foundation, find the weight of
steel bars. Use concrete density of 2400 kg/m3 .
173.47 kg *B. 183.47 kg C. 163.47 kg D. 153.47 kg
SOLUTION:
A = (8 + 2) (10 + 2) = 120 m2
V = Ah = 120(4.5) = 540 ft3 = 15.29 m3
W = wV = (2400)(15.29) = 36,693.25 kg
Weight of steel bars = (1/2%) W f = 0.005(36,693.25) = 183.47 kg

A steam pipe having a surface temperature of 250C passes through a room where the
temperature is 27 C. The outside diameter of pipe is 100 mm and emissivity factor is
0.8. Calculate the radial heat loss for 3 m pipe length.
1434.7 W B. 37.46 W *C. 2651.82 W D. 3545.45 W
SOLUTION:
A = DL = = 0.425m2
Solving for heat due to radiation:
Tg = 250 +273 = 523K
T2 = 27 +273 = 300K
Qa = 20,408.4 x 104 AF(T14 T24), J/hr = 20,408.4 x 104(0.8)(0.7539)[(523)4
(300)4]
Qr = 10,266,539.06 j/hr x 1hr/3600sec = 2851.82 W
Brine enters a circulating brine cooler at the rate of 60 m 3/hr at -*C and leaves at -18C.
Specific heat of brine is 1.072 KJ/kg-K and specific gravity of 1.12. Determine the tons
of refrigeration.
53.5 TR B. 65.3 TR C.33.5 TR *D. 56.9 TR
SOLUTION:
Density of brine = 1.12(1000 kg/m3) = 1120 kg/m3
m = (1120)(60)/3600 = 18.67 kg/sec
Q = mcp = 18.67(1.072)(-8 + 18) = 200.11 KW
TR = 200.11/3.516 = 56.91 Tons of refrigeration
A turbo-charged, 16 cylinder, Vee-type diesel engine has an air consumption of 3,000
kg/hr per cylinder at rate load and speed. This air is drawn in through a filter by a
centrifugal compressor directly connected to the exhaust gas turine. The temperature of
the air from the compressor is 135C and a counter flow air cooler reduces the air
temperature to 45C before it goes to the engine suction heater. Cooling water enters air
cooler at 30C and leaves at 40C. Calculate the log mean temperature difference.
47.23C B. 87.82C *C. 43.34C D. 65.24C
SOLUTION:
a = 45-30 = 15C
b = 135 40 = 95C
mean = [a - b] / [ln(ab)] = [95-15] / ln(95/15) = 43.34C
Water is flowing in a pipe with radius of 30 cm at a velocity of 5 m/s at the temperature
in pipe. The density and viscosity of the water are as follows: density 997.9 kg/sec
viscosity = 1.131 Pa-s. What is the Reynolds Number for this situation?
*A. 2647 B. 96.2 C. 3100 D. 1140
SOLUTION:
n = Dvg / v
Where:
D = 2(0.30) = 0.60 m
vg = 5 M/SEC
v = 1.131/997.9 = 0.0011334 m2 / sec
Nm = 0.60(5)/0.0011334 = 2,647
Compute the amount of condensate form during 10 minutes warm-up of 180 meter pipe
conveys the saturated steam with enthalpy vaporization hfg = 1,947.8 KJ/kg. The
minimum external temperature of pipe is 2C. The final temperature of pipe is 195C. The
specific heat of pipe material is 0.6 KJ/kg-C. The specific weight is 28 kg/m.
249.69 kg B. 982.45 kg *C. 299.64 kg D. 423.45 kg
SOLUTION:
mp = mass of pipe = 28(180) = 5,040 kg
Heat Loss by steam = Heat loss from pipe
m(hg - hf) = mpcp (t2 t1)
m(1947.8) = (5040)(0.6)(195-2)
m = 299.64 kg
The discharge pressure of an air compressor is 5 times the suction pressure. If volume
flow at suction is 0.1 m3/sec, what is the suction pressure if compressor work is 19.57
kw? (use n=1.35)
97 kpa *B.98 kpa C. 99 kpa D.100 kpa
SOLUTION:
W = [(P2/P1)n-1/n 1]
19.57 = 1.35(P1)(0.1)/(1.35-1)[(5)1.35-1/1.35 1]
P1 = 98 KPa

The initial condition of air in an air compressor is 98 KPa and 27C and discharge air at
450 KPa. The bpre and stroke are 355 mm and 381 mm, respectively with percent
cleared of 8% running at 300 rpm. Find the volume of air at suction.
541.62 m3/hr B. 551.62 m3/hr C. 561.62 m3/hr *D. 571.62 m3/hr
SOLUTION:
ev = 1 + c c(P2/P1)1/n = 1 + 0.08 - 0.08(450/98)1/1.4 = 0.842
VD = D2 LN = (0.355)2 (0.381)(300/60) = 0.1885 m3/sec
V1 = 0.1885(0.842) = 0.15878 m3/sec = 571.62 m3/hr
An air compressor has a suction volume of 0.35 m3/sec t 97 KPa and discharges to 650
KPa. How much power saved by the compressor of there are two stages?
18.27 KW B. 16.54 KW C. 13.86 KW *D. 11.58 KW
SOLUTION:
W = [(P2/P1)n-1/n 1] = (1.4 x 97 x 0.35)/(1.4 -1) [(650/97)1.4-1/1.4 1] = 85.79 KW
For two stages :
Px = (P1P2)1/2 = (97 x 650)1/2 = 251.097 KPa
W = [(Px/P1)n-1/n 1] = 2(1.4)(97)(0.35)/(1.4 1) [(251.0.97/97)1.4-1/1.4 1] =
74.208 KW
POWER SAVED = 85.79 74.208 = 11.582 KW

A twop stage air compressor has an intercooler pressure of 4 kg/cm2. What is the
discharge pressure if suction pressure is 1 kg/cm2?
3 kg/cm2 B. 9 kg/cm2 C. 12 kg/cm2 *D. 16 kg/cm2
SOLUTION:
Px = (P1P2)1/2
Px2 = P1(P2)
42 = 16 kg/cm2

A two stage air compressor compresses air at 100 KPa and 22C discharges to 750
KPa. If intercooler intake is 105C. Determine the value of n.
1.400 *B. 1.325 C. 1.345 D. 1.288
SOLUTION:
Px = (100 x 750)1/2 =273.86 KPa
Tx/T1 = (Px/P1)n-1/n
(105 + 273)/(22 + 273) = (273.86/100)n-1/n
1.281 = (2.6268)n-1/n
n = 1.326
A single acting compressor has a volumetric efficiency of 89%, operates at 500 rpm. It
takes in air at 100 KPa and 30C and discharges it at 600 KPa. The air handled is 8
m3/min measured at discharge condition. If compression is isentropic, find mean
effective pressure in KPa.
*A. 233.34 B. 973.17 C. 198.34 D. 204.82
SOLUTION:
P1V1K = P2V2K
100(V11.4) = 600(6)1.4
V1 = 28.768 m3/min
VD = 28.768/0.89 = 32.32 m3/min
W = n P1V1/n-1 x [(P2 / P1)n-1/n 1] = [(1.4 x 100 x 32.32)/(1.4 1)] x
[(600/100)1.4-1/1.4 1]
W = 7562.19 KJ/min
W = Pm x Vd
7562.19 = Pm x 32.32
Pm = 233.34 KPa
A water-jacketed air compressed handles 0.343 m3/s of air entering at 96.5 KPa and
21C and leaving at 460 KPa and 132C; 10.9 kg/h of cooling water enters the jacket at
15C and leaves at 21C. Determine the compressor brake power.
26.163 KW *B. 62.650 KW C. 34.44 KW D. 19.33 KW
SOLUTION:
T2/T1 = (P2/P1) n-1/n
(132+273) / (21+273) = (480/96.5)n-1/n

n = 1.249
W = (1.249 x 96.5 x 0.343) / (1.249-1) [(480 / 96.5)1.249-1/1.249 1]
W = 62.57 KW
Q = heat loss = mcp(t2 t1) = (10.9/3600)(4.187)(21 15) 0.075 KW
Brake power = W + Q = 62.57 + 0.076 = 62.65 KW

A double suction centrifugal pumps delivers 20 ft3/sec of water at a head of 12 m and


running at 650 rpm. What is the specific speed of the pump?
5014.12 rpm B. 6453.12 rpm *C. 2770.73 rpm D. 9966.73 rpm
SOLUTION:
N = N(Q)1/2 / h3/4
Q = 20/2 ft3/sec x 7.481 gal/ft3 x 60 sec/1min = 4,488.6 gal/min
h = 12 x 3.281 = 39.37 ft
N = (650 x (4,488.6)1/2)/(39.37)3/4
N = 2,770.73 rpm
Determine the number of stages needed for a centrifugal pump if it is used to deliver
400 gal/min of water and pump power of 15 Hp. Each impeller develops a head of 30 ft.
6 B. 4 *C. 5 D. 7
SOLUTION:
Wp = w Q h
15 x 0.746 = 9.81(400 gal/min x 0.00785m3/gal x 1/60)h
h = 45.20 m x 3.281 ft/m = 148.317 ft
Number of stages = 148.317/40 = 4.94 stages = 5 stages
The suction pressure of a pump reads 3 in. of mercury vacuum and discharge pressure
reads 140 psi is use to deliver 120 gpm of water with specific volume of 0.0163 ft 3/lb.
Determine the pump work.
4.6 KW B. 5.7 KW *C. 7.4 KW D. 8.4 KW
SOLUTION:
P1 = -3 in Hg x 101.325/29.92 = -10.16 KPa
P2 = 140 psi x 101.325/14.7 = 965 KPa
w = 1/v = 1/0.163 = 61.35 lb/ft3 x 9.81/62.3 = 9.645 KN/m3
h = (P2 P1)/w = (965 +10.16)/9.645 = 101.105 m
Q = 120 gal/min x 3.785/1gal x 1m3/1000li x 1/60 = 0.00757 m3/sec
P = w Q h = 9.645(0.00757)(101.105) = 7.38 KW
A submersible pump delivers 350 gpm of water to a height of 5 ft from the ground. The
pump were installed 150 ft below the ground level and draw down of 8 ft during the
operation. If water level is 25 ft above the pump, determine the pump power.
7.13 KW B. 4.86 KW C. 7.24 KW *D. 9.27 KW
SOLUTION:
h = 5 + 150 (25 8) = 138/3.281 = 42.06 m
Q = 350 gal/min x 0.003785 m3/gal x 1 min/60sec = 0.02246 m3/sec
Wp = w Q h = 9.81(0.02246)(42.06) = 9.27 KW
A vacuum pump is used to drain a flooded mine shaft of 20 water. The pump pressure
of water at this temperature is 2.34 KPa. The pump is incapable of lifting the water
higher than 16 m. What is the atmospheric pressure?
*A. 159.30 B. 32.33 C. 196.22 D. 171.9
SOLUTION:
Using Bernoullis Theorem:

P1/w + V12/2g + z1 = P2/w + V2/2g + z2


P1/w = P2/w + (V22 - V12)/2g + (z2 - z1)
P1/9.81 = 2.34/9.81 + 0 + 16
P1 = 159.30 KPa
A submersible, multi-stage, centrifugal deep well pump 260 gpm capacity is installed in
a well 27 feet below the static water level and running at 3000 rpm. Drawdown when
pumping at rated capacity is 10 feet. The pump delivers the water into a 25,000 gallons
capacity overhead storage tank. Total discharge head developed by pump, including
friction in piping is 243 feet. Calculate the diameter of the impeller of this pump in inches
if each impeller diameter developed a head of 38 ft.
3.28 B. 5.33 *C. 3.71 D. 6.34
SOLUTION:
V= DN
V=
D (3000/60) = (2(32.2)(38))1/2
D = 0.315 ft = 3.708 inches
A fan pressure of 2.54 cm of water t 1.42 m3 per second of air at static pressure of 2.54
cm of water through a duct 300 mm diameter and discharges it through a duct 275 mm
diameter. Determine the static fan efficiency if total fan mechanical is 75% and air
measured at 25 and 60 mm Hg.
50.11% *B. 53.69% C. 65.67% D. 45.34%
SOLUTION:
wA = P/RT = 101.325/(0.287)(25 + 273) = 1.18 kg/m3
hA = hwww/wA = (0.0254)(1000)/1.18 = 21.52 m
vA = 1.42/(/4)(0.3)2 = 20.09 m/s
Vd = 1.42/(/4)(0.275)2 = 23.9 m/s
hv = (23.9)2 (20.09)2 / 2(9.81) = 8.54 m
h = ha + hv = 21.52 + 8.54 = 30.06 m
eT = wa Q h/BP
0.75 = (1.18 x 0.00981)(1.42)(30.06) / BP
BP = 0.6588 KW
ep = wa Q hs/BP = (1.18 x 0.00981)(1.42)(21.52) / 0.6588 = 53.69%
A water cooler uses 50 lb/hr of melting ice to cool running water from 80 to 42. Based
on te inside coil area, U1 = 110 Btu/hr-ft2-. Find the gpm of water cooled.
0.10 GPM B. 0.21 GPM *C. 0.38 GPM D. 0.45 GPM
SOLUTION:
Q = mf L = mwcpw(t1 t2)
50 (144) = mW(1)(80-42)
mw = 189.474 lb/hr
V = (189.474/62.4) (7.48/60) = 0.38 GPM
The charge in a Diesel engine consists of 18.34 grams of fuel, with lower heating value
of 42,571 KJ/kg, and 409 grams of fuel and products of combustion. At the beginning of
compression, t1 = 60. Let rk = 14. For constant cP = 1.11 KJ/kg-C, what should be the
cut-off ratio in the corresponding ideal cycle?
2.05 B. 2.34 C. 5.34 *D. 2.97
SOLUTION:
QA = mfQh = 0.018.34(42,571) = 780,752 KJ
T2/T1 = rkk-1
T2 = (60 + 273)1.4-1 = 956.964K
mt + mg = 409
mt + ma + mf = 409
ma = 409 2(18.34) = 372.32 grams
QA = macp(t3 t2)
780.752 = 0.37232(1.11)(T3 956.964)
T3 = 2846,146
rC = T3/T2 = 2846.146/956.964 = 2.97

The gain of entropy during isothermal nonflow process of 5 lb of air at 60 is 0.462 Btu/R.
Find the V1/V2.
3.85 *B. 0.259 C. 1.0 D. 0.296
SOLUTION:
s = m R T ln(V2/V1)
0.462 = 5 (53.33/778) ln (V2/V1)
V2/V1 = 3.85
V1/V2 = 1/3.85 = 0.259

An auditorium seating 1500 people is to be maintained at 80 dry bulb and 85 wet bulb
temperature when outdoor air is at 91 dry bulb and 75 wet bulb. Solar heat load is
110,000 Btu/hr and supply air at 60 determine the amount of supply air.
*A. 93,229.17 lb/hr B. 83,229.17 lb/hr C. 73,229.17 D. 63,229.17 lb/hr
SOLUTION:
Sensible heat per person = 225 Btu/hr
Qa = 225(1500) + 110,000 = 447,500 Btu/hr
Qa = m cp(t1 t2)
447,500 = ma(0.24)(80 60)
ma = 93,229.17 lb/hr

In a Brayton cycle that operates between temperature limits of 300K and 1773K wit k =
1.4, determine the temperature at the end of the compression (isentropic) for maximum
work of the cycle.
700K B. 590.5K *C. 730K D. 350K
SOLUTION:
For maximum work: T2 = (T1T3)1/2
T2 = (300 x 1773)1/2 = 730K

At 35% solution leaves the absorber and 30% solution enters the absorber. The heat
removed from the absorber by cooling water is 547.6 Btu and ammonia is superheated
by 10. Find the pound per pound of ammonia gas from the evaporating coils.
11 B. 12 *C. 13 D. 14
SOLUTION:
n = lb/lb of ammonia gas from the coils
n = (1 - x2) / (x1 - x2) 1
n = (1- 0.3) / (0.35 0.3) 1 = 13
A Carnot refrigeration system operates at Tmax / Tmin = 1.5. Find the KW per tons of
refrigeration.
1.91 B. 2.15 *C. 1.76 D. 1.55
SOLUTION:
Wo / TR = 3.516 / COP = 3.516 / (Tmin / (Tmax Tmin)
Wo / TR = [3.516(Tmax Tmin)] / Tmin
Wo / TR = 3.516[(Tmax / Tmin) 1] = 3.516(1.5-1) = 1.758 KW/TR
Assume 8 ft3 of air at 100 psi, 100 are compressed isothermally to a volume of 2 ft 3. For
each of end states of the process, find the bulk modulus.
*A. 400 and 100 psi B. 400 and 110 psi C. 400 and 120 psi D. 400 and 130

An empty open can is 30 cm high with a 15 cm diameter. The can, with the open end
and down, is pushed under water with a density of 1000kg/m 3. Find the water level in
the can when the top of the can is 50 cm below the surface.
17.20 cm *B. 2.12 cm C. 4.20 cm D. 5.87cm
SOLUTION:
Consider the water pressure
Pw = w h + 1010.325 + (0.8-x)(9.81) + 101.325 = 109.173 9.81x
Consider the air pressure
P1V1 = P2V2
101.325(Ax0.3) = P2[A(0.3-x)]
P2 =
Pw = P 2

109.173 9.81x =
9.81x2 112.116x + 2.3705 = 0
By quadratic formula:
X = 0.02118 m = 2.12 cm

A cylindrical pipe with water flowing downward at 0.03 m3/s having top diameter of 0.08,
bottom diameter of 0.04 m and a height of 1.5m. Find the pressure between the pipe
154.63 Kpa B. 197.93 Kpa *C. 252.44 Kpa D. 243.92 Kpa
SOLUTION:
+ Z1 = + Z 2

Z1 Z2 = 1.5 m
Z2 Z1 = -1.5 m

V1 = = 5.968 m/s

V2 = = 23.87 m/s

P1 P2 = 252.44 Kpa

Determine the size of pipe which will deliver 8 liters of medium oil (v= 6.10 x 10-6 m2/s)
assuming laminar flow conditions:
622 mm B. 754 mm C. 950 mm *D. 835 mm
SOLUTION:

V=

Re =
For laminar flow, Re = 2000

2000 =
d = 0.835 m = 835 mm

The type of flow occupying in a 1 cm diameter pipe which water flows at a velocity of
2.50 m/s. Use v = 1.13 x 10-6 m2/s for water.
*A. turbulent B. constant C. laminar D. none of the above

SOLUTION:

Re =
Re =

Re = 22,124
Since it is greater than 2000 then it is turbulent flow

What is the force is exerted by water jet 60 mm diameter if it strikes a wall at the rate of
15 m/s?
*A. 636.17 N B.442.62 N C. 764.23 N D. 563.34 N
SOLUTION:
F=wQv
Q = A v = = 0.0424 m3/s
F = (1000)(0.0424)(15) = 636.17 N

A 300 mm diameter pipe discharges water at the rate of 200 li/s. Point 1 on the pipe has
a pressure of 260 kpa and 3.4 m below point 1 is point 2 with a pressure of 300 kpa.
Compute the head loss between points 1 and 2.
4.29 m B. 2.59 m C. 6.32 m *D. 1.87 m
SOLUTION:
hL
hL =

Water flowing at the rate of 10 m/s from an orifice at the bottom of a reservoir. Find the
pressure at the bottom of the reservoir.
30 kpag B. 40 kpag *C. 50 kpag D. 60 kpag
SOLUTION:
h = V2/ 2g = 102/ 2(9.81) = 5.0968 m
P = w h = 9.81(5.0968) = 50 kpag

Steam flows through a nozzle at 400oC and 1 Mpa (h = 3263.9 KJ/kg) with velocity of
300 m/s. Find the stagnation enthalpy.
3300 KJ/kg B. 3290 KJ/kg *C. 3320 KJ/kg *D. 3309 KJ/kg
SOLUTION:
ho = h + v2/2000 = 3263.9 + 3002/2000 = 3309 KJ/kg

Air flows through a nozzle at a speed of 350 m/s. Find the stagnation temperature if
entrance temperature is 200oC.
241.25oC B. 251.25oC *C. 261.25oC D. 271.25oC
SOLUTION:
To = T1 + v2/2000Cp = (2000 + 273) + 3502/2000(1)
To = 534.25oK = 261.25
Carbon dioxide flows through a nozzle with a speed of 400 m/s. Compute the dynamic
temperature.
92.56oK *B. 94.56oK C. 96.56oK D. 98.56oK
SOLUTION:
For CO2: Cp = 0.846 KJ/kg-K
Dynamic temperature = v2/2000Cp = 4002/2000(0.846) = 94.56oK

Carbon dioxide flows through a nozzle with a speed of 380 m/s. The entrance condition
of nozzle is 250oC and 1200 kpa. Find the stagnation pressure.
*A. 2,136.34 kpa B. 2,146.34 kpa C. 2,156.34 kpa D. 2,166.34 kpa
SOLUTION:
T1 = 250 + 273 = 523oK
To = T1 + v2/2000 = 523 = 3802/2000 = 595.2oK
P1 = 1200 kpa
T1/To = (P1/Po)k-1/k
For CO2: k = 1.289
523/595.2 = (1200/Po)1.289-1/1.289
P0 = 2,136.34 kpa

Air enters a diffuser with a velocity of 200 m/s. Determine the velocity of sound if air
temperature is 30oC.
*A. 349 m/s B. 359 m/s C. 369 m/s D. 379 m/s
SOLUTION:
C=

Air flows through a nozzle with temperature of entrance of 420 oK stagnation


temperature of 468oK. Find the mach number.
0.744 *B. 0.754 C. 0.764 D. 0.774
SOLUTION:
To = T1 + v2/2000Cp
468 = 420 + v2/2000
v = 309.838 m/s
C=
M = v/C = 309.838/410.8 = 0.754

Air at 300oK and 200 kpa is heated at constant pressure to 600oK. Determine the
change of internal energy.
245.58 KJ/kg B. 235.58 KJ/kg C. 225.58 KJ/kg *D. 215.58 KJ/kg
SOLUTION:
U = mCv (T2 T1) = 1(0.7186)(600 -300) = 215.58 KJ/kg

An insulated rigid tank initially contains 1.5 lb of helium at 80 oF and 50 psia. A paddle
wheel with power rating of 0.02 hp is operated within the tank for 30 min. Determine the
final temperature.
159.22oF B. 169.22oF *C. 179.22oF D. 189.22 oF
SOLUTION:
W = U = m Cv (T2 T1)
0.02 hp (0.50hr)(2545Btu/hr/hp) = 1.5(0.171)(t2 80)
t2 = 179.22oF

A 4m2 asphalt pavement with emissivity of 0.85 has a surface temperature of 50 oC. Find
the maximum rate of radiation that can be emitted from the surface.
A. 2,068.32 watts B. 2,078.32 watts C. 2,088.32 watts *D. 2.098.32 watts
SOLUTION:
Qr = e kev A Ts4
Kev = 5.67 x 10-8 ( Stefan Boltzman constant)
Qr = 0.85(5.67 z 10-8)(4)*50 +273)4 = 2,098.32 watts

Air at 10oC and 90 kpa enters a diffuser of a jet engine steadily with a velocity of 200
m/s. The inlet area diffuser is 0.40 m2. Determine the mass flow rate of air.
A. 72.79 kg/s B. 74.79 kg/s C. 76.79 kg/s *D. 78.79 kg/s
SOLUTION:
W = P/RT = 80/0.287(10 + 273) = 0.985 kg/m3
m = w v A = 0.985(200)(0.40) = 78.79 kg/s

Consider a refrigeration whose 40 watts light bulb remains on continuously as a result of


a malfunction of the switch. If the refrigerator has a COP of 1.3 and the cost of electricity
is 8 cents per kw-hr., determine the increase in the energy consumption of the
refrigerator and its cost per year if the switch is not fixed.
*A. P49.59 B. P47.59 C. P45.59 D. P43.59
SOLUTION:
COP = RE/W ref
1.3 = 40/W ref
Wref = 30.769 watts
W = W b + W ref = 40 + 30.769 = 70.77 watts
W = 0.07077 Kw
Cost = 0.07077(8760)(P0.08) = P49.59

A 75 hp motor that has an efficiency of 91% is worn out and is replaced by a high-
efficiency motor that has an efficiency of 95.4%. Determine the reduction in heat gain of
the room due to higher efficiency under full-load conditions.
A. 2.24 KW *B. 2.44 KW C. 2.64 KW D. 2.84 KW
SOLUTION:
P01 = (75 x 0.746)(0.91) = 50.91 KW
P02 = (75 x 0.746)(0.954) = 53.376 KW
Qreduced = 53.376 50.91 = 2.44 KW

A household refrigerator that has a power input of 450 watts and a COP of 2.5 is to cool
five large watermelons, 10 kg each, to 8oC. If the watermelons are initially at 20oC,
determine how long will take for the refrigerator cool them. The watermelons can be
treated as a water whose specific heat is 4.2 KJ/kg-oK.
A. 2220 seconds B. 2230 seconds *C.2240 seconds D. 2250 seconds
SOLUTION:
COP = RE/Wc
2.5 = RE/450
RE = 1,125 watts
RE = m cp (t2 t1)
1125 t = (10 x 5)(4.2)(20-8)
t = 2240 seconds

When a man returns to his wall-sealed house on a summer day, he finds that the house
is at 32oC. He returns on the air conditioner which cools the entire house to 20 oC in 15
minutes, if COP is 2.5, determine the power drawn by the airconditioner. Assume the
entire mass within the house is 800 kg of air for which cv = 0.72 KJ/kg-K, cp = 1.0KJ/kg-
K.
1.072 KW B. 2.072 KW *C. 3.072 KW D. 4.072 KW
SOLUTION:
RE = m cv (T2 T1) = (800/15x60)(0.72)(32-20)
RE = 7.66 KW
Wc = 7.68/2.5 = 3.072 KW

A heat source at 8000K losses 2000 KJ of heat to a sink at 500oK. Determine the
entropy generated during this process.
*A. 1.5 KJ/K B. 2.5 KJ/K C. -2.5 KJ/K D. 4 KJ/K
SOLUTION:
Ssource = -2000/800 = -2.5
Ssink = 2000/500 = 4
Sgen. = -2.5 + 4 = 1.5 KJ/K

Helium gas is compressed in an adiabatic compressor from an initial state of 14 psia


and 50oF to a final temperature of 320oF in a reversible manner. Determine the exit
pressure of Helium.
38.5 psia *B. 40.5 psia C. 42.5 psia D. 44.5 psia
SOLUTION:
T2/T1 = (P2/P1)n-1/n
(320 + 460)/(50 +460) = (P2/14)1.587-1/1.587
P2 = 40.5 psia

Air pass thru a nozzle with efficiency of 90%. The velocity of air at the exit is 600 m/s.
Find the actual velocity at the exit.
A. 382 m/s B. 540 m/s C. 458 m/s *D. 568 m/s
SOLUTION:
e = (v2/v3)2
0.9 = (v2/600)2
v2 = 568.21 m/s
A 50 kg block of iron casting at 500K is thrown into a large lake that is at a temperature
of 285oK. The iron block eventually reaches thermal equilibrium with the lake water.
Assuming average specific hear of 0.45 KJ/kg-K for the iron, determine the entropy
generated during this process.
*A. -12.65 KJ/k B. 16.97KJ/K C. 4.32 KJ/K D. 6.32 KJ/K
SOLUTION:
Siron = m c ln (T2/T1) = 50(0.45)ln(285/500) = -12.65 KJ/K
Slake = Q/T = [50(0.45)(500-285)]/285 = 16.97 KJ/K
Sgen. = -12.65 + 16.97 = 4.32 KJ/K

A windmill with a 12 m diameter rotor is to be installed at a location where the wind is


blowing at an average velocity of 10 /s. Using standard conditions of air (1 atm, 25oC),
determine the maximum that can be generated by the windmill.
A. 68 KW *B. 70 KW C. 72 KW D. 74 KW
SOLUTION:
w = P/RT = 101.325/(0.28)(25+ 273) = 1.1847 kg/m3
m = w A v = 1.1847(/4 x 122)(10) = 1,1339.895 kg/s
KE = v2/2000 = 102/2000 = 0.05 KJ/kg
Power = mKE = 1,1339.895(0.05) = 70 KW

Consider a large furnace that can supply heat at a temperature of 2000 oR at a steady
rate of 3000Btu/s. Determine the energy of this energy. Assume an environment
temperature of 77oF.
A. 2305.19 KW *B. 2315.19 KW C. 2325.19 KW D. 2335.19 KW
SOLUTION:
e = = 0.7315
W = e Q = 0.7315(3000) = 2194.5 Btu/s = 2315.19 KW

A heat engine receives hat from a source at 1200oK at a rate of 500KJ/s and rejects the
waste heat to a medium at 300oK. The power output of the heat engine is 180 KW.
Determine the irreversible rate for this process.
A. 190 KW *B. 195 KW C. 200 KW D. 205 KW
SOLUTION:
e = (1200 300) / 1200 = 0.75
W = 0.75(500) = 375 KW
Irreversibilities = 375 180 = 195 KW

A dealer advertises that he has just received a shipment of electric resistance heaters
for residential buildings that have an efficiency of 100 percent. Assuming an indoor
temperature of 21oC and outdoor temperature of 10oC, determine the second law
efficiency of these heaters.
A. 8.74% B. 6.74% *C. 3.74% D. 4.74%
SOLUTION:
COP1 = 100% efficient = 1
COP2 = (21 + 273) / (21 10) = 26.72
e = COP1 /COP2 = 1/ 26.72 = 3.74 %

A thermal power plant has a heat rate of 11,363 Btu/KW-hr. Find the thermal efficiency
of the plant.
34% B. 24% C. 26% *D. 30%
SOLUTION:
e = 3412 / Heat rate = 3412 / 11363 = 30 %

A rigid tank contains 2 kmol of N2 and 6 kmol of CO2 gasses at 300oK and 15 Mpa. Find
the tank volume us ideal gas equation.
7.33 m3 B. 5.33 m3 C. 3.33 m3 *D. 1.33 m3
SOLUTION:
PmVm = Nm R Tm
15,000 Vm = (6 + 2)(8.314)(300)
Vm = 1.33 m3

A spherical balloon with a diameter of 6 m is filled with helium at 20 oC and 200 kpa.
Determine the mole number.
*A. 9.28 Kmol B. 10.28 Kmol C. 11.28 Kmol D. 13.28
Kmol
SOLUTION:
PV=NRT
(200)[(4/3)()(6/2)3] = N (8.314)(20 + 273)
N = 9.28 Kmol

The air in an automobile tire with a volume of 0.53 ft3 is at 90oF and 20 psig. Determine
the amount of air that must be added to raise the pressure to the recommended value of
30 psig. Assume the atmospheric to be 14.7 psia and the temperature and the volume
to remain constant.
*A. 0.026 lb B. 0.046 lb C. 0.066 lb D. 0.086 lb
SOLUTION:
PV=mRT
(20 + 14.7)(144)(0.53) = m1 (53.3)(90 + 460)
m1 = 0.09034 lb
(30 + 14.7)(144)(0.53) = m2(53.3)(90 + 460)
m2 = 0.11634 lb
madded = m2 m1 = 0.11634 0.09034 = 0.026 lb

A rigid tank contains 20 lbm of air at 20 psia and 70oF. More air is added to the tank
until the pressure and temperature rise to 35 psia and 90 oF, respectively. Determine
the amount of air added to the tank.
11.73 lb *B. 13.73 lb C. 15.73 lb D. 17.73 lb
SOLUTION:
P1V1 = m1 R1T1
(20 x 144)(V1) = 20 (53.3)(70 + 460)
V = 196.17 ft3
P2V2 = m2R2T2
(35 x 144)(196.17) = m2 (53.3)(90 + 460)
m2 = 33.73 lbs
madded = m2 m1 = 33.73 20 = 13.73 lb

A rigid tank contains 5 kg of an ideal gas at 4 atm and 40oC. Now a valve is opened,
and half of mass of the gas is allowed to escape. If the final pressure in the tank is 1,5
atm, the final temperature in the tank is:
*A. -38oC B. -30oC C. 40oC D. 53oC
SOLUTION:
PV = m R T
(4 x 9.81)(V) = 5(0.287)(40 + 273)
V = 11.446 m3
PV = mRT
(1.5 x 9.81)(11.446) = (5/2)(0.287)(T)
T = 234.74oK = -38.26oC

The pressure of an automobile tire is measured to be 200 kpa(gage) before the trip and
220 kpa(gage) after the trip at a location where the atmospheric pressure is 90 kpa. If
the temperature of the air in the tire before the trip is 25oC, the air temperature after the
trip is:
*A. 45.6oC B. 54.6oC C. 27.5oC D. 26.7oC
SOLUTION:
T 2 / T 1 = P 2 / P1
T2 / (25+ 273) = (220 +90) / (200 + 90)
T2 = 318.55 K
t2 = 45.55oC

Water is boiling at 1 atm pressure in a stainless steel pan on an electric range. It is


observed that 2 kg of liquid water evaporates in 30 mins. The rate of heat transfer to the
water is:
2.97 KW B. 0.47 KW *C. 2.51 KW D. 3.12 KW
SOLUTION:
Q = mL = (2257) = 2.51 KW

Consider a person standing in a breezy room at 20oC. Determine the total rate of heat
transfer from this person if the exposed surface area and the average outer surface
temperature of the person are 1.6 m2 and 29oC, respectively, and the convection heat
transfer coefficient is 6 W/K-m2 with emissivity factor of 0.95.
86.40 watts B. 61.70 watts C. 198.1 watts *D. 168.1 watts
SOLUTION:
Qc = h A (t2 t1) = (6)(1.6)(29.20) = 86.40 watts
Qr = (0.95)(5.67 x 10-6)[(1.6)(29 + 273)4 (20 + 273)4] = 81.7 watts
Q = Qc + Qr = 86.40 + 81.7 = 168.1 watts
Water is boiler in a pan on a stove at sea level. During 10 minutes of boiling, it is
observed that 200 grams of water has evaporated. Then the rate of heat transfer to the
water is:
0.84 KJ/min *B. 45.1 KJ/min C. 41.8 KJ/min D. 53.5 KJ/min
SOLUTION:
Q = m L = (0.2 / 10) (2257) = 45.1 KJ/min

An aluminum pan whose thermal conductivity is 237 W/m-C has a flat bottom whose
diameter is 20 cm and thickness 0.4 cm. Heat is transferred steadily to boiling water in
the pan through its bottom at a rate of 500 watts. If the inner surface of the bottom of the
pan is 105oC, determine the temperature of the surface of the bottom of the pan.
95.27 oC *B. 105.27oC C. 115.27oC D. 125.27oC
SOLUTION:
A = / 4 ( 0.20)2 = 0.0314 m2
Q=

500 =

T2 = 105.27oC

For heat transfer purposes, a standing man can be modeled as a 30 cm diameter, 170
cm long vertical cylinder with both the top and bottom surfaces insulated and with the
side surface at an average temperature of 34oC. For a convection heat transfer
coefficient of 15 W/m2- oC, determine the rate of heat loss from this man by convection
in an environment at 20oC.
316.46 watts B. 326.46 watts *C. 336.46 watts D. 346.46 watts
SOLUTION:
Qc = k A (t2 t1) = 15 ( x 0.30 x 1.7) (34 20) = 336.46 watts

A 5cm diameter spherical ball whose surface is maintained at a temperature of 70 oC is


suspended in the middle of a room at 20oC. If the convection heat transfer coefficient is
15 W/m2 C and the emissivity of the surface is 0.8, determine the total heat transfer
from the ball.
23.56 watts *B. 32.77 watts C. 9.22 watts D. 43.45 watts
SOLUTION:
A = 4 r2 = 4 (0.05)2 = 0.0314 m2
Qc = h A (t2 t1) = 15 (0.0314)(70 20) = 23.56 watts
Qr = (0.80)(5.67 x 10-6)(0.0314)[(70 + 273)4 (50 + 273)4] = 9.22 watts
Q = Qr + Qc = 23.56 + 9.22 = 32.77 watts

A frictionless piston-cylinder device and rigid tank contain 1.2 kmol of ideal gas at the
same temperature, pressure, and volume. Now heat is transferred, and the temperature
of both system is raised by 15oC. The amount of extra heat that must be supplied to the
gas in the cylinder that is maintained at constant pressure.

SOLUTION:
0 B. 50 KJ C. 100 KJ *D. 150 KJ
Q = m cp (t2 t1) = (1.2 x 8.314)(1)(15) = 150 KJ

A supply of 50 kg of chicken needs at 6oC contained in a box is to be frozen to -18oC in


a freezer. Determine the amount of heat that needs to be removed. The latent heat of
chicken is 247 KJ/kg, and its specific heat is 3.32 KJ/kg-oC above freezing and 1.77
KJ/kg-oC below freezing. The container box is 1.5 kg, and the specific heat of the box
material is 1.4 Kj/kg-oC. Also the freezing temperature of chicken is -2.8oC.
*A. 15,206.4 KJ B. 50.4 KJ C. 15,156 KJ D. 1,863 KJ
SOLUTION:
Qchicken = 50 [3.32(6 + 2.8) = 247 1.77(-2.8 + 18)] = 15,156 KJ
Qbox = 1.5(1.4)(6 + 8) = 50.4 KJ
Q = 15,156 + 50.4 = 15, 206.4 KJ

Water is being heated in a closed pan on top of a range while being stirred by a paddle
wheel. During the process, 30 KJ of heat is transferred to the water, and 5 KJ of heat is
lost to the surrounding air. The paddle wheel work amounts to 500 N-m. Determine the
final energy of the system if its initial energy is 10 KJ.
*A. 35.5 KJ B. 45.5 KJ C. 25.5 KJ D. 14.5 KJ
SOLUTION:
Final energy = Qa + U Qloss + W = 30 + 10 5 + 0.50 = 35.5 KJ

A classroom that normally contains 40 people is to be air- conditioned with window air-
conditioning units of 5 KW cooling capacity. A person at rest may be assumed to
dissipate heat at a rate of about 360 KJ/hr. There are 10 light bulbs in the room, each
with a rating of 100 watts. The rate of heat transfer to the classroom through the walls
and the windows is estimated to be 15,00 KJ/hr. If the room air is to be maintained at a
constant temperature of 21oC, determine the number of window air- conditioning units
required.
1 unit *B. 2 units C. 3 units D. 4 units
SOLUTION:
Q = total head load = 40(360/3600) + 10(0.100) + 15,000/3600 = 9.167 KW
No. of air-conditioning = 9.167/5 = 1.833 = 2 units

A 4m x 5m x 6m room is to be heated by a baseboard resistance heater. It is desired


that the resistance heater be able to raise the air temperature in the room from 7 to
23oC within 15 minutes. Assuming no heat losses from the room and an atmospheric
pressure of 100 kpa, determine the required power of the resistance heater. Assume
constant specific heats at room temperature.
2.34 KW *B. 1.91 KW C. 4.56 KW D. 6.34 KW
SOLUTION:
w = P / R T = 100 / (0.287)(7 +273) = 1.244 kg / m3
m = 1.244 (4 x 5 x 6) = 149.28 kg
Q = m cv (t2 t1 ) = 149.28 (0.7186)(23 7) = 1,716.36 KJ
Power = 1,716.36 / (15 x 60) = 1.91 KW
A student in a 4m x 6m x 6m dormitory room turns on her 150 watts fan before she
leaves the room on a summer day, hoping that the room will be cooler when she comes
back in the evening. Assuming all the doors and windows are tightly closed and
disregarding any heat transfer through the walls and the windows, determine the
temperature in the room when she comes back 10 hours later. Use specific heat values
at room temperature, and assume the room to be at 100 kpa and 15 oC in the morning
when she leaves
28.13oC B. 38.13oC C. 48.13oC *D. 58.13oC
SOLUTION:
w = P / R T = 100 / (0.287)(15 + 273) = 1.2098 kg / m 3
m = 1.2098(4 x 6 x 6) = 174.216 kg
Q = m cv (t2 t1)
0.15(10 x 3600) = 174.216 (0.7186)(t2 15)
t2 = 58.13oC

A piston cylinder device whose piston is resting on top of a set stops initially contains
0.50 kg of helium gas at 100 kpa and 25oC. The mass of the piston is such that 500 kpa
of pressure is required to raise it. How much heat must be transferred to the helium
before the piston starts rising?
1557.13 KJ B. 1657.13 KJ C. 1757.13 KJ *D. 1857.13 KJ
SOLUTION:
For helium: cv = R / (k-1) = (8.314 / 4) (1.667 1) = 3,116 KJ/ kg-K
T2 = (25 + 273)(500 / 100) = 1,490oK
T1 = 25 + 273 = 298o
Q = m cv (T2 T1) = 0.50(3.116)(1490 298) = 1857.13 KJ

In order to cool 1 ton (100kg) of water at 20oC in an insulated tank, a person pours 80
kg of ice at -5oC into the water. Determine the final equilibrium temperature in the tank.
The melting temperature and the hat of fusion of ice at atmospheric pressure are 0 oC
and 333.7 kJ/kg, respectively.
*A. 12.43oC B. 14.43oC C. 16.43oC D. 18.43oC
SOLUTION:
Qwater =Qice
1000(4.187)(20 te) = 80(2.09)(0 + 5) + 80(333.7) + 80(4.187)(te 0)
te = 14.43oC

A fan is powered by a 0.5 hp motor and delivers air at a rate of 85 m 3/min. Determine
the highest value for the average velocity of air mobilized by the fan. The density of air
to 1.18 kg/m3.

18.23 m/s *B. 21.12 m/s C. 25.34 m/s D. 32.23 m/s


SOLUTION:
P=wQh
0.50(0.746) = ( 1.18 x 0.00981)( 85 / 60) (h)
h = 22.74 m
v = = 21.12 m/s

An Ocean Thermal Energy Conversion power plant generates 10,000 KW using a


warm surface water inlet temperature of 26oC and a cold deep- water temperature of
15oC. ON the basis of a 3oC drop in the temperature of the warm water and a 3oC rise in
the temperature of the cold water due to removal and addition of heat, calculate the
power required in KW to pump the cold- deep water to the surface and through the
system heat.Assume a Carnot cycle efficiency and density of cold water to be 1000
kg/m3.
108 *B. 250 C. 146 D. 160
SOLUTION:
e = (Th TL)/ TH = [(26 + 273) (15 + 273)] / (26 + 273) = 0.0679
e =W / Qa
0.03676 = 10, 000 / QA
QA = 271, 612. 99 KW
QR = Qa W = 271,812.99 10, 000 = 261,813 KW
QR = m cp (t)
261, 813 = m (4.187)(3)
M = 20, 643.32 kg/s
Q = 20,843.32 kg/s or 20,843.32 li/s = 20.843 m3/s
H = P / w = 12 / 9.81 = 1.223 m
Wp = w Q h = 9.81(20.843)(1.223) = 250.12 KW

A plate type solar energy collector with an absorbing surface covered by a glass plate
is to receive an incident radiation of 800 W/m2. The glass plate has a reflective of 0.12
and a transmissivity of 0.85. The absorbing surface has an absorptivity of 0.90. The
area of the collector is 5m2. How much solar energy in watts is absorbing by the
collector?
2500 B. 2880 C. 3510 *D. 3060
SOLUTION:
Q = heat absorbed from sun
Q = (800 W/m2)(5 m2)(0.85)(0.9) = 3,060 watts

A tank contains liquid nitrogen at -190oC is suspended in a vacuum shell by three


stainless steel rods 0.80 cm in diameter and 3 meters long with a thermal conductivity of
16.3 W/m2-C. If the ambient outside the vacuum shell is 15oC, calculate the magnitude
of the conductive heat flow in watts along the support rods.
*A. 0.168 B. 0.0587 C. 0.182 D. 0.176
SOLUTION:
Q = h A (t) = 16.3 (/4 x 0.0082)(15 (-190)) = 0.168 watts
An elastic sphere containing gas at 120 kpa has a diameter of 1.0 m. Heating the
sphere causes it to expand to a diameter of 1.3 m. During the process the pressure is
proportional to the sphere diameter. Calculate the work done by the gas in KJ.
41.8 B. 50.6 *C. 87.5 D. 35.4
SOLUTION:
PD
P=kD
120 = k(1)
K = 120
P = 120 D
V = 4/3 (D/d)2 = 4/24 D3
dV = (12/24) D2 dD
W = D3dD
W = 87.47 KJ

An ideal gas with a molecular weight of 7.1 kg/kg mol is compressed from 600 kpa and
280oK to a final specific volume of 0.5 m3/kg. During the process the pressure varies
according to p = 620 + 150v + 95v2 where p is in kpa and v in m3/kg. Calculate the work
of compression in KJ/kg?
32.8 B. 28.7 C. 35.6 *D. 33.3
SOLUTION:
V1 = R T / P = (8.314 / 7.1)(280) / (600) = 0.546 m3/kg
W=

One cubic meter container contains a mixture of gases composed of 0.02 kg-mol of
oxygen and 0.04 kg-mol of helium at a pressure of 220 kpa. What is the temperature of
this ideal gas mixture in degrees Kelvin?
*A. 441 B. 350 C. 400 D. 450
SOLUTION:
V = V1 + V2
VT =
1=

T = 441 oK

Methyl alcohol (CH3OH) is burned with 25% excess air. How much unburned oxygen in
kg-mol-oxygen / kg-mol fuel will there be in the products if the combustion is complete?
0.35 B. 0.45 *C. 0.37 D. 0.65
SOLUTION:
CH3OH + O2 + (3.76)N2 = CO2 + H2O + (3.76)N2
CH3OH + 1.5O2 + 1.5(3.76)N2 = 1 CO2 + 2H2O + 1.5(3.76)N2
Consider 25% excess air:
CH3OH + 1.25(1.5)O2 + 1.25(1.5)(3.76)N2 = 1CO2 + 2H2O + 1.25(1.5)(3.76)N2 +
0.25(1.5)O2
Unburned O2 = 0.25(1.5) = 0.375

A 12 DC electrical motor draws a current of 15 amps. How much work in KJ does this
motor produce over a 10 minute period of operation?
*A. 108.0 B. 129.6 C. 216.0 D. 318.2
SOLUTION:
W=E=QV
W = (15 x 10 x 60)(12) = 108,000 J = 108 KJ

A 4 liter (2-liter per revolution at standard pressure and temperature) spark ignition
engine has a compression ratio of 8 and 2200 KJ/kg heat addition by the fluid
combustion. Considering a cold air-standard Otto cycle model, how much power will the
engine produce when operating at 2500 rpm?
*A. 166.53 hp B. 73.12 hp C. 97.4 hp D. 148 hp
SOLUTION:
W = 1.2 kg/m3 (standard density of air)
M = 2 li/rev x 2500 rev/min x 1.2 kg/m3 x 1 m3/ 1000 li x 1 min/ 60 sec = 0.10 kg/s
e = W / QA
e = 1 -1 / 8 1.4 -1 = 0.5647
0.5647 = W / 2200
W = 1,242.34 KJ/kg (0.10 kg/s) = 124.23 KW = 166. 53 hp

A simple Rankine cycle produces 40 MW of power, 50 MW of process heated and


rejects 50 MW of heat to the surroundings. What is the utilization factor of this
cogeneration cycle neglecting the pump work?
50% B. 60% *C. 64% D. 80%
SOLUTION:
QA = WT + W processW p = 40 + 50 + 50 = 140 KW
UF = = 64 %

The rate of heat transfer to the surroundings from a person at a rest is about 400 KJ/hr.
Suppose that the ventilation system fails in an automobile in an auditorium containing
120 people and assuming that the energy goes into the air of volume 1500 m 3 initially at
300oK and 101 kpa, calculate the rate in oC/min of air temperature change.
0.81 B. 0.53 *C. 0.63 D. 1.0
SOLUTION:
Q = m cv t
PV = m RT
101(1500) = m (0.287)(300)
m = 1,759.58 kg
Q = m cv t
120(400 / 60) = 1759.58 (0.7186)(t)
t = 0.633 oC / min

An insulated box containing helium gas falls from a balloon 4.5 km above the earths
surface. Calculate the temperature rise in oC of the helium when box hits the ground.
15.2 B. 12.6 C. 25.3 *D. 14.1
SOLUTION:
Cv of helium = 3118.9 j/kg-C
m g h = m cv t
m (9.81)(4500) = m (3118.9) t
t = 14.15 oC

Consider two Carnot heat engines operating in series. The first engine receives heat
from the reservoir at 2500oK and rejects the waste heat to another reservoir at
temperature T. The second engine receives heat by the first one, convert some of it to
work, and rejects the rest to a reservoir at 300oK. If thermal efficiencies of both engines
are the same, determine the temperature T.
*A. 849oK B. 578 oK C. 763 oK D. 976 oK
SOLUTION:
et = e 2

T = 849oK

An ideal gas mixture consists of 2 kmol of N2 and 6 kmol of CO2. The mass fraction of
CO2 is:
0.175 B. 0.250 *C. 0.825 D. 0.750
SOLUTION:

6(44)
Mass fraction of CO2 = = 0.825
6(44) + 2(28)
An ideal gas mixture consists of 2 kmol of N2 and 6 kmol of CO2. The apparent gas
constant of mixture is:
*A. 0.208 B. 0.231 C. 0.531 D. 0.825
Solution:
M= (2/8)(28) + (6/8)(44) = 40
R= 8.314/M = 8.314/40 = 0.208 KJ/kg-K
A Carnot cycle operates between the temperature limits of 300OK and 1500OK, and
produces 600 KW of net power.
The rate of entropy changes of the working fluid during the heat addition process is:
A. 0 B. 0.4KW/K *C. 0.5KW/K D.2.0KW/K
Solution:
W= ( s) (TH TL)
600 = ( s) (1500 300)
( s)= 0.50 KW/K
Air in an ideal Diesel cycle is compressed from 3L to 0.15L and then it expands during
the constant pressure heat addition process to 0.3L. Under cold air standard conditions,
the thermal efficiency of this cycle is:
rk = 3/0.15= 20
rc= 0.3/0.15= 2

1 rck- 1 1 21.2 - 1
e=1- =1- = 0.6467
rkk-1 k(rc 1) 201.4-1 1.4 (2-1)
=64.67%
Helium gas is an ideal Otto cycle is compressed from 20oC and 2L to 0.25L and its
temperature increases by an additional 800oC during the addition process. The
temperature of helium before the expansion process is:
*A. 1700oC B. 1440oC C. 1240oC D.880oC
Solution:
rk =2/0 25 = 8
T2 = (20+273) (8)1.667-1 =1,172K
T3 = T2 + 800 = 1172 + 800 = 1972oK
t3 = 1699oC = 1700oK
In an ideal Brayton cycle has a net work output of 150KJ/kg and backwork ratio of 0.4.
If both the turbine and the compressor had an isentropic efficiency of 80%, the net work
output of the cycle would be.
A. 50KJ/kg *B. 75KJ/kg C. 98KJ/kg D.120KJ/kg
Solution:
Backwork ratio = W O/WT
0.40 = W O/W T
WO = 0.40 W T
Wnet = W T - W O
150 = W T 0.4 W T
WT =250 KJ/kg
WT =250(0.8) = 200KJ/kg
WP = 0.40(200) =100KJ/kg
WP = 100/0.80 =125 KJ/kg
Wnet= WT W C= 200 125 = 75 KJ/kg
Air entered a turbojet engine at 200 m/s at a rate of 20 kg/s, and exists at 800 m/s
relative to the aircraft. the thrust developed by the engine is:
A. 6KN *B. 12KN C.16KN D. 20KN
Solution:
Thrust developed = m(v2 v1) = 20(800 200) = 12,000N = 12KN
A thermal power has a net power 10MW. The backwork ratio of the plant is 0.005.
Determine the compressor wor.
A. 50.15KW B. 50.35KW *C.50.25KW D. 50.45KW
Solution:
Wnet= WT + WP
BW= WP / W T
0.005 =WP / W T
WP= 0.005WT
Wnet= WT - WP
10,000 = W T 0.005WT
WT= 10,050.25 KW
WC= 0.005(10,050.25) = 50.25KW
A heat engine receives heat from a source at 1200oK at a rate of 500KJ/s and rejects
the waste heat to a sink at 300oK. If the power output of the engine is 200KW, the
second law efficiency of the heat engine is:
A.35% B.40% *C.53% D.75%
Solution:
ea= 200/500 = 0.40
et= (TH TL)/TH = (1200 300)/1200 = 0.75
es= 0.40/0.75= 53.33%
A water reservoir contains 100,000 kg of water at an average elevation of 60 m. The
maximum amount of electric power that can be generated from this water is:
A.8KWh *B.16KWh C.1630KWh D.58, 800KWh
Solution:
P= m h = (100,000 x 0.00981)(60)= 58,860 KJ
P= 58,860 KJ x KWh/3600 KJ = 16.35KWh
A house is maintained at 22oC in winter by electric resistance heaters. If the outdoor
temperature is 5oC, the second law of efficiency of the resistance heaters is:
A.0% *B.5.8% C.34% D.77%
Solution:
ea= 100% of resistance heaters
et= (22 5)/(22 + 273) = 5.8%
es= 5.8/100 = 5.8%
A thermoelectric refrigerator that resembles a small ice chest is powered by a car
battery, and has a COP of 0.10. If the refrigerator cools at 0.350L canned drink from
20OC to 4OC in 30 min. determine the average electric power consumed by the
thermoelectric refrigerator.
*A.130 watts B.110 watts C.120 watts D.140 watts
Solution:
(1 x 0.35)
Q= m cp(t2 t1) = (4.187)(20 4) = 13 watts
30 x 60
COP= RE/Wc
0.10= 13/0.10= 130 watts
A Carnot refrigerator operates in a room in which the temperature is 25OC and
consumes 2 kW of power when operating. If the food compartment of the refrigerator is
to be maintained at 3OC, determine the rate of heat removal from the food compartment.
*A.1504.8 kJ/min B.12.86 kJ/min C.1625 kJ/min
D.9.57 kJ/min
Solution:
COP= TL /TH TL = (3 + 273)/ (25 + 273) (3+273) =12.54
QL= COP x W = 12.54 x 2(60) = 1504.8 kJ/min
A household refrigerator with EER 8.0 removes heat from the refrigerated space at a
rate of 90 kJ/min. Determine the rate of heat transfer to the kitchen air.
A.101.25 kJ/min B.63.05 kJ/min *C.128.46 kJ/min
D.80 kJ/min
Solution:
COP= EER /3.412 = 8/ 3.412 = 2.34
COP= QL /QH QL = 2.34 = 90 / QH 90
QH=128.46Kj/min
An air-conditioning system is used to maintain a house at 75OF when the temperature
outside is 95OF. The house is gaining heat through the walls and windows at a rate of
1250 Btu/min, and the heat generation rate within the house from people, lights and
appliances amounts to 350 Btu/min. Determine the minimum power input required for
this air-conditioning system.
A.10.06 hp B.1.36 hp *C.1.41 hp D.7.94 hp

Solution:
QL= 1250 + 350 = 1600 Btu/min
COP= TL / TH TL= (75 + 460)/(95 + 460) - (75+460) = 26.75
W= QL /COP = (1600 / 26.75) / 42.4 = 1.41 hp
A refrigeration system is to cool bread loaves with an average mass of 450 g from 22 OC
to -10OC at a rate of 500 loaves per hour by refrigerated air. Taking the average specific
and latent heats of bread to be 2.93 kJ/kg, OC and 109.3 kJ/kg, respectively, determine
the product load.
A.541.7 kJ/min B.351.6 Kj/min *C.761.5 kJ/min
D.409.9 kJ/min
Solution:
Mbread= (500 breads/h) (0.45 kg / bread) = 225 kg/h
Qtotal= Qbread + Qfreezing
= (mcpT)breadl (mhlatent)bread = (225)(2.93)[22-(-10)] l (225)(109.3)
Qtotal= 45,688.5 kJ/h = 761.5 kJ/min
A house that was heated by electric resistance heaters consumed 1200 kWh of electric
energy in a winter month. If this house were heated instead by a heat pump that has an
average performance factor, PF of 2.4, determine how much money the homeowner
would be saved that month. Assume a price of 0.085$/kWh for electricity.
A. $42.5 *B. $59.50 C.$109 D.$97.75
Solution:
W= QH/PF = 1200kWh / 2.4 = 500 kWh
$ Savings per month = (1200 500) (0.085) = $59.50
An ammonia simple saturation cycle operates with a suction pressure of 291.6 kPa and
a condenser pressure of 1204 kPa develops 15 tons of refrigeration. Determine the
theoretical horsepower of the compressor.
The following enthalpies have been found: condenser entrance = 1653 kJ/kg, exit
=346.6 kJ/kg, compressor entrance = 1450.2 kJ/kg, exit= 1653kJ/kg.
A.7.23 hp *B.13 hp C. 15 hp D.8.23 hp
Solution:
m= Qe/ (h1 h4) = (15 x 3.52) / (1450.2 346.6) = 0.0478 kg/s
W= m (h2 h1) = (0.0478)(1653 1450.2) / 0.746 = 13 hp
An ammonia ice plant operates between a condenser temperature of 35 OC and
evaporator of -15OC. It produces 10 metric tons of ice per day from water at 30OC to ice
at -5OC. Assuming simple saturation cycle, determine the horsepower of the motor if
the adiabatic efficiency of the compressor c=0.85 and mechanical efficiency m=0.95.
The specific heat of ice is 2,094kJ/kg. OC and the latent heat is 335kJ/kg.
From the table for ammonia the following enthalpies are: condenser entrance = 1703
kJ/kg, exit= 366.1 kJ/kg; compressor entrance= 1443.9kJ/kg, exit = 1703kJ/kg
A.17.68 hp B.18.61 hp C.15.5 hp *D.21.9 hp
Solution:
qe=cpa (te tf) lhlatentlcph(tf ts)
= (4.187) (30 0) + 335 + (2.094) [(0- (-5)] = 471.08 kJ/kg
Qe= (10 x 1000) (471.08) / 24 = 196,283.33 kJ/hr= 54.523kJ/s
m= Qe / (h2 h4) = (54.523)/ (1443.9 366.1) = 0.05059kg/s
W= m (h2 h1) = (0.0509) (1703 1443.9)/ 0.746 = 17.68 hp
Wmotor= 17.68 / (0.85) (0.95) =21.9 hp
A Freon 22 air conditioning under standard operating conditions of 35OC is condensing
and 5OC evaporating temperatures. The volume flow rate entering the compressor is
23.72 L/s. determine the refrigerating capacity if the refrigerating effect is 164 kJ/kg.
From the table for R22 the specific volume at the compressor entrance is 40.36L/kg.
A.393.3 TR B.79.3 TR C.96.4 TR *D.27.4 TR
Solution:
m= V1/v1= 23.72 / 40.36 = 0.5877 kg/s
Qe= m (qe) = 0.5877 (164)/ 3.52 = 27.4 TR

The refrigerant volume flow rate at the entrance of compressor were obtained from a
test on a twin cylinder single acting 15 cm x 20 cm, 320 rpm compressor ammonia
refrigerating plant 33 L/s. Determine the volumetric efficiency of the compressor.
A.77.65% *B.87.6% C.97.6 TR D.65.65%
Solution:
VD = ( D2 L/4) N= ( /4) (0.15)2(0.2) (320) (2) = 2.26 m3/min
nv =V1/VD = 33/2.26 (1000/60) = 0.876 or 87.6 %

A twin cylinder ammonia compressor with volume displacement of 14,726 cm 3 operates


at 300 rpm. Condenser and evaporator pressure are 1200 kPa and 227 kPa
respectively. Specific volume of refrigerant at the entrance of compressor is 528.26
L/kg. Compression process is polytrophic with a 1.20 and clearance factor of
compressor is 2%. Determine horsepower required.
A.60hp B.70hp C.80hp D.90hp
Solution:
nv= 1+c-c(p2/p1)1/n= 1 + 0.02 0.02(1200/227)1.20= 0.8725
VD= ( D2 L/4) N= (0.014726) (320) (2) = 9.424 m3/min
V1= (nv) VD- (0.8725) (9.424) - 8.2224 m3/min
W= (n p1 V1 /1-n) [(p2/p1) n-1/n-1]
= [(1.20 x 2.27 x 8.2224)/ (1-1.20)] [(1200/2.27)1.2-1/1.2 -1] = 3582 kJ/min
W= 3582/ (60) (0.746) = 80 hp

A reversed Carnot cycle has a refrigerating COP of 2.5. Determine the ratio T H/TL?
A.1.4 B.1.5 C.1.25 D.1.2
Solution:
COP= TH/ (TH-TL)
1/COP = (TH-TL)/TL= TH/TL-1
TH/TL =1 + 1/COP =1 +1/2.5 =1.4
Three thousand cubic feet per minute of air are circulated over an air-cooled condenser.
If the load on the condenser is 64,800 Btu/hr, compute the temperature rise of the air
passing over the condenser. Specific volume of standard air (13.34ft3/lb)
A.10OF B.15OF C.20OF D.25OF
Solution:
Qc= m c t
t = Qc/ m c = 64,800/ [3000 (60) / (13.34)] (0.24) = 20OC

Saturated vapor ammonia at -16OC (h1 = 1442.60 kJ/kg) leaves the evaporator and
enters the compressor at -6OC (h1 = 1465kJ/kg. The refrigerant leaves the condenser as
saturated liquid at 40OC (h4= 390.6 kJ/kg) and enter the expansion valve at 35OC (h5
=366.1kJ/kg). Heat rejected from the compressor amount to 50kW. The work to
compressor is 208kJ/kg, while the heat loss from compressor is 33kJ/kg. If 95kJ/kg of
heat are lost in the piping between the compressor discharge and condenser inlet,
determine the refrigerating capacity of the system.
A.49.5 TR B.46.61 TR C.12.88 TR D.13.24 TR
Solution:
Solving for the enthalpy at the exit of compressor using energy balance about the
compressor
h2 = h1 + w - qwc = 1465 + 208 33= 1640 kJ/kg
Solving for the enthalpy at the entrance of condenser using energy balance about piping
from compressor exit to condenser entrance
h3= h2 q2-3= 1640 95 = 1545kJ/kg
Solving for heat rejected in condenser using energy balance about the condenser
qc = h3 h4 = 1545 390.6 = 1154.4kJ/kg
m = Qc/qc = 50/1154.4 = 0.0433kg/s
Solving for refrigerating effect using energy balance about the evaporator h5 h4 =
366.1kJ/kg

qe= h1 h5 = 1442.6 366.1 = 1076.5kJ/kg


Therefore, the refrigerating capacity
Qe= m qe = [(0.0433) (1076.5)] / 3.52 = 13.24 TR

An actual refrigerating cycle using R12 as working fluid, the refrigerant flow rate is 0.05
kg/s. Vapor enters the expansion valve at 1.15 MPa, 40OC (h = 238.5kJ/kg) and leaves
the evaporator at 175 kPa, -15OC (h = 345kJ/kg). The electric input to motor driving the
compressor is measured and found 3.0 kW. Motor efficiency at this load is 92% and
mechanical efficiency 82%. Determine the actual coefficient of performance for this
cycle.
A.1.58 B.2.36 C.1.78 D.1.34
Solution:
Qe= (0.05) (345 - 238.5) = 5.325kW
W = (3) (0.92) (0.82) = 2.26kW
COPactual= 5.325 / 2.26 = 2.36
An ammonia refrigeration system the temperature in the evaporator is -12OC and the
ammonia at the evaporator entry is 0.1511 dry while at exit is 0.95 dry. If the rate of
ammonia circulation is 5.64 kg/min, determine the refrigerating capacity of the system.
Enthalpy of saturated liquid and vapor at -12OC is 144.929kJ/kg and 1447.74kJ/kg
respectively.
A.17.82 B.34.82TR C.27.82TR D.4.82TR
Solution:
h4= hf4 + x4 (hg4 hf4) = 144.929 + (0.1511) (1447.74 144.929) = 341.78
h1= hf1 + x1 (hg1 hf1) = 144.929 + (0.95) (1447.74 144.929) = 1382.6kJ/kg
Qe = m (h1 h4) = (5.64) (1382.6 341.78) / 211 = 27.82 TR

A two stage cascade vapor compression refrigeration system uses ammonia in the low-
temperature loop and R-12 in the high-temperature loop. The ammonia provides 15
tons cooling. If the high-loop temperature requires 10.12 kW compressor power and
low-loop 15.93 kW, determine the COP of the system.
A.2.027 B.5.22TR C.3.314TR D.9.1TR
Solution:
Wtotal= 10.12 + 15.93 = 26.05 kW
COP = 15 (3.52) /26.05 = 2.027

When a man returns to his well-sealed house on a summer day, he finds that the house
is at 32OC. He turns on the air conditioner, which cools the entire house to 20 OC in
15min. If the COP of the air-conditioner system is 2.5, determine the power drawn by
the air conditioners. Assume the entire mass within the house is equivalent to 800 kg of
air for which c= 0.72kJ/kg OC
A.7.68Kw B.3.07kW C.19.2kW D.12.03kW
Solution:
Qe = m c (t1 t2) / time = (800) (0.72) (32 20) / (15 x 60) = 7.68kW
W= Qe / COP = 7.68 / 2.5 = 3.07kW

It is desired to double the COP of a reversed Carnot engine for cooling from 5.0 by
raising the temperature of the heat addition while keeping the temperature of the heat
rejection constant. By what percentage must the temperature of heat addition be
raised?
A.10.1% B.9.1% C.8.1% D.7.1%
Solution:
Percent increase of the temperature = (TL TL) TL = TL / TL 1 of heat addition
Original COP: 5 = TL / (TH TL); (TH TL) TL = 1/5; TH / TL -1 = 1/5 = 02; TH / TL =
1.2
eq.1
Doubling the COP:
10 = TL / (TH TL); (TH TL) TL = 1/10; TH / TL 1 =1/10 = 0.1; TH/TL=1.1
Dividing eq.1 and eq.2:
(TH/TL) (TH/TL) = 1.2 / 1.1 = 1.091
Percent Increase of heat addition = 1.091 1 = 0.091 or 9.1%
An ammonia water-cooled compressor receives the refrigerant at specific volume 62
L/kg. It has a piston displacement rate of 3m3/min. If a squirrel cage motor is running at
1200 rpm drives the compressor and average piston speed is 490m/min, calculate size
of cylinder bore.
A.20.4 cm B.26.0 cm C.16.13 cm D.13.6 cm
Solution:
Piston speed = 2LN = 490 = 2(L) (1200); L= 0.204 m = 20.4 cm
Vp = ( D2 L/4) N = (/4) D2 (0.204) (1200) =5
D = 0.1613m = 16.13cm
If the initial volume of an ideal gas is compressed to one-half its original volume and to
twice its temperature, the pressure:
A. Doubles B. Quadruples C. Remains constant
D. Halves
Solution:
P1V1/T1 = P2V2/T2
P1V1/T1= P2 (1/2 V1) / (2T1)
P2 = 4 P 1
If the gage pressure of a medium is 30kPa (vacuum) and the atmospheric pressure is
101.3kPa, the absolute pressure will be:
A.131.3kPa B.-71.3kPa C.71.3kPa D.-131.3kPa
Solution:
Pabs = Patm - Pvac
= 101.3kPa - 30kPa = 71.3kPa
If a particle has a velocity of 4 meters per second and a kinetic energy of 144 Joules,
then the mass, in kilograms of this particle must be:
A.44 B.16 C.18 D.24
Solution:
KE = (1/2) (m) (V2) / 2k
144 (1/2) (m) (4)2 / [(2) (1)]
m = 18kg
A condenser vacuum gauge reads 715 mm Hg when the barometer stands at 757 mm
Hg. State the absolute pressure in the condenser in kN/m or kPa.
A.5.6kPa B.5.9kPa C.6.5kPa D.5.2kPa
Solution:
Pabs = Patm Pvac = 757 715 = 42 mm Hg (101.325kPa/760 mm Hg) = 5.60kPa
Determine the force in Newton in a piston of 465 mm2 area with a pressure of
0.172MPa.
A.65N B.72N C.80N D.111
Solution:
F= PA (0.172MPa) (105 Pa/MPa) (465 mm2) (m2 / 105 mm2)
F = 79.98N
One piston of a hydraulic press has an area of 1cm2. The other piston has an area of
25cm2. If a force of 130N is applied on the smaller piston, what will be the total force on
the larger piston is both piston surfaces are the same level?
A.6N B.175N C.3750N D.4250N
Solution:
F1/A1 = F2/A2
(150/1) = (F2/ 25)
F2 = 3750N
If the pressure of a confined gas at a constant temperature is tripled, what will happen
to the volume?
A. The volume will be tripled B. The volume will
remain
C. The volume will be reduced to one-third of its original value D. The
volume is constant
Solution:
P1V1 = P2V2
P1V1 = (3P1) V2
V2 = (1/3) V1

The work done on air is 10.86kJ/kg, determine the compressor power if it is receiving
272 kg/min if air.
A.36.72 hp B.49.23 hp C. 29.54 hp D.66 hp
Solution:
W= (10.86) (272.60) = 49.232 kJ/s or kW
= 49.232 kW (1 hp/ 0.746kW) = 65.99 hp
A water tank of 18 ft long and 4 ft high, calculate the pressure at the bottom of the tank.
A.1.733psi B.1.999psi C.2.337psi D.3.773psi
Solution:
P= (62.4 lbf/ft3) (4ft) (1ft2 / 144 in2) = 1.733psi
The pressure of 750 mm Hg in kN/m2
A.90 B.100 C.103 D110
Solution:
P= 750 mm Hg (101.325 kPa/ 760 mm Hg)
= 99.99 kPa
A double purpose tank 18 ft wide, 24 ft long and 4 ft depth is filled with water. What is
the weight of water in the tank in long tons?
A.49tons B.48tons C.54tons D.50tons
Solution:
W = (62.4 lb/ft2) [(18) (24)(4)] ft3 = 107,827.2 lbf
= 107,827.2 lb (1ton/ 2200 lb) = 49tons

Oil flow through a 16 tubes on a single cooler with a velocity of 2 m/s, the internal
diameter of the tube is 30mm and oil density is 0.85gm/ml. Find the volume flow in liters
per sec.
A.22.62 B.32.22 C. 62.22 D.42.62
Solution:
Volume flow rate = (3.1416) (0.015)2 (2) (16) = 0.02262 m3/s or 22.62 liters/s

A substance temperature was 620OR. What is the temperature in OC?


A.50.7 B.45.54 C.71.11 D.94.44
Solution:
T, OC = [(620 460) 32] (5/9) = 71.11

Unknown volume of container gas of gas of 1 atmosphere is allowed to expand to


another container of 10 m3 volume at 500 mm Hg at constant temperature. Find the
unknown volume.
A.6.58 m3 B.6.75 m3 C.5.67 m3 D.7.65 m3
Solution:
P1V1 P2V2
(760) V1 = (500) (10)
V1= 6.58 m3

An iron block weighs 5 Newton and has volume of 200 cm 3. What is the density of the
block?
A. 2458kg/m3 B. 2485 kg/m3 C. 2584 kg/m3 D.
2549 kg/m 3

Solution:
Density = specific weight [at sea level or near the surface of the earth]
= (5N/200 cm) (10 cm3/m3) (1kg/9.8066N) = 2549.30 kg/m3

If air is at a pressure of 22.22 psia and at temperature of 800 OR, what is the specific
volume?
A.11.3 ft3/lbm B.33.1 ft3/lbm C.13.3 ft3/lbm D.31.3 ft3/lbm
Solution:
Pv = RT
v = (53.34) (800) / [(22.22) (144)] = 13.33 ft3/lbm

The specific gravity of mercury is 13.55. What is the specific weight of mercury?
A.123.9 kN/m3 B.139.2 kN/m3 C.139.9 kN/m3
D.193.2 kN/m3
Solution:
? = (13.55) (9.8066) = 132.88 kN/m3
The equivalent weight of mass 10 kg at a location where the acceleration of gravity is
9.77 m/sec2
A. 97.7N B.79.7N C.77.9N
D.977N
Solution:
Weight = mg/k = [(10) (9.77)] /1 =97.7N
A transportation company specializes in the shipment of pressurized gaseous materials.
An order is received from 100 liters of a particular gas at STP (32OF and 1 atm). What
minimum volume tank is necessary to transport the gas at 80OF and maximum pressure
of 8 atm?
A.16 liters B.14 liters C.10 liters D.12 liters
Solution:
P1V1/ T1 = P2V2/ T2
[(1) (100)/ (32 + 460)] / [(8) (V2)/ (80 + 460)
V2 = 13.72 liters
100 g of water are mixed with 150 g of alcohol (density = 790 kg/m 3). What is the
specific volume of the resulting mixtures, assuming that the fluids mixed completely?
A.0.82x10-3 cu.m/kg B.0.88x10-3 cu.m/kg C.0.63x10-3 cu.m/kg D.1.16x10-3
cu.m/kg
Solution:
Mass of mixtures = 100 + 150 =250 g
Volume of mixture = [(0.100) / (1000)] + [(0.150)/ (790)] = 0.00029 m 3
Specific volume of mixture = (0.00029)/ (0.250) = 1.16x10-3 cu.m/kg
How much does 30 lbm weigh on the moon? (gmoon = 5.47 ft/s2
A.20 lbr B.3.2 lbr C.3.4 lbr
D.5.096 lbr
Solution:
Weight = mg/k = {[(30) (5.42)]/32.174} = 5.1 lbf
A 10 kg block is raised vertically 3 meters. What is the change in potential energy?
A.320 J B.350 kg-m2/s2 C.294 J
D.350 N-m
Solution:
PE = mgs/k = {[(10) (9.8066) (3)]/1} = 294.2 J
How many cubic meters is 100 gallons of liquid?
A.3.7850 cu.m B.0.1638 cu.m C.0.3785 cu.m
D.1.638 cu.m
Solution:
100gal (785 liters/gal) (1m3/1000 liters) = 0.3785 m3
Steam turbine is receiving 1014 lbm/hr of steam, determine the horsepower output of the
turbine if the work done by steam is 251 Btu/lbm
A.100 hp B.462.7 hp C.200 hp D.6002.7 hp
Solution:
W= (251 Btu/lbm) (1014 lbm/hr) 1hp/2545 Btu/hr) = 100 hp
What is the resulting pressure when one pound of air at 15 psia and 200 OF is heated at
constant volume to 800OF?
A.52.1 psia B.15 psia C.28.6 psia D.36.4 psia
Solution:
P1/T1 = P2/T2
P2 = [(800+460) / (200+460)] (15) = 28.64 psia
A bicycle tire has a volume of 600 cm3. It is inflated with carbon dioxide to pressure of
551.43kPa at 20OC. How many grams of CO2 are contained in the tire? Rcoz = 0.18896
kJ/kg.K
A.5.98 g B.6.43 g C.4.63 g D.3.83
g
Solution:
m = rv/RT = [(551.43) (600)/ (102)] / [(0.18896) (20+273)] = 0.00598 kg or 5.98 g

Is the most common dryer used which consist of rotating cylinder inside which the
materials flow while getting in contact with hot gas.
A. Tower dryer B. Centrifugal dryer C. Tray dryer D.
Rotary dryer
ANSWER: D
Is the ratio of the mass water-vapor in air and the mass of air if it is saturated is called:
A. Humidity ratio B. Mass ratio C. Vapor ratio D.
Relative humidity
ANSWER: D
The hands feel painfully cold when the skin temperature reaches:
A.8OC B.10OC C.12OC D.14OC
ANSWER: B
The refrigerant used in steam jet cooling is:
A. Steam B.R-11 C. Ammonia D. Water
ANSWER: D
The total heat of the air is a function of
A. WB temperature B.DP temperature C.DB temperature D.WB
depression
ANSWER: A
Boiling point of Freon-12 at atmospheric pressure is:
A. 21OF B.15OF C.5OF D.28OF
ANSWER: A
Which of the following is NOT a type of water cooled condenser in refrigeration?
A. Double pipe B. Double shell C. Shell and coil
D. Shell and tube
ANSWER: B
Component of absorption refrigeration system in which the solution is cooled by cooling
water:
A. Rectifier B. Generator C. Evaporator D. Absorber
ANSWER: D
Cascade refrigeration cycle is often used in industrial process where objects must be
cooled to temperature below:
A. -46OC B. -56OC C. -66OC D. -76OC
ANSWER: A
Type of refrigerant control designed to maintain a pressure difference while the
compressor is operating.
A. Thermostatic expansion valve B. Automatic expansion valve
C. Using low side float flooded system D. Capillary tube
ANSWER: D
As a rule of thumb, for a specified amount of compressed air, the power consumption of
the compressor decreased by
for each 3OC drop in the temperature inlet air to the compressor.
A. 1% B. 1.5% C. 2% D. 2.5%
ANSWER: A
Modern way of detecting air compressor leak is by using
A. Soup and water B. Air leak detector C. Acoustic leak detector D. Ammonia
leak detector
ANSWER: C
For foundation of stacks, the maximum pressure on the soil is equal to the pressure due
to the weight and the .
A. Soil movement B. Wind movement C. Ground movement D.
Engine movement
ANSWER: B
Foundation bolts of specified size should be used and surrounded by a pipe sleeve with
an inside diameter of at least
A. 3 times the diameter of engine bolt B. 3 times the diameter of anchor
bolt
C. 2 times the diameter of engine bolt D. 2 times the diameter of anchor
bolt
ANSWER: B
For multi stage compression of an ideal Brayton cycle, the backwork ratio will
A. Increase B. Decrease C. Remain the same D.
none of these
ANSWER: B
Type of turbine that has a specific speed below 5
A. Impulse turbine B. Propeller turbine C. Francis turbine D. Deriaz turbine
ANSWER: A
A high discharge type of turbine
A. Impulse turbine B. Francis turbine C. Propeller turbine
D. Deriaz turbine
ANSWER: C
Use to minimize the speed rise due to a sudden load rejection
A. needle valve B. wicket gate C. Shut-off valve
D. jet deflector
ANSWER: D
Is the speed of a turbine when the head on the turbine is one meter?
A. specific speed B. rated speed C. utilized speed
D. unit speed
ANSWER: D
Is a fluid property which refers to the intermolecular attraction by which the separate
particles of the fluid arc held together?
A. Cohesion B. Adhesion C. Surface tension D.
hypertension
ANSWER: A
Which of the following is NOT the cause of black smoke in diesel engine?
A. fuel valve open too long B. high compression pressure C. carbon in
exhaust pipe D. overload on engine

BB94-BB102
Which of the following is not a method of starting a diesel engine?
Manual rope, crank and kick
Electric (battery)
Compressed air
Using another generator
ANSWER D
Two-stroke engine performs to complete one cycle
Suction and discharge stroke
Power and exhaust stroke
Compression and power stroke
Suction and exhaust stroke
ANSWER C
A type of geothermal plant used when there is a presence of brine extracted from
underground
Dry geothermal plant
Double-flesh geothermal plant
Single-flash geothermal plant
Binary geothermal plant
ANSWER D
Is the most important safety device on the power boiler
Check valve
Gate valve
Safety valve
Globe valve
ANSWER C
During hydrostatic test, the safety valves should be
Removed
Open
Closed
Partially closed
ANSWER A
Where deaerating heaters are not employed, it is recommended that the temperature of
the feed less than
197C
102C
104C
106C
ANSWER A
Is a reaction during which chemical energy is released in the form of heat?
Cosmic reaction
Ethnic reaction
Endothermic reaction
Exothermic reaction
ANSWER D
By reheating the steam in an ideal Rankine cycle the heat rejected will:
Increase
Decrease
Remains the same
None of these above
ANSWER A
By increasing the boiler pressure in Rankine cycle the moisture content at boiler exit
will:
Increase
Decrease
Remains the same
None of these above
ANSWER A
Presently the highest steam temperature allowed at the turbine inlet is about
340C
520C
620C
1020C
ANSWER C
The most common gases employed in Stirling and Ericsson cycles are:
Air and helium
Oxygen and helium
Hydrogen and helium
Nitrogen and helium
ANSWER C
In most common designs of Gas turbine, the pressure ratio ranges from
10 to 12
11 to 16
12 to 18
15 to 20
ANSWER B
In Brayton cycle, the heat is transformed during what process?
Constant temperature
Isentropic process
Isobaric process
Isochoric process
ANSWER C
The fuel injection process in diesel engine starts when piston
Is at the TDC
Leaving TDC
Approaches TDC
Halfway of the stroke
ANSWER C
If the cut-off ratio of diesel cycle increases, the cycle efficiency will
Decrease
Increase
Remains the same
None of these
ANSWER A
The fuel used in a power plant that is used during peak periods
Gas
Solid
Liquid
None of these
ANSWER C
Typical compression ratio of Otto cycle is
6
8
10
12
ANSWER B
If Joule Thompson coefficient is equal to zero, then the process will become
Isentropic
Isenthalpic
Isobaric
Isothermal
ANSWER D
If the fluid passed through a nozzle its entropy will:
Increase
Decrease
Remains the same
None of these
ANSWER C
Refrigerants consisting of mixtures of two or more different chemical compounds, often
used individually as refrigerants for other applications
Suspension
Compound reaction
Blends
Mixing of refrigerants
ANSWER C
Pairs of mating stop valves that allow sections of a system to be joined before opening
these valves or separated after closing them
Check valve
Gate valve
Safety valve
Companion valve
ANSWER D
An enclosed passageway that limits travel to a single path
Corridor
Hallway
Lobby
Tunnel
ANSWER A
For Immediate Dangerous to Life or Health (IDHL) the maximum concentration from
which unprotected persons have time to escape within without escape-impairing
symptoms or irreversible health
16 minutes
1 minute
20 minutes
30 minutes
ANSWER D
The volume as determined from internal dimensions of the container with no allowance
for the volume of internal parts
Internal allowance volume
Internal gross volume
Internal interference volume
Internal fots volume
ANSWER B
A waiting room or large hallway serving as a waiting room
Terrace
Test room
Compound room
Lobby
ANSWER D
A continuous and unobstructed path of travel from any in a building or structure to a
public way
Average of aggress
Mean o aggress
Hallway of aggress
Pathway of aggress
ANSWER B
Any device or portion of the equipment used to increase refrigerant pressure
Pressure relief device
Pressure-imposing element
Pressure lift device
Pressure limiting device
ANSWER B
The quantity of refrigerant stored at some point is the refrigeration system for
operational, service, or standby purposes
Pressure vessel
Pumpdown charge
Liquid receiver
Accumulator
ANSWER B
Secondary refrigerant is a liquid used for the transmission of heat, without a change of
state, and having no flash point or a flash point above as determined from ASTM
150F
160F
180F
200F
ANSWER A
A service valve for dual pressure-relief devices that allows using one device while
isolating the other from the system maintaining one valve in operation at all times
Three-way valve
Two-way valve
One-way valve
Four-way valve
ANSWER A
Tubing that is unenclosed and therefore exposed to crhsing, abrasion, puncture, or
similar damage after installation
Protected tubing
Bare tubing
Open tubing
Unprotected tubing
ANSWER D
Refers to blends compromising multiple components of different volatile that, when used
in refrigeration cycles, change volumetric composition and saturation temperature as
they evaporate (boil) or condense at constant pressure
Zeoline
Blending
Composition
Zertropic
ANSWER D
Is a premises or that portion of a premise from which, because they are disabled,
debilitated, or confined, cooperate cannot readily leave without the assistance of others
Institutional occupancy
Public assembly occupancy
Residential occupancy
Commercial occupancy
ANSWER A
Is one in which a secondary coolant is in direct contact with the air or other substance to
be cooled or heated
Double indirect open spray system
Indirect closed system
Indirect open spray system
Indirect vented closed system
ANSWER B
Refrigerant number R-744 is:
Butane
Carbon monoxide
Propane
Carbon dioxide
ANSWER D
Refrigerant number R-1150 is:
Propylene
Ethane
Ethane
Methyl formale
ANSWER B
Refrigerant number R-40 is:
Chlorodifluoromethane
Difluoromethane
Ammonia
Chloromethane
ANSWER D
When the air duct system serves several enclosed spaces, the permissible quantity of
refrigerant in the system shall not exceed the amount determined by using the total
volume of those spaces in which the airflow cannot be reduced to less than of
its maximum when the fan is operating
One-quarter
One half-quarter
Three-quarter
One fourth-quarter
ANSWER A
The space above a suspended ceiling shall not be include 1 in calculating the
permissible quantity of refrigerant in the system unless such space is continuous and is
part of the air return system
Partition
Plenum
Separator
Plate divider
ANSWER B
Which of the following is NOT a possible location of service valve?
Suction of compressor
Discharge of compressor
Outlet of liquid receiver
Outlet of condenser
ANSWER D
A cool to serve with evaporator that is use to prevent the liquid refrigerant entering the
compressore
Accumulator
Liquid superheater
Drier loop
Liquid suction heat exchanger
ANSWER C
A type of valve connected from discharge of compressore directly to suction that is
normally closed and will open automatically only if there is high discharge pressure
Check valve
Solenoid valve
King valve
Relief valve
ANSWER B
Use to increase the capacity of condenser
Water regulating valve
Desuperheating coils
Liquid-suction heat exchanger
Condenser heating coils
ANSWER B
Is use to subcooled the refrigerant from the condenser
Liquid subcooler
Condenser subcooler
Desuperheating coils
Liquid receiver
ANSWER A
Which of the following is NOT a part of low pressure side in refrigeration system?
Liquid line
Refrigerant flow control
Evaporator
Suction line
ANSWER A
Which of the following is NOT a part of high pressure side in refrigeration system?
Compressor
Condenser
Liquid line
Suction line
ANSWER D
Which of the following is NOT a part of condensing unit?
Compressor
Discharge line
Condenser
Liquid line
ANSWER D
By subcooling the refrigerant in refrigerant system, the compressor power per unit mass
will
Increase
Decrease
Remains the same
None of these
ANSWER C
Superheating the refrigerant in refrigeration system without useful cooling, the
refrigeration effect per unit mass will
Increase
Decrease
Remains the same
None of these
ANSWER C
Which of the following is NOT a type of air-cooled condenser?
Shell and tube
Natural draft
Forced draft
Induced draft
ANSWER A
A type of refrigerant control typically used in household refrigeration
Thermostatic expansion valve
Automatic expansion valve
Capillary tube
High side float
ANSWER C
Type of condenser that operates like cooling tower
Air-cooled condenser
Evaporative condenser
Shell and tube condenser
Water-cooled condenser
ANSWER B
The major problem of heat pump is
Refrigerant used
Outside air
Supply air
Frosting
ANSWER D
Dominant refrigerant used in commercial refrigeration system
R-11
R-22
R-12
R-502
ANSWER D
Cascade refrigeration system are connected in
Series
Parallel
Series-parallel
Parallel-series
ANSWER A
Is use to heat up the solution partially before entering the generator in absorption
refrigeration system
Rectifier
Absorber
Regenerator
Pump
ANSWER C
The COP of actual absorption refrigeration system is usually
Less than 1
Less than 2
Less than 3
Less than 4
ANSWER A
Sight glass is often located at:
Discharge line
Liquid line
Between condenser and liquid receiver
Suction line
ANSWER B
Use to detects a vibration in current caused by the ionization of decomposed refrigerant
between two opposite-charged platinum electrodes
Electronic detector
Halide torch
Bubble method
Pressurizing
ANSWER B
Joints and all refrigerants containing parts of a refrigerating system located in an air
duct carrying conditioned air to and from an occupied space shall be constructed to
withstand a temperature of without leakage into the airstream
550F
600F
650F
700F
ANSWER D
Refrigerant piping crossing an open space that affords passageway in any building shall
be less than above the floor unless the piping is located against the ceiling of
such space and is permitted by the authority having jurisdiction
2.2 m
3.2 m
4.2 m
5.2 m
ANSWER A
Methyl chloride shall not be in contact with
Aluminum
Zinc
Magnesium
All of these
ANSWER D
Shall not be in contact with any halogenated refrigerants
Aluminum
Zinc
Magnesium
All of these
ANSWER C
Are suitable for use in ammonia system
Copper
Aluminum and its alloy
Plastic
Cast iron
ANSWER B
If a pressure-relief device is used to protect a pressure vessel having an inside
dimension of 6 in or less, the ultimate strength of the pressure vessel so protected shall
be sufficient to withstand a pressure at least the design pressure
2 times
3 times
4 times
5 times
ANSWER B
Seats and discs shall be limited in distortion, by pressure or other cause, to a set
pressure change of not more than in a span of five years
1%
5%
10%
50%
ANSWER B
Liquid receivers, if used, or parts of a system designed to receive the refrigerant charge
during pumpdown charge. The liquid shall not occupy more than of the volume
when temperature of the refrigerant is 90F
80%
85%
90%
95%
ANSWER C
The discharge line (B4) shall be vented to the atmosphere through a fitted to its
upper extremity
Nozzle
Convergent-divergent nozzle
Pipe
Diffuser
ANSWER D
Convert fossil fuels into shaft work
Nuclear power plant
Gas turbine power plant
Dendrothermal power plant
Thermal power plant
ANSWER D
Ultimate strength drops by 30% as steam temperature raises from for unalloyed
steel
300 to 400C
400 to 500C
600 to 700C
700 to 800 C
ANSWER B
Recent practice limits steam temperature to
438C
538C
638C
738C
ANSWER A
In a closed feed water heater, the feed water pass through
Inside the tube
Outside the tube
Inside the shell
Outside the shell
ANSWER A
In use if extracted steam upon condensation gets subcooled
Trap
Deaerator
Filter
Drain cooler
ANSWER D
Needs only single pump regardless of number of heaters
Open heater
Closed heater
Mono heater
Regenerative heater
ANSWER B
Is also known as deaerator
Open heater
Closed heater
Reheat heater
Regenerative heater
ANSWER A
Dissolve gases like makes water corrosive react with metal to form iron oxide
O2 and N2
O2 and CO
O2 and CO2
N2 and SO2
ANSWER C
A cycle typically used in paper mills, textile mills, chemical factories, sugar factories and
rice mills
Cogeneration cycle
Combined cycle
By-product cycle
Cascading cycle
ANSWER A
When process steam is basic need and power is byproduct, this cycle is known as
Cogeneration cycle
Combined cycle
By-product cycle
Cascading cycle
ANSWER C
A type of turbine employed where steam continuously extracted for process heating
Back pressure turbine
Gas turbine
Steam turbine
Passout turbine
ANSWER D
Which of the following is used for Binary cycle power generation for high temperature
application
Mercury
Sodium
Potassium
All of these
ANSWER D
critical temperature of mercury is
1160C
1260C
1360C
1460C
ANSWER D
Critical pressure of mercury is
100 Mpa
108 Mpa
128 Mpa
158 Mpa
ANSWER B
Method used in converting heat directly to electricity by magnetism
Electronic induction
Magnetodynamic
Magnetohyrdodynamic
Thermoelectric
ANSWER C
Which of the following is NOT a material used for thermoelectric elements
Bismuth telluride
Lead telluride
Zinc telluride
Germanium
ANSWER C
A type of coal formed after anthracite
Lignite
Bituminous
pear
graphite
ANSWER D
Which of the following is lowest grade of coal?
Peat
Lignite
Sub-bituminous
Bituminous
ANSWER B
Which of the following helps in the ignition of coal?
Moisture
Ash
Fixed carbon
Volatile matter
ANSWER D
is the ratio of fixed carbon and volatile matter
Air-fuel ratio
Fuel ratio
Combustion ratio
Carbon-volatile ratio
ANSWER B
A suspension of a finely divide fluid in another
Filtration
Floatation
Emulsion
Separation
ANSWER C
Contains 90% gasoline and 10% ethanol
Gasohol
Gasonol
Gasothanol
Gasethanol
ANSWER A
Process used commercially in coal liquefaction
Tropsch process
Fisher process
Fisher-Tropsch process
Mitch-Tropsch process
ANSWER C
Is an organic matter produced by plants in both land and water
Bio-ethanol
Biomass
Petroleum
Biodegradable
ANSWER B
In thermal power plant, induced draft fans are located at the
Exit of furnace
Foot if the stack
Above the stock
Top of the stock
ANSWER B
In thermal power plant, forced draft fans are installed at the
Foot of the stack
Top of the stack
Exit of the preheater
Inlet of the preheater
ANSWER D
Known as drum less boiler
La Mont boilers
Fire tube boiler
Forced circulation boiler
Once-through boiler
ANSWER D
Reduces the steam temperature by spraying low temperature water from boiler drum
Reheater
Preheater
Desuperheater
Superheater
ANSWER C
Carbon dioxide can be removed by:
Deaeration
Aeration
Evaporation
Vaporization
ANSWER B
Is often used to absorb silica from water
Sorbent
Rectifier
Silica gel
Magnesium hydroxide
ANSWER D
Presence of excess hydrogen ions makes the water
Acidic
Alkalinity
Base
Hydroxicity
ANSWER A
PH of water varies with
Pressure
Temperature
Density
Volume
ANSWER B
Ph value of is usually maintained for boiler water to minimized corrosion
8.5
9.5
10.5
11.5
ANSWER C
What type of turbine that has a degree of reaction of?
Impulsive turbine
Reaction turbine
Rarsons turbine
Deriaz turbine
ANSWER C
The cooling water is made to fall in series of baffles to expose large surface area for
steam fed from below to come in direct contact
Spray condenser
Surface condenser
Jet condenser
Barometric condenser
ANSWER D
Show the variation of river flow (discharge) with time
Hydrograph
Hyetograph
Mass curve
Flow duration curve
ANSWER A
Is an open channel erected on surface above the ground
Canal
Tunnel
Penstock
Flume
ANSWER D
Type of turbine used up to 300 m head
Impulsive turbine
Francis turbine
Propeller turbine
Deriaz turbine
ANSWER D
A turbine that has a diagonal flow
Impulsive turbine
Francis turbine
Propeller turbine
Deriaz turbine
ANSWER D
Oil is optimized either by air blast or pressure jet at about
60 bar
70 bar
80 bar
90 bar
ANSWER B
Type of solid injection that use single pump supplies fuel under high pressure to a fuel
header
Common rail injection
Individual pump injection system
Distributor system
Single rail injection
ANSWER A
Water flow in diesel engine that is caused by density differential
Thermosiphon cooling
Thermostat cooling
Pressurized water cooling
Evaporating cooling
ANSWER A
Type of lubrication system in diesel engine in which oil from pump is carried to a
separate storage tank outside the engine cylinder and used for high capacity engine.
Mist lubrication system
Wet pump lubrication system
Splash system
Dry sump lubrication system
ANSWER D
Produces extreme pressure differentials and violent gas vibration
Vibration
Detonation
Explosion
Knocking
ANSWER B
In a spark ignition engine, the detonation occurs near the
End of combustion
Middle of combustion
Beginning of combustion
Beginning of interaction
ANSWER A
In a compression ignition engine, the detonation occurs near the
End of combustion
Middle of combustion
Beginning of combustion
Beginning of interation
ANSWER C
Morse test is use to measure the of multi-cylinder engine
Brake power
Indicated power
Friction power
Motor power
ANSWER B
Ignition delay can be minimized by adding to decrease engine knocking
Ethel ether
Ethyl chloride
Ethyl nitrate
Ethyl oxide
ANSWER C
For the submerged plane surface, the point on the surface where the resultant force
acts is called the
Center of buoyancy
Center of gravity
Center of pressure
Center of attraction
ANSWER C
At any point in fluid at rest, the pressure is the same in all direction. This principle is
known as:
Bernoulli Principle
Archimedes Principle
Pascals Law
Torricellis Law
ANSWER C
The hot-wire manometer is used to measure
Pressure in gases
Pressure in liquids
Wind velocities at airports
Gas velocities
ANSWER D
The pitot static tube measures
The static pressure
The gage pressure
The total pressure
The dynamic pressure
ANSWER A
The terminal velocity of a small sphere setting in a viscous fluid varies as the
First power of its diameter
Inverse of fluid viscosity
Inverse square of the diameter
Inverse of the diameter
ANSWER B
Pressure drag results from
Skin friction
Deformation drag
Breakdown of potential flow near the forward stagnation point
Occurrence of wake
ANSWER A
The pressure coefficient is the ratio of pressure forces to
Viscous forces
Inertia forces
Gravity forces
Surface tension force
ANSWER B
Tranquil flow must always occur
Above the normal depth
Above the critical depth
Below the normal depth
Below the critical depth
ANSWER B
Which of the following head loss coefficient among the following types of entrance?
Bell mouth
Square edge
Reentrant
It depends
ANSWER C
What waste water treatment method involves of algae from stabilization pond effluents?
Sedimentation
Floatation
Filtration
Microscreening
ANSWER D
The number of nozzles will depend on the quantity of steam required by the turbine. If
nozzle occupy the entire arc of the ring, the turbine is said to have
Partially full peripheral admission
One-half full peripheral admission
Maximum peripheral admission
Full peripheral admission
ANSWER D
Tandem compound units may also have two low pressure castings that produces
Single flow
Double flow
Triple flow
Quadruple flow
ANSWER C
A type of turbine used for driving pumps, fans, and other auxiliaries in power plant
commonly operate at exhaust pressure approximating atmospheric
Tandem compound turbine
Passout turbine
Cross-compound turbine
Back pressure turbine
ANSWER D
A governor with 0% regulation is termed as:
Isochronous governor
Synchronous governor
Isenchronous governor
Isobarnous governor
ANSWER A
The speed of regulation for most turbine-generators is adjustable from
2 to 6%
4 to 8%
6 to 10 %
8 to 12%
ANSWER A
Poppet valves of steam turbine are used for extraction pressures of
20 to 120 psig
20 to 150 psig
20 to 130 psig
20 to 140 psig
ANSWER B
When both bearings of steam engine are on one side of the connecting rod, the engine
is referred to as
Center-crank engine
Side crank engine
Under crank engine
Standard crank engine
ANSWER B
When the valve in steam engine is in mid-position of its travel, it will cover the steam
port by an amount known as
Steam lap
Partial lap
Full lap
Angular lap
ANSWER A
A type of governor in steam engine that do not control the actual admission of steam to
the cylinder but controls the pressure of the steam
Flyball governor
Variable cut-off governor
Throttling governor
Shaft governor
ANSWER C
By inter-cooling using two stage compressor of brayton cycle, the backward ratio will
Increase
Decrease
Remains the same
None of these
ANSWER B
On dynamic similitude, the relation which represents the ratio of inertia force to pressure
force is
Froude number
Cauchy number
Euler number
Raynolds number
ANSWER C
What is the maximum velocity in a sewer flowing full?
0.6 m/sec
0.9 m/sec
1.2 m/sec
1.8 m/sec
ANSWER A
A temporary structure constructed to exclude water from the site of the foundation
during its excavation and construction is called:
Caisson
Retaining wall
Coffer dam
Earth dam
ANSWER C
Which is not a physical characteristics of water?
Total suspended and dissolved solids
Turbidity
Color
Hardness
ANSWER D
Which dam is best for weak foundation?
Gravity
Arch
Buttress
Earth
ANSWER C
What is the volume of water which will drain freely the aquifer?
Specific yield
Reservoir yield
Safe yield
Secondary yield
ANSWER A
What is the line defined by water level in a group of artesian walls?
Water table
Peizometric surface
Specific yield
All of the above
ANSWER B
Select the one that is positive indication of pollution of a river
Acidity
Oxygen content
Chloride content
Nitrite content
ANSWER C
Which instruments is used to measure humidity of the atmosphere continuously?
Barograph
Thermograph
Hydrograph
Thermo-hydrograph
ANSWER C
Entrance losses between tanks and pipe or losses through elbows, fittings and valves
are generally expresses as a function of
Kinetic energy
Pipe diameter
Friction factor
Volume flow rate
ANSWER A
The air that contains no water vapor is called
Zero air
Saturated air
Dry air
Humid air
ANSWER C
In psychometric chart, the constant enthalpy lines coincide with constant temperature
lines at temperature
Above 50C
Below 40C
Below 50C
Above 10C
ANSWER C
The amount of moisture in air depends on its
Pressure
Volume
Temperature
Humidity
ANSWER C
The deep body temperature of healthy person is maintained constant at
27C
37C
47C
48C
ANSWER B
Air motion also plays important role in
Surroundings
Cooling
Human comfort
None of these
ANSWER C
During simple heating and cooling process has a humidity ratio
Increasing
Decreasing
Constant
None of these
ANSWER C
The follows a line of constant wet bulb temperature on the psychometric chart
Evaporative cooling process
Condensing process
Direct cooling process
None of these
ANSWER A
A vapor which is not about to condense is called a
Mixture of vapor and liquid
Critical vapor
Superheated vapor
None of these
ANSWER C
Passing from the solid phase directly into vapor phase is called
Condensation
Fusion
Sublimation
None of these
ANSWER C
Robert Boyle observed during his experiments with a vacuum chamber that the
pressure of gases is inversely proportional to their
Temperature
Pressure
Volume
None of these
ANSWER C
Is energy in transition
Heat
Work
Power
None of these
ANSWER A
Is the mode of energy transfer between solid surface and the adjacent liquid or gas
which is in motion, and it involves combine effects of conduction and fluid motion
Conduction
Convection
Radiation
None of these
ANSWER B
Radiation is usually considered as
Surface phenomenon
Surface interaction
Surface corrosion
None of these
ANSWER A
Work is between the system and the surroundings
Work interaction
Energy interaction
Heat interaction
None of these
ANSWER B
Is a process during which the system remains in equilibrium at all times
Quasi-equilibrium
Static equilibrium
Dynamic equilibrium
None of these
ANSWER A
In the absence of any work interactions between a system and its surroundings, the
amount of net heat transfer is equal
To the change in total energy of a closed system
To heat and work
Energy interactions
None of these
ANSWER A
The constant volume and constant pressure specific heats are identical for
Compressible substance
Incompressible substance
Compressible gas
None of these
ANSWER B
The velocity of fluid is zero at the wall and maximum at the center because of the
Velocity effect
Viscous effect
Temperature effect
None of these
ANSWER B
For steady flow devices, the volume of the control volume is
Increase
Decrease
Constant
None of these
ANSWER C
The work done in a turbine is since it is done by the fluid
Positive
Negative
Zero
None of these
ANSWER A
Reheating process in Brayton cycle, the turbine work will
Increase
Decrease
Remains the same
None of these
ANSWER A
Which of the following is the chemical formula of ethanol
C7H16
C2H6O
C7H8
C6H12
ANSWER B
Which of the following is the chemical formula of Heptane?
C7H16
C2H6O
C7H8
C6H12
ANSWER D
Which of the following is the chemical formula of Toluene?
C7H16
C2H6O
C7H8
C6H12
ANSWER C
As the air passes through a nozzle, which of the following will increase?
Temperature
Enthalpy
Internal energy
Mach number
ANSWER D
As the air passes through a diffuser, which of the following will decrease?
Temperature
Enthalpy
Internal energy
Mach number
ANSWER D
As the air passes through a nozzle, which of the following will decrease?
Entropy
Velocity
Internal energy
Mach number
ANSWER C
As the air passes through a diffuser, which of the following will increase?
Density
Entropy
Mach number
Velocity
ANSWER A
As the air passes through a diffuser, which of the following will NOT be affected?
ANSWER B
After passing through a convergent-divergent nozzle, the temperature of air will:
Increase
Decrease
Remains the same
None of these
ANSWER B
After passing though a convergent-divergent nozzle, the density of air will:
Increase
Decrease
Remains the same
None of these
ANSWER B
After passing through a convergent-divergent nozzle, the match number of air will:
Increase
Decrease
Remains the same
None of these
ANSWER B
By increasing the temperature source of Carnot cycle, which of the following will not be
affected?
Efficiency
Work
Heat added
Heat rejected
ANSWER D
By decreasing the temperature source of Carnot cycle, which of the following will not be
affected?
Efficiency
Work
Heat added
Heat rejected
ANSWER C
By superheating the refrigerant in vapor compression cycle with useful cooling, which of
the following will increase? (Use per unit mass analysis)
Condenser pressure
Evaporator pressure
Quality after expansion
Heat rejected from condenser
ANSWER D
By superheating the refrigerant in vapor compression cycle with useful cooling, which
of the following will decrease? (Use per unit mass analysis)
Refrigerant effect
COP
Compressor power
Mass flow rate
ANSWER D
By superheating the refrigerant in vapor compression cycle without useful cooling,
which of the following will decrease? (Use per unit mass analysis)
Heat rejected
COP
Compressor power
Specific volume at suction
ANSWER B
By superheating the refrigerant in vapor compression cycle without useful cooling,
which of the following will increase? (Use per unit mass analysis)
Refrigerant effect
COP
Compressor power
Mass flow rate
ANSWER C
By superheating the refrigerant in vapor compression cycle without useful cooling,
which of the following will not be affected? (Use per unit mass analysis)
Refrigerant effect
COP
Compressor power
Mass flow rate
ANSWER C
By sub-cooling the refrigerant in vapor compression cycle at condenser exit, which of
the following will increase? (Use per unit mass analysis)
Refrigerating effect
Specific volume at suction
Compressor power
Mass flow rate
ANSWER A
By sub-cooling the refrigerant in vapor compression cycle at condenser exit, which of
the following will decrease (Use per unit mass analysis)

A. Coefficient of performance B. Heat rejected from condenser


C. Refrigerating effect D. Mass flow rate
ANSWER: D
By increasing the vaporizing temperature in vapor compression cycle, which the
following will increase? (Use per unit mass analysis)
A. mass flow rate B. COP C. specific volume D. compressor work
ANSWER: B
By increasing the vaporizing temperature in vapor compression cycle, which the
following will decrease? (Use per unit mass analysis)
A. Refrigerating effect B. COP C. evaporator temperature

D. temperature difference between evaporator and compressor


ANSWER: D
By increasing the condenser pressure in vapor compression cycle, which of the
following will increase? (Use per unit mass analysis)
A. Refrigerating effect B. COP C. Specific volume at suction
D. Compressor power
ANSWER: D
By increasing the condenser pressure in vapor compression cycle, which of the
following will decrease? (Use per unit mass analysis)
A. moisture content after expansion B. compressor power
C. heat rejected in the condenser D. mass flow rate
ANSWER: A
If the pressure drop in the condenser increases in a vapor compression cycle, which of
the following will increase? (Use per unit mass analysis)
A. mass flow rate B. compressor power
C. heat rejected in the condenser D. specific volume at suction
ANSWER: C
If the pressure drop in the condenser increases in a vapor compression cycle, which of
the following will decrease? (Use per unit mass analysis)
A. Refrigerating effect B. mass flow rate C. heat rejected in the
condenser
D. Compressor power
ANSWER: B
If the pressure drop in the condenser increases in a vapor compression cycle, which of
the following will not be affected? (Use per unit mass analysis)
A. compressor power B. mass flow rate
C. heat rejected in the condenser D. COP
ANSWER: A
If the pressure drop in the evaporator increases in a vapor compression cycle, which of
the following will increase? (Use per unit mass analysis)
A. Refrigerating effect B. vaporizing temperature
C. heat rejected in the condenser D. COP
ANSWER: C
If the pressure drop in the evaporator increases in a vapor compression cycle, which of
the following will decrease? (Use per unit mass analysis)
A. specific volume at suction B. compressor power
C. heat rejected in the condenser D. COP
ANSWER: D
By lowering the condenser pressure in Rankine cycle, which of the following will
decrease? (Use per unit analysis)
A. Pump work B. turbine work C. heat rejected
D. cycle efficiency
ANSWER: C
By increasing the boiler pressure in Rankine cycle, which of the following will decrease?
(Use per unit analysis)
A. heat rejected B. pump work C. cycle efficiency D.
moisture content
ANSWER: A
By superheating the steam to a higher temperature in Rankine cycle, which of the
following will decrease? (Use per unit analysis)
A. moisture content at the turbine exhaust B. turbine work C.
heat added
D. heat rejected
ANSWER: A
By superheating the steam to a higher temperature in Rankine cycle, which of the
following will increase? (Use per unit analysis)
A. moisture content at the turbine exhaust B. pump work
C. condenser pressure D. cycle efficiency
ANSWER: D, B
By reheating the steam before entering the second stage in Rankine cycle, which of the
following will decrease?
A. Turbine work B. moisture content after expansion
C. heat added D. heat rejected
ANSWER: B
What Rankine cycle is modified with regeneration, which of the following will increase?
A. turbine work B. heat added C. heat rejected
D. cycle efficiency
ANSWER:
Is the combination of base load and peaking load?
A. rated load B. intermediate load C. combine load
D. over-all load
ANSWER: B
Sum of the maximum demand over the simultaneous maximum demand?
A. use factor B. capacity factor C. demand factor D. diversity factor
ANSWER: D
Regenerative with feed heating cycle with infinite number of feed water heaters thus
efficiency is equal to?
A. otto cycle B. stirling cycle C. erricson cycle D. carnot
cycle
ANSWER: D
A type of turbine used in desalination of sea water.
A. back pressure turbine B. passout turbine C. peaking turbine
D. reaction turbine
ANSWER: A
State that when conductor and magnetic field move relatively to each other, an electric
voltage is induced in the conductor.
A. Maxwells law B. Kirchhoffs law C. Faradays law
D. Newtons law
ANSWER: C
Transfers heat directly to electrical energy by utilizing thermionic emissions.
A. thermionic motor B. thermionic generator C. thermionic converter
D. thermionic cell
ANSWER: B
Is the largest group of coal containing 46-89% of fixed carbon and 20% to 40% volatile
matter.
A. anthracite B. sub-anthracite C. bituminous D. sub-
bituminous
ANSWER: C
When 1 gram of coal is subjected to a temperature of about 105C for a period of 1 hour,
the loss in weight of the sample gives the:
Volatile matter B. ash C. fixed carbon D. moisture content
ANSWER: D
When 1 gram of sample of coal is placed in a crucible and heated 950C and maintain at
the temperature for 7 minutes there is a loss in weight due to elimination of:
A. volatile matter and moisture B. ash C. fixed carbon
D. moisture content
ANSWER: A
Consist of hydrogen and certain hydrogen carbon compounds which can be removed
from coal by heating.
A. moisture content B. product of combustion C. ash D. volatile matter
ANSWER: D
By heating 1 gram of coal in an uncovered crucible until the coal is completely burned,
the ____ will formed.
A. volatile matter and moisture B. ash C. fixed carbon
D. moisture content
ANSWER: B
Caking coal are use to produce coke by heating in a coke oven in the absence of ____
with volatile matter driven off.
A. air B. oil C. oxygen D. nitrogen
ANSWER: A
Gindability of standard coal is:
A. 80 B. 90 C. 100 D. 110
ANSWER: C

Major constituent of all natural gases is:


A. ethane B. methane C. propane D. cethane
ANSWER: B
Two types of fans are:
A. centrifugal and axial B. reciprocating and axial C. centrifugal and
rotary
D. tangential and rotary
ANSWER: A
Enthalpy of substance at specified state due to chemical composition.
A. Enthalpy of reaction B. enthalpy of combustion C. enthalpy of
formation
D. enthalpy of product
ANSWER: C
A type of boiler used for super critical pressure operation.
A. La Mont boiler B. Once- through-circulation boiler
C. Force circulation boiler D. Natural circulation boiler
ANSWER: B
Economizer in a water tube boiler is heated by:
A. electric furnace B. electric current C. incoming flue gas
D. outgoing flue gas
ANSWER: D
Receives heat partly by convection and partly by radiation.
A. radiant superheater B. desuperheater C. convective superheater
D. pendant superheater
ANSWER: D
Regenerative superheater is a storage type of heat exchangers have an energy storage
medium called:
A. matrix B. regenerator C. boiler D. recuperator
ANSWER: A
Stirling cycle uses a ____ as working fluids.
A. incompressible gas B. incompressible fluids
C. compressible refrigerant D. compressible fluids
ANSWER: D
In stirling process the heat added is added during?
A. isobaric process B. isentropic process C. isothermal process

D. heat process
ANSWER: C
Brayton cycle is also known as:
A. carnot cycle B. joule cycle C. diesel cycle D.
rankine cycle
ANSWER: B
Is applied to propulsion of vehicles because of certain practical characteristics.
A. diesel cycle B. otto cycle C. carnot cycle D.
brayton cycle
ANSWER: D

Heat exchangers typically involve


A. no work interactions B. no heat interactions C. no energy
interactions
D. none of these
ANSWER: A
A device that is used to convert the heat to work is called
A. adiabatier B. regenerator C. heat engines D.
none of these
ANSWER: C
The objective of a heat pump is to maintain a heated space at
A. Low temperature B. high temperature C. medium temperature
D. none of these
ANSWER: B
A device that violates the second law of thermodynamics is called
A. perpetual motion machine of second kind B. perpetual motion machine of
third kind
C. perpetual motion machine of first kind D. none of these
ANSWER: A
A process is called ____ if no irreversibilitys occur outside the system boundaries
during the process.
A. externally reversible B. internally reversible C. reversible
D. none of these
ANSWER: A
An energy interaction which is not accompanied by entropy transfer is
A. energy B. heat C. work D. none of these
ANSWER: C
A ____ is used in aircraft engines and some automotive engine. In this method, a
turbine driven by the exhaust gases is used to provide power to compressor or blower
at the inlet.
A. discharging B. turbo charging C. supercharging D.
scavenging
ANSWER: B
The only devices where the changes in kinetic energy are significant are the
A. compressor B. pumps C. nozzles and diffusers D.
none of these
ANSWER: C
The distance between TDC and BDC in which the piston can travel is the
A. Right extreme position B. displacement stroke C. stroke of
the engine
D. swept stroke
ANSWER: C
In compression-engine, the combustion of air-fuel mixture is self-fuel ignited as a result
of compressing the mixture above its
A. self developed temperature B. mixing temperature
C. self feed temperature D. self ignition temperature
ANSWER: D

The thermal efficiency of an ideal Otto cycle depends ____ of the working fluid.
the pressure ratio of the engine and the specific heat ratio
the temperature ratio of the engine and the specific heat ratio
the moles ratio of the engine and the specific heat ratio
the compression ratio of the engine and the specific heat ratio
ANSWER: D
As the number of stages is increased, the expansion process becomes
A. isentropic B. isothermal C. isometric D. polytropic
ANSWER: B

Aircraft gas turbines operate at higher pressure ratio typically between


A. 6 to 8 B 12 to 24 C. 10 to 18 D. 10 to 25
ANSWER: D
The first commercial high-pass ratio engines has a bypass ratio of
A. 1 B. 3 C. 5 D. 7
ANSWER: C
The single-stage expansion process of an ideal Brayton cycle without regeneration is
replaced by a multistage expansion process with reheating the samepressure limits. As
a result of modification, thermal efficiency will:
A. increase B. decrease C. remain constant D. none of these
ANSWER: B
Which of the following is/are the application of Brayton cycle.
A. propulsion system B. automotive turbine engine
C. aircraft turbine engine D. all of these
ANSWER: D
It is used as working fluid in high-temperature applications of vapor cycles.
A. helium B. deuterium C. mercury D. water
ANSWER: C
The superheated vapor enters the turbine and expands isentropically and produces
work by the rotating shaft. The ____ may drop during the process.
A. density B. viscosity of fuel C. temperature and pressure
D. none of these
ANSWER: C
Only ____ of the turbine work output is required to operate the pump.
A. 0.01% B. 0.02% C. 0.03% D. 0.04%
ANSWER: D
Superheating the steam to higher temperatures decreases the moisture content of the
steam at the ____.
A. turbine inlet B. compressor inlet C. compressor exit
D. turbine exit
ANSWER: D

Regeneration also provides a convenient of dearating the feedwater to prevent?


A. boiler explosion B. boiler scale production C. boiler corrosion
D. compressor damage
ANSWER: C
Can be apply steam turbine cycle(rankine). Gas turbine cycle(brayton) and combined
cycle.
A. hydroelectric plant B. nuclear power plant
C. cogeneration plant D. tidal power plant
ANSWER: C
In a rankine cycle with fixed turbine inlet conditions. What is the effect of lowering the
condenser pressure, the heat rejected will:
A. increase B. decrease C. remains the same D. none of
these
ANSWER: B
In an ideal rankine cycle with fixed boiler and condenser pressure. What id the effect of
superheating the steam to a higher temperature, the pump work input will.
A. increase B. decrease C. remain the same D. none of these
ANSWER: C
The fact that total energy in any one energy system remains constant is called the
principle of ____?
A. conversion of energy B. second law of thermodynamics
C. conservation of mass D. zeroth law of thermodynamics
ANSWER: A
A process for which the inlet and outlet enthalphies are the same
A. isntropic B. enthalphy conservation C. throttling D. steady state
ANSWER: C

The sum of energies of all the molecules in system, energies that in several complex
forms.
A. kinetic energy B. internal energy C. external energy D. flow work
ANSWER: B
A system that is completely impervious to its surrounding. Neither mass nor energy
cross its boundaries.
A. Open system B. closed system C. adiabatic system D. isolated
system
ANSWER: D
A device used to measure small and moderate pressure difference.
A. manometer B. bourdon gage C. barometer D.
piezometer
ANSWER: A
A vapor having a temperature higher that the saturation temperature corresponding to
its pressure.
A. superheated pressure B. saturated vapor
C. super saturated vapor D. subcooled vapor
ANSWER: A
The energy or stored capacity for performing work possessed by a moving body, by
virtue of its momentum.
A. internal energy B. work C. gravitational potential energy
D. kinetic energy
ANSWER: D
A thermodynamic process wherein temperature is constant and the change in internal
energy is zero.
A. isobaric process B. isometric process C. isothermal process

D. polytropic process
The science terminology concerned with precisely measuring energy and enthalpy
A. thermodynamics B. chemistry C. calorimetry D. none of these
ANSWER: C
The rate of doing work per unit time
A. torque B. power C. force D. moment
ANSWER: B
A vapor having a temperature higher than the saturation temperature corresponding to
the existing pressure.
A. superheated vapor B. saturated vapor C. wet vapor D. none of the
above
ANSWER: A
It is the work done in pushing a fluid across a boundary, usually or out of a system.
A. mechanical work B. nonflow work C. flow work D. electrical work
ANSWER: C
A liquid that has a temperature lower that the saturation temperature corresponding to
the existing pressure.
A. subcooled liquid B. saturated liquid C. unsaturated liquid D. water
ANSWER: A
In this type of boiler, the water passes through the tubes while the flue gases burn
outside the tubes.
A. water-tube boiler B. fire-tube boiler C. steam generator D. electric boiler
ANSWER: A
It shows the water level in the boiler drum.
A. water column B. try cocks C. gauge glass D. all of the above
ANSWER: C
It prevents damage to the boiler by giving warning of low water.
A. safety valve B. fusible plug C. relief valve D. try cocks
ANSWER: B
it is heat exchanger which utilizes the heat of the flue gases to preheat air needed for
combustion.
A. economizer B. feedwater heater C. reheater D. air preheater
ANSWER: D
It is the subject that deals with the behavior of moist air.
A. psychrometer B. psychometry C. refrigeration D.
pneumatics
ANSWER: B
Air whose condition is such that any decreases in temperature will result in
condensation of water vapor into liquid.
A. saturated air B. unsaturated air C. saturated vapor D. moist air
ANSWER: A
It is the warm water temperature minus the cold water temperature leaving the cooling
tower
A. approach B. terminal difference C. cooling range D.
LMTD
ANSWER: C
The surrounding air ____ temperature is the lowest temperature to which water could
possibly cooled in a cooling tower.
A. dry-bulb B. wet-bulb C. dew-point D. saturated temperature
ANSWER: B

PRE ELEMENTS

Which of the following compressors are so widely used for todays refrigeration system?
Ans. Centrifugal
A valve sometimes known as the magnetic valve.
Ans. Solenoid valve
Which of the following valve use to regulate the flow of refrigerant to the evaporator?
Ans. Expansion valve
Which of the following use control the flow of refrigerant gas from the evaporator coil.
This type of control valve is also known as back pressure regulator of an evaporator.
Ans. Suction line regulator
Which of the following refrigerant control is used to limit the flow of gas to the
compressor to prevent surge of excessive load from overloading the compressor.
Ans. Hold-back valve
Which of the following refrigerant control is used in the liquid suction or discharge to
interrupt the flow on demand from any one of the several types of temperature or
pressure sensing devices.
Ans. Solenoid valve
What is the pressure drop from the receiver to the expansion valve?
Ans. 5 psi
If any of the electricity controlled device in a Freon system malfunction, which following
valves will also automatically shut off?
Ans. Solenoid valve
The relief valve on a CO2 machine is located:
Ans. On the discharge pipe between the compressor and the discharge valve.
When checking zinc plates in a condenser, one should:
Ans. Clean the plate and renew worn-out ones.
A refrigerant should have a
Ans. High latent heat
The mechanical energy of a device is the ratio of:
Ans. Mechanical energy input to the mechanical energy output
When the winding or circuit is open, what is the resistance reading?
Ans. Infinity
If the meter scale of the ohmmeter reads 0 ohms and the range adjustment is R 10,
what is the resistance reading?
Ans. 100
What is the resistance reading of an electric motor in good condition?
Ans. With resistance
The basic unit of electrical pressure is:
Ans. Volt
What is the most commonly used conductor?
Ans. Copper
Which of the following is the effect of superheating the refrigerant?
Ans. Increase is COP
Which of the following statement is true in ice making capacity?
Ans. Ice making capacity is always proportional to the refrigerating effect
Which of the following has the largest heat load in cold storage room?
Ans. Product heat load
Which of the following is a halocarbon refrigerant?
Ans. Methyl chloride
A refrigerant system in which pressure- imposing element is mechanically operated?
Ans. Compressor
Which of the following compressors have the compressing element and drive seals in a
single housing?
Ans. hermitically sealed compressor
Which of the following consists of two mating helically grooved male and female
grooves?
Ans. Helical rotary-screw
What is the another name of helical-rotary screw compressors?
Ans. Lysholm type
At what head is the helically-rotary compressor are designed to operate?
Ans. High head pressure
The bulk quantity of oil separated from the refrigerant in the helically-rotary compressor
falls by?
Ans. Gravity to a sump
If the temperature in the icebox is too high, the trouble could be:
Ans. Automatic control not functioning properly
If any of the electrically controlled devices in a Freon system malfunction, which of the
following valves also automatically shut-off?
Ans. Solenoid valve
Natural ice in an ice box was used in the early
Ans. 19th century
The amount of matter present in a quantity of any substance is called:
Ans. Mass
What pressure is indicated by a barometer?
Ans. Atmospheric pressure
The basic unit measurement for heat is:
Ans. BTU
The force that acts upon a unit of area is termed:
Ans. Pressure
What is the physical state of a refrigerant entering the flow control?
Ans. Sub-cooled
What method of heat transfer does not depend on molecular motion?
Ans. Radiation
Where s the filter drier is located?
Ans. After the condenser
A motor drive device which removes the heat-laden vapor refrigerant from the
evaporator is the:
Ans. Compressor
The function of the compressor is to:
Ans. Squeeze out refrigerant from the evaporator
In which part of the compressor is the piston attached to the crankshaft?
Ans. Connecting rod
This is a compressor which has an off center-rotor.
Ans. Rotary
Which of the following compressor is commonly used in household refrigerators?
Ans. Hermetic
What is the length of the tubing extended from the part to be swaged
Ans. inch
The best method of making leak proof connection is:
Ans. Silver brazing
The refrigerant that should not be used copper or brass tubing:
Ans. R-717
The color of the hose connected to suction line or process be
Ans. Blue
Where is the compound gauge installed in a refrigerant system?
Ans. Suction line
A gauge used to measure pressure both below and above atmospheric pressure
Ans. Compound gauge
What is the valve position if the valve stem is turned all the way in?
Ans. Front-seated
What is the valve position if the valve stem is turned all the way out?
Ans. Back-seated
Ohms law states that resistance is inversely proportional to:
Ans. Current
To prevent corrosion within the flue gas, what must be prevented from condensing?
Ans. Vapor
Ans. Evaporative cooling
# An adiabatic saturation process. This process can be produced with _____, which is
essentially
a regular thermometer with its bulb wrapped in wet cotton or gauze.....
Ans. Sling psychrometer
# A high velocity gas is defined as a gas moving with a velocity in excess off
approximately
Ans. 300ft/s/ 100m/s
# The theoretical maximum velocity is achieved when.....
Ans. All internal and pressure energies are converted to kinetic energies
# If the gas flow is adiabatic and frictionless (i.e. reversible)
Ans. Both A and B
# A property by which the refrigerant remains at its original chemical form/original
condition is imposed by the operation.....
Ans. Stability of refrigerant
# The maximum temperature at which a gas is condense into liquid. Above temperature,
a vapor
imposed by the operation.....
Ans. Stability of refrigerant
# Which of the following is the name given for halogenated hydrocarbon?
Ans. Halocarbon
# Which of the following refrigerants are produced by carrier corporation?
Ans. Carrene
# Which of the following refrigerants is produced by E.L.DU FONT?
Ans. Freon
#Which of the following is the other name of R 718?
Ans. Water
# Energy per unit volume stored in a deformed material is called:
Ans. Strain energy
# The ratio of ultimate failure strain to yielding strain is known as:
Ductility
# One ton of refrigeration is equal to:
Ans. 3.5KW
# With what cycle does air refrigeration works?
Ans. Bell-coleman cycle
# What cycle does air refrigeration works?
Ans. Coal
# What is produced by fission process?
Ans. Radiation
# Loss of power is due to:
Ans. Low injection pressure
# A branch system of pipes to carry waste emissions away from the combustion
chamber.
Ans. Exhaust manifold
# The type of filter where the filtering elements is replaceable.
Ans. Metal-edge filter
# When four events takes place in one revolution of a crankshaft of an engine, the
engine is called
Ans. Two stroke engine
# Which of the following does not belong to the group.
Ans. Time injection system
# The function of a super heater is to:
Ans. Exchange heat to increase energy to the flow during an adiabatic, internally
reversible process
# What is true about change in entropy?
Ans. it is always zero
# How does an adiabatic process compare to an isentropic process?
Ans. Both heat transfer = Q, isentropic reversible
# Adiabatic heat transfer within a vapour cycle refers to :
Ans. The transfer of energy from one stream to another in a heat exchanger that
the energy of
the input streams equal to the energy of output.
# All of the following process are irreversible except:
Ans. An isentropic compression of a perfect fluid
# Refers to the system where the evaporator coils are placed in the ice box
Ans. Direct system
# What is the usual dehydrating agent in Freon system?
Ans. Activated alumina
# Refers to the system where the evaporator coils are indicated in a brine solution and
brine is pumped thru the ice box
Ans. Indirect system
# Is the most common dryer which consist of rotating cylinder inside which the materials
flow
while getting in contact with hot gas.
Ans. Rotary dryer
# Is the ratio of the mass of water-vapor in air and mass of air if it is saturated is called:
Ans. Relative humidity
# The hands feel painfully cold when the skin temperature reaches
Ans. 10C
# The total heat of air is a function of
Ans. WB temperature
# Boiling point of Freon-12 at atmospheric pressure is:
Ans. 21F
# Which of the following is NOT a type of water cooled condenser in refrigeration?
Ans. Double shell
# Component of absorption refrigeration system in which the solution is cooled by
cooling water.
Ans. Absorber
# Cascade refrigeration cycle is often used in industrial process where objects must be
cooled to temperature below:
Ans. -46C
# Type of refrigerant control designed to maintain a pressure difference while the
compressor is operating.
Ans. Capilliary tube
# As a rule of thumb, for a specified amount of compressed air, the power consumption
of the compressor decreased by ____________for each 3C drop in the temperature
inlet air to the compressor.
Ans. 1 percent
# Modern way of detecting air compressor leak is by using
Ans. Acoustic leak detector
# For foundation of stacks, the maximum pressure on the soil is equal to the pressure
due to the
weight and the _______.
Ans. Wind movement
# Foundation bolts of specified size should be used and surrounded by a pipe sleeve
with an inside
diameter of at least
Ans. 3 times the diameter of anchor bolt
# For multi stage compression of an ideal Brayton cycle, the back work ratio will
Ans. decrease
# Type of turbine that has a specific speed below 5.
Ans. Impulse turbine
# A high discharge type of turbine
Ans. Propeller turbine
# Use to minimize the speed rise due to a sudden load rejection
Ans. jet deflector
# Is the speed of a turbine when the head on the turbine is one meter.
Ans. Unit speed
# Is a fluid property which refers to the intermolecular attraction by which the separate
particles of the fluid arc held together.
Ans. Cohesion
# Which of the following is NOT the cause of black smoke in diesel engine?
Ans. high compression pressure
# Which of the following is not a method of starting a diesel engine?
Ans. Using another generator
# Two-stroke engine performs _____ to complete one cycle.
Ans. compression and power stroke
# A type of geothermal plant used when there is a presence of brine extracted from
undergroungd
Ans. Binary geothermal plant
# Is the most important safety device on the power boiler.
Ans. Safety valve
# During hydrostatic test, the safety valves should be
Ans. removed
# Where deaerating heaters are not employed, it is recommended that the temperature
of the feed water be not less than _______
Ans. 197C
# What is the possible cause of too much suction pressure?
Ans. Shortage refrigerant
Where is the oil separator located?
Ans. Between the compressor and condenser
# What is the purpose of the low pressure cutout switch?
Ans. To cut compressor in and out at present pressure
# Freon unit will tend to short cycle when operating under
Ans. Light loads
# What cause hot suction line?
Ans. Insufficient refrigerant
# When changing a Freon system which valve is not used?
Ans. King valve
# Where is the scale trap located?
Ans. Between the king valve and expansion valve
# Where is the solenoid valve located?
Ans. Between the scale trap and thermal expansion valve
# Water tube boilers have how many fusible plugs?
Ans. 4
# What is the main cause of air pollution as a result of burning fuel?
Ans. Nitrogen dioxide
# Refers to the increase of enthalpy of a substance when it undergoes some phase
change at constant pressure and temperature.
Ans. Heat of vaporization
# Which of the following keeps moisture from passing thru the system?
Ans. Dehydrator
# What is the lowest temperature to which water could possibly be cooled in a cooling
tower?
Ans. Temperature of adiabatic saturation
#What do you call the intake pipe to a hydraulic turbine from a dam?
Ans. Penstock
# An ideal fluid is one that
Ans. is frictionless and incompressible
# What principal was used by McLeod gauge which is used for low pressure
measurement/
Ans. Boyles Law
#The difference between the brake and hydraulic power is
Ans. Friction power
# Product of motor pump efficiency is known as
Ans. Over-all efficiency
# The most efficient cross-section of an open channel is the one that
Ans. All of the above
#Refers to the structure across an open channel over which water flows.
Ans. Weir
# When can we say that a trapezoidal weir is a cipoletti weir?
Ans. When side slope 4 vertical and 1 horizontal
# A weir is broad-crested if:
Ans. The weir length is greater than half of the head
# A feature of a dam over where water is discharged is called:
Ans. Spillway
# Laminar flow occurs when the Reynolds number is approximately less than
Ans. 2100
# Turbulent flow occurs when the Reynolds number exceeds:
Ans. 4000
# Laminar flow is typical when
Ans. All of the above
# When the Reynolds number is between 2100 to 4000, the flow is said to be in
Ans. Critical zone
# Which of the following characterizes a laminar flow?
Ans. All of the above
# Without a pump and turbine, which of the following rules is true in a frictionless
environment about energy and hydraulic gradient?
Ans. All of the above
# What is the latent heat of vaporization of water?
Ans. 970 Btu
# A dry bulb temperature at which water starts to condense but when moist appears in a
constant pressure process
Ans. Dew point temperature
# A mixture of dry air and saturated vapour is known as:
Ans. Saturated air
# What is another term for humidity ratio?
Ans. Specific humidity
# The water vapour mass is often reported in
Ans. Grains of water
# Approximately how many grains of water are there in 1 pound?
Ans. 7000
# The ratio of actual humidity ratio to the saturated humidity ratio at the inlet
temperature and p
Ressure is known as
Ans. Degree of saturation
# What is another term for degree of saturation?
Ans. Percentage of humidity
# The partial pressure of the water vapour divided by the saturation pressure?
Ans. Relative humidity
# The point where sonic velocity has been achieved (M=1) is known as:
Ans. Critical point
# The ratio of the thrust in equal mass flow rate is known as...
Ans. Effective exhaust velocity
# Is the velocity of gas dropped from supersonic to subsonic, gas will experience
Ans. Shock wave.
# An adiabatic flow with friction, which would be approximately as flow through an
insulated duct...
Ans. Fan no flow
# What is an inevitable by product of the combustion of hydro carbon fuel?
Ans. Water vapour
# A compound pressure gauge is used to measure:
Ans. Positive and negative pressure
# Which of the following best describes a Curtis turbine?
Ans. It is a velocity pressure compound turbine
# What causes low head pressure?
Ans. Too much cooling water and in sufficient refrigeration gas
# What causes high head pressure?
Ans. All of the above
# A commercial pipe which has the characteristics of being immune to electrolysis and
corrosion, light in weight weak structurally.
Ans. Asbestos cement pipe
# A commercial pipe which is durable, water tight, low maintenance and smooth
interior.
Ans. Concrete pipe
# A commercial pipe which has the characteristics of being corrosion, and scour and
erosion
Resistance.
Ans. Vitrified clay pipe
# A commercial pipe which that is ductile, high strength, shock resistant, very smooth
internally.
Ans. Steel pipe
a
# A commercial pipe that is chemically insert, resistant to corrosion, very smooth, light
weigth and very cheap
Ans. Plastic (PVC and ABS)
# Which commercial pipe that is primarily used for water condensate, and refrigerant
lines easier to bent by hand, good thermal conductivity
Ans. Brass and copper pipe
# A fluid pathway that expose parts of the fluid to the atmosphere is the:
Ans. Open channel
# The ratio of the area in flow to the width of the channel at fluid surface in an open
channel is
known as:
Ans. Hydraulic depth
# The most efficient cross-section for an open channel is known as:
Ans. Semi-circular section
# The most efficient section rectangular section has:
Ans. A width twice the depth
# What is the maximum power that the motor can provide
Ans. Rated horsepower
# Which of the following is a possible cause of cavitation
Ans. All of the above
# Is a reaction during which chemical energy is released in the form of heat
# When forebay is not part of the generating plant design, it will be desirable to produce
a ________ in order to relieve the effect of rapid changes in flow rate.
Ans. Surge chamber
#What is used to keep the turbine in a hydroelectric generating plant up to 15 feet
above the tail
water surface?
Ans. Draft tube
# If a draft tube is not employed, water may be return to the tail water by way of a
chamber known as the:
Ans. Tail race
# An impulse turbine consists of a rotating shaft on which buckets of blades are
mounted. What is
the term used for rotating shaft?
Ans. Turbine runner
# Which of the following refrigerant that belong to group B refrigerants and harmful or
toxic refrigerants?
Ans. All of the above
# Which of the following refrigerants are combustible?
Ans. All of the above
# Which of the following refrigerants is used for frozen foods and ice cream display
cases
Warehouses and food freezing plants, medium temperature display cases truck
refrigeration and heat pumps?
Ans. R-502
# The refrigerant that was formerly the most widely used for air conditioning and
refrigeration. It was the principal refrigerant for automotive air conditioning.
Ans. R-12
# How do we know that there is shortage of refrigerant considering that we are in the
liquid line?
Ans. Vapor bubbles in the sight glass appear
# The amount of power actually entering a fluid is known as:
Ans. All of the above
#The input power to the pump will be:
Ans. Equal to the output power of the electric motor driving the pump
# A bent or curved carries fluid from a container at a high elevation to another container
at a lower elevation.
Ans. Siphon
# A water path, usually a large diameter pipe used to channel water around or through
an obstruction feature.
Ans. Culvert
# Which of the following principles governs the distributions of flow between the two
branches in a pipe system.
Ans. All of the above
# A method used to determine the network flow in a multi-hoop piping system.
Ans. Hardy-cross method
# A device used to measure velocity by determining the cooling effect of fluid flowing
over an electrically heated tungsten.
Ans. Hot-wire anemometer
# Which of the following is used for R-12?
Ans. All of the above
# Which of the following tubing (pipes) usually used in ammonia refrigerant?
Ans. Ferrous pipe
# Which of the following tubing (pipes) usually used in halo-carbon refrigerant?
Ans. K and L copper tubing
# Properties of superheated vapour can be found in?
Ans. Superheated table
# Properties of non-reacting gas mixture are given by:
Ans. Volumetric weight for molecular weight and density and geometric
weighing for all the
properties except entropy.
# The relation between the total volume of a mixture of a non-reacting gases for initial
volume is given by:
Ans. Amagats law
# Which of the following is the first definition of enthalpy?
Ans. The amount of useful energy in a system.
# A consists press thermodynamics process obeys:
Ans. Charles law
# The volume of an ideal is halved, while its temperature is double, what happens to
the pressure?
Ans. Pressure is multiplied by 4
# A liquid boils when its pressure equal.
Ans. Ambient pressure
# A system composed of ice and water at zero C is said to be:
Ans. All of the above
# A heat of fusion for a pure substance is ________.
Ans. The energy required to melt the substance.
#The heat of vaporization involves the change of enthalpy due to:
Ans. The change in phase from liquid to gas.
# The heat of sublimation involves the change of enthalpy due to:
Ans. The change in phase from solid to gas.
# The unit for absolute viscosity which is dyne-sec/cm3 is also known as:
Ans. Poise
# What is the unit of absolute viscosity?
Ans. Pa-second
# The ratio of absolute viscosity to mass density is also known as:
Ans. Kinetic viscosity
# What is the kinetic viscosity?
Ans. m2/s
# Stroke is a unit of kinetic viscosity which is equivalent to:
Ans. cm2/s
# What are volatile liquids?
Ans. The liquid that vaporizes immediately
# When does boiling occur?
Ans. When the liquid temperature is increased to the point that the vapour
present is equal to
the local ambient pressure.
# The liquid boiling pressure is dependent on:
Ans. Both A and B
# Referred as the mass flow the rate divided by the generator output in kilowatts
Ans. Water rate of stream rate
# The power after the auxiliary loads has been removed is known as:
Ans. Net electrical output
# _________ of a turbine is the ratio of actual to ideal energy extraction?
Ans. Both A and B
# What is the approximate maximum practical metallurgical limit on superheat?
Ans. 1150C and 625C
# What is used to increased the mean effective temperature at which heat is added
without producing significant expansion in the liquid-vapor region.
Ans. Superheat
# If some of the heat energy from these waste product is recovered and used for space
heating or cooling the process is called
Ans. Cogeneration
# In cogeneration, the recovered heat
Ans. Is used as heat
# The ratio of useful energy to the energy input is called?
Ans. Fuel utilization
# The ratio of the energy of a turbine to the recovered heat is known as:
Ans. Power to heat ratio
# If the recovered heat is used to vaporized water in a vapour cycle this is called:
Ans. Combined cycle
# What is the best coolant for high temperature gas reactor?
Ans. Helium (He)
# What is the liquid metal most frequently used as coolant, in liquid metal reactor?
Ans. Sodium (Na)
# Combustion power cycles differ from vapour power in that:
Ans. The combustion product cannot be returned into their initial condition for
reuse.
# A closed system using a fixed amount of ideal air as the working fluid?
Ans. Air standard cycle
# Which of the following is not a specific compound but is a mixture of octane and
lighter hydrocarbon?
Ans. Gasoline
# A series of process that eventually brings the system back to its original condition is
known as?
Ans. Cycle
# The _____ of a power cycle is defined as the ratio of usual work output to the supplied
input energy?
Ans. Thermal efficiency
# Is use to heat up the solution partially before entering the generator in absorption
refrigeration system.
Ans. regenerator
# The COP of actual absorption refrigeration system is usually
Ans. less than 1
# Sight glass is often located at:
Ans. liquid line
# Use to detects a vibration in current caused by the ionization of decomposed
refrigerant between two opposite-charged platinum electrodes.
Ans. Electronic detector
# The ability of oil to mix with refrigerants
Ans. miscibility
# For the submerged plane surface, the point on the surface where the resultant force
acts is called the
Ans. center of pressure
# At any point in fluid at rest, the pressure is the same in all directions. This principle is
known as:
Ans. Pascals Law
# The hot-wire manometer is used to measure
Ans. gas velocities
# The pitot static tube measures
Ans. the static pressure
# The terminal velocity of a small sphere setting in a viscous fluid varies as the
Ans. inverse of fluid viscosity
# Pressure drag results from
Ans. skin friction
# The pressure coefficient is the ratio of pressure forces to:
Ans. inertia forces
# Tranquil flow must always occur
Ans. above the critical depth
# Which of the following head loss coefficient among the following types of entrance?
Ans. re-entrant
# What waste water treatment method involves of algae from stabilization pond
effluents?
Ans. microscreening
# The number of nozzles will depend on the quantity of steam required by the turbine. If
the nozzles occupy the entire arc of the ring, the turbine is said to have:
Ans. full peripheral admission
# Tandem compound units may also have two low-pressure casting that produces:
Ans. triple flow
# By inter-cooling using two stage compressor of Brayton cycle, the backwork ratio will:
Ans. decrease
# On dynamic similitude, the relation which represents the ratio of inertia force to
pressure force is:
Ans. Euler number
# What is the maximum velocity in a sewer flowing full?
Ans. 0.6 m/sec
# A temporary structures constructed to exclude water from the site of the foundation
during its excavation and construction is called:
Ans. coffer dam
# Which is not a physical characteristics of water?
Ans. hardness
# Which dam is best for weak foundation?
Ans. buttress
# What is the volume of water which will drain freely from the aquifer?
Ans. specific yield
# What is the line defined by the water level in a group of artesian wells?
Ans. peizometric surface
# Select the one that is a positive indication of pollution of a river.
Ans. chloride content
# Which instruments is used to measure humidity of the atmosphere continuously?
Ans. hydrograph
# Entrance losses between tank and pipe or losses through elbows, fittings and valves
are generally expresses as a function of:
Ans. kinetic energy
# The air that contains no water vapour is called:
Ans. dry air

# What effectively states that it is impossible to built a cylindrical engine that will have a
thermal efficiency of 100%?
Ans. Kelvin-Planc statement of second law
# Refers to the maximum possible work that can be obtained from a cycle.
Ans. Availability
# The difference between the maximum and the actual work output is known as?
Ans. Process inversibility
# The study of the property of atmospheric air?
Ans. Psychrometry
# The temperature of the air that has gone through an adiabatic saturation process is
known as?
Ans. Wet-bulb temperature
# How often should compressor oil be changed?
Ans. At least semi-annually
# Which of the following is the possible effect of the weak solution of brine in a
refrigeration system?
Ans. Corrosion
# In Freon refrigeration system, where is the scale trap located?
Ans. On the suction side of the compressor
# At what pressure is the high pressure cut-off in the Freon (R-12)
Ans. 125-150 psi
# What is the other name for brine refrigeration system?
Ans. Indirect refrigeration system
# What is the indication that there is air in the refrigeration system?
Ans. Unusual high head pressure
# A Freon-12 leaking can be detected by halide torch. What color will it turn to in the
presence of a refrigerant if this torch has a normal blue flame?
Ans. Green
# All of the following process are irreversible except
Ans. Inelastic tension and release of steel bar
# Which of the following is a point function except
Ans. Work
# All heat transfer process require medium of energy exchange except
Ans. Radiation
# Thermal conduction is described by:
Ans. Fourier Law
# Convection is described by:
Ans. Newtons Law
# Radiant heat transfer is described by:
Ans. The Stefan-Boltzman Law
# The equivalence of ratios of emissive power to absorptivity equilibrium is described by
Ans. Kirchoffs Law
# The temperature potential between temperature difference.
Ans. The logarithmic temperature difference
# A thermodynamic process whose deviation from equilibrium is infinitesimal at all
times.
Ans. In quasi-equilibrium
# A thermodynamic property best describes the molecular activity of a substance?
Ans. Internal energy
# The combination of conditions that best describes thermodynamic process is given:
Ans. Has successive states thru which the system passes
# A substance whose properties are uniform throughout is called a:
Ans. Pure substance
# All of the following are thermodynamic properties except:
Ans. Modulus
# A process that is adiabatic and reversible is also called:
Ans. Isentropic
# The first law of thermodynamics for a closed system is Q= U + W, the sign
convention is:
Ans. Q positive in, W positive out and U negative for decreased internal energy
# The heat transfer term in the first law of thermodynamics may be due to any of the ff.
except:
Ans. Internal heat generation
# A system that experiences no mass crossing the system boundaries called:
Ans. Closed system
# A substance in which a substance is allowed to enter and have is most properly
called:
Ans. Open system
# First and second law of thermodynamics are:
Ans. Energy equations
# A constant temperature thermodynamic process obeys:
Ans. Boyles Law
# A series of process that eventually bring the system back to the original condition is
called a:
Ans. Cycle
# If the refrigerant leaves the evaporator with a quality of < 1, the cycle is known as:
Ans. Wet vapor compression cycle
# For even lower temperature, are required as in commercial freezing equip. What is
used as refrigerant?
Ans. Ammonia
# What is another name for Air-Refrigeration cycle?
Ans. Brayton cycle
# What is the major disadvantage of Air-Refrigeration cycle?
Ans. High power consumption
# What refrigeration is practically used when a large quantity of waste-inexpensive heat
is avail?
Ans. Heat-driven refrigeration cycle
# For an absorption cycle, how many working fluids are required?
Ans. Two (2)
# In absorption cycle, what is needed in NH3 water system to remove any remaining
traces of absorbents from the refrigerant and is placed between generator and
condenser.
Ans. Rectifier
# Which is a TEMA stands for?
Ans. All of the above
# The acronym TEMA stands for?
Ans. Tubular Exchangers Manufacturing Association
# The acronym API stands for?
Ans. American Petroleum Institute
# What occurs when the exit temperature of the cold fluid is above the exit temperature
of the hot fluid?
Ans. Temperature cross
# Are heat exchangers whose purpose is to heat with condensing stream.
Ans. Closed feedwater heater
# Refers to the corrosion, precipitation of compound in solution, setting of particular
solid and biological activity that adhere to a heat transfer
Ans. Fouling
# Fouling in a heat exchanger industries is commonly known as:
Ans. Silent thief
# An operation with 1 fluid flow rate substantially known as:
Ans. Down turn
# Indicate the false statement?
Ans. Duo to viscosity, liquid cannot resist instantaneously change velocity
# Ideal liquid are assumed to be?
Ans. Newtonian fluids
# Which of the following is a Newtonian fluids?
Ans. All of the above
# Thermal radiation heating body can be absorb, reflected or transmitted, this is known
as
Ans. Radiation Conservation law
# The rate of thermal emitted per unit area of a body is known as:
Ans. Emissive power
# States that for any two bodies in thermal equilibrium, the ratio of emissive power to
absorptivity are
Equal.
Ans. Kirchoffs Radiation law
# Bodies that radiate at absorptivity of 1 are known as:
Ans. Black bodies or ideal radiators
# Which of the following does not radiate at the ideal level?
Ans. Real body
# The ratio of actual to ideal emissive power is known as:
Ans. Emissivity
# What is the refrigerators main function?
Ans. To cool this low temperature area
# The rate of energy removal from the low temperature area is known as:
Ans. Either of the refrigeration capacity or refrigerating effect
# The refrigeration capacity is measured in refrigeration ton where one ton is _____
heat remove
Ans. All of the above
# The refrigeration ton is derived from the heat flow required to melt the ton of ICE in:
Ans. 24 hours
# In psychrometric chart, the constant- enthalpy lines coincide with constant-
temperature lines at temperature
Ans. below 50C
# The amount of moisture in air depends on its
Ans. temperature
# The deep body temperature of healthy person is maintained constant at
Ans. 37C
# Air motion also plays important role in
Ans. human comfort
# During simple heating and cooling process has a_____ humidity ratio.
Ans. constant
# The ________ follows a line of constant wet-bulb temperature on the psychrometric
chart.
Ans. evaporative cooling process
# A vapor which is not about to condense is called a
Ans. superheated vapor
# Passing from the solid phase directly into vapor phase is called
Ans. sublimation
# Robert Boyle observed during his experiments with a vacuum chamber that the
pressure of gases is inversely proportional to their
Ans. volume
# _____ is energy in transition.
Ans. Heat
# Is the mode of energy transfer between a solid surface and the adjacent liquid or gas
which is in motion, and it involves combine effects of conduction and fluid motion.
Ans. convection
# Radiation is usually considered as
Ans. surface phenomenon
# Work is _____ between the system and the surroundings.
Ans. energy
# Is a process during which the system remains in equilibrium at all times
Ans. Quasi-equilibrium
# In the absence of any work interactions between a system and its surrounding, the
amount of net heat transfer is equal
Ans. to the change in the total energy of a closed system
# The constant volume and constant pressure specific heats are identical for
Ans. incompressible substance
# The velocity of fluid is zero at the wall and maximum at the center because of the
Ans. viscous effect
# For steady flow devices, the volume of the control volume is
Ans. constant
# The work done in a turbine is _____ since it is done by the fluid.
Ans. positive
# Reheating process in Brayton cycle, the turbine work will
Ans. increase
# As the air passes through a nozzle, which of the following will increase?
Ans. mach number
# As the air passes through a diffuser, which of the following will decrease?
Ans. mach number
# As the air passes through a nozzle, which of the following will decrease?
Ans. internal energy
# As the air passes through a diffuser, which of the following will increase?
Ans. density
# As the air passes through a diffuser, which of the following will NOT be affected?
Ans. entropy
# After passing through a convergent-divergent nozzle, the temperature of air will:
Ans. decrease
# After passing through a convergent-divergent nozzle, the density of air will:
Ans. decrease
# After passing through a convergent-divergent nozzle, the mach number of air will:
Ans. decrease
# By increasing the temperature source of Carnot cycle, which of the following will not
be affected?
Ans. heat rejected
# By decreasing the temperature sink of Carnot cycle, which of the following will not be
affected?
Ans. heat added
Ans. Exothermic reaction
# By reheating the steam in an ideal Rankine cycle the heat rejected will:
Ans. increase
# By increasing the boiler pressure in Rankine cycle the moisture content at boiler exit
will:
Ans. increase
# Presently the highest steam temperature allowed at the turbine inlet is about ______.
Ans. 620C
# Two most common gases employed in Stirling and Ericson cycles are:
Ans. Hydrogen and helium
# In most common designs of Gas turbine, the pressure ratio ranges from:
Ans. 11 to 16
# In Brayton cycle, the heat is transformed during what process?
Ans. isobaric process
# The fuel injection process in diesel engine starts when the piston_______.
Ans. approaches TDC
# If the cut-off ratio of diesel cycle increases, the cycle efficiency will:
Ans. decrease
# The fuel used in a power plant that is used during peak periods.
Ans. liquid
# Typical compression ratio of Otto cycle is
Ans. 8
# If Joule Thompson coefficient is equal to zero, then the process will become
Ans. isothermal
# If the fluid passed through a nozzle its entropy will:
Ans. remains the same
# Which of the following is NOT a possible location of service valve?
Ans. outlet of condenser
# A coil in series with evaporator that is use to prevent the liquid refrigerant entering the
compressor.
Ans. Drier loop
# A type of valve connected from discharge of compressor directly to suction that is
normally closed
And will open automatically only if there is high discharge pressure.
Ans. solenoid valve
# Use to increase the capacity of condenser.
Ans. desuperheating coils
# Is use to subcooled the refrigerant from the condenser.
Ans. liquid subcooler
# Which of the following is NOT a part of low pressure side in refrigeration system?
Ans. liquid line
# Which of the following is NOT a part of high pressure side in refrigeration system?
Ans. suction line
# Which of the following is NOT a part of condensing unit?
Ans. liquid line
# By subcooling the refrigerant in refrigeration system, the compressor power per unit
mass will
Ans. remains the same
# Superheating the refrigerant in refrigeration system without useful cooling, the
refrigeration effect
Per unit mass will
Ans. remains the same
# By subcooling the refrigerant in refrigeration system, the specific volume at
compressor suction will
Ans. remains the same
# Pressure loss due to friction at the condenser, the compressor power per unit mass
will
Ans. remains the same
# Which of the following is NOT a type of air-cooled condenser?
Ans. shell and tube
# A type of refrigerant control typically used in household refrigeration.
Ans. Capilliary tube
# Type of condenser that operates like a cooling tower.
Ans. evaporative condenser
# The major problem of heat pump is
Ans. frosting
# Dominant refrigerant used in commercial refrigeration system
# The Carnot refrigeration cycle is:
Ans. All of the above
# The vapour compression cycle is essentially a reverse of:
Ans. Rankine vapor cycle
# What can be considered to be theoretically exact and can be used to determine local
thermal film coefficient at a distance X from the leading edge of an isothermal plate in
laminar flow?
Ans. Pohlhausen Solution
# Which one is exact and can be derived from the Pohlhausen correction by setting
cube root of
Prandtl number equal to 108 and is useful in gases and water?
Ans. Blausius Solution
# What another term for skin friction coefficient?
Ans. Fanning friction factor
# The Darcy friction factor is ________.
Ans. 4 times
# The ______ for submerged bodies in the sum of the skin friction coefficient is?
Ans. Drag coefficient
# What can predict the average film coefficient along the entire length of laminar flow?
Ans. Labarsky-Kauffman Correlation
# When can we say that the laminar flow is Fully Developed?
Ans. When the difference between the surface (wall) and the mean fluid
temperature is
Constant
# Where is the accumulator usually located?
Ans. Before the suction line
# A device used to remove moisture and foreign particles inside the refrigeration
system.
Ans. Filter drier
# What is the standard value of the atmospheric pressure at the atmospheric sea level?
Ans. 14.7 psi
# Requirements are classified according to:
Ans. Manner of absorbing heat
# A push or pull on any thing is called:
Ans. Force
# Anything that takes up space has mass
Ans. Matter
# Factor use in calculating the overall heat transfer through the tube walls of the
condenser tube of
Another heat transfer surface.
Ans. Fouling factor
# What could be the possible cause of the abnormal discharge temperature?
Ans. All of the above
# Where is the relief valve located?
Ans. At the discharge side of the compressor
# What will you do before starting a refrigeration system?
Ans. Vent the condenser
# What is the purpose of the expansion valve bypass?
Ans. To control the refrigerant to the evaporator in case the automatic valve
fails.
# How many feed water lines are connected to the boiler?
Ans. 2
# Exhaust gases from the engine possesses what type of energy?
Ans. Kinetic energy
# What is the instrument used to measure density?
Ans. Hydrometer
# Engine overall efficiency is ________.
Ans. Brake thermal efficiency
# What is the ratio of the fuel usage rate to the power generated?
Ans. Specific fuel consumption
# What is the ratio of the air mass that enters the engine to each mass of fuel?
Ans. Air-fuel ratio
# The ohmmeter is an instrument used to measure
Ans. Resistance
# The ammeter is an electrical instrument used to measure
Ans. Current
# The electrical pressure that pushes the electrical current or objections.
Ans. Voltage
# The voltmeter is an instrument used to measure:
Ans. Voltage
# When the winding is shorted, what is the resistance reading?
Ans. Zero ohm
# Which tools are used to enlarge the end of the tubing to connect a fitting?
Ans. Flaring tools
# The tubing is never used when the fluid temperature goes beyond 100 to 175 mm?
Ans. Plastic
# The tubing to be swaged is damped in a
Ans. Flaring block
# The hand tools used to lightened and loosen the fitting are
Ans. Wrenches
# In soldering process, What substance is used to prevent oxidation?
Ans. Flux
# The process of applying molten metal to heated metals is called:
Ans. Soldering
# What is the physical state of the refrigerant?
Ans. Gas
# Which of the following appliances uses a finned-coil evaporator?
Ans. Window-type airconditioner
# The operation of condenser is the opposite of that of
Ans. Evaporator
# Which of the following devices maintains the pressure difference between the
evaporator and the condenser?
Ans. Flow control
# What device is ideal for measuring the flow of liquid metal?
Ans. Magnetic flow meter
# The frictional force that acts parallel but opposite to the direction of motion.
Ans. Drag
# A pump that transform kinetic energy into fluid static pressure.
Ans. Kinetic pump
# Due to friction losses between the fluid and the pump and mechanical losses in the
pump itself.
Ans. The brake pump power will be greater than hydraulic power
# The net energy actually transferred to fluid per unit time is:
Ans. Hydraulic power
# The input power delivered by motor to pump.
Ans. Brake pump power
# Boiler tubes are specified by
Ans. Outside diameter
# The maximum size of the boiler down lines valves and fittings shall not exceed
Ans. 2.5 inches
# The purpose of an expansion tank in a hot water heating system is to provide for the
expansion of
Ans. Water
# Heavy accumulation of a soot in the boiler will result in:
Ans. Loss of boiler efficiency
# How are tubes secured in a fired tube boiler will result in:
Ans. Rolled and beaded over
# A tube in a fire tube boiler is surrounded by:
Ans. Water
# What is the formula for the factor of evaporation?
Ans. H-b/970.3
# What can cause a super heater tube to overheat?
Ans. Dirty tube
# How many square feet of heating surface does a 3-in fire tube 20 ft long with a 3/16 in
. wall have?
Ans. 13.7
# In a high temperature hot water boiler; what could cause a tube to over heat?
Ans. Poor circulation
# A down corner is:
Ans. Found in a water tube boiler
# Extreme firebox temperature changes cause:
Ans. Spalling
# Low water cu. off:
Ans. Shuts the burner when the boiler is low on water
# Steam coming from the bottom by cock would indicate:
Ans. Low water level
# Why is the method to cool water called condenser not practical?
Ans. Water is expensive
# A device use to collect liquid refrigerant entering the compressor to prevent trouble in
compressor is
Ans. Accumulator
# A disiccant is a material that has high affinity to water. Which is the common used
dissicant
Ans. Silica gel
# The only means of preserving food in its original fresh state is _____________.
Ans. Refrigeration
# PVC means
Ans. Poly Vinyl Chloride
# What does ABS means?
Ans. Acrylonitrite Butadiene Styrene
# What measures the average roughness of imperfection inside the pipe?
Ans. Specific roughness
# Darcy factor, friction factor is not constant but decreases as the Reynolds number
increases up to a certain point known as:
Ans. Full turbulent flow
# What presents the friction factor graphically as 6 functions of Reynolds number and
relative roughness.
Ans. Moody diagram
# If the flow in truly laminar, and fluid flowing in a circular pipe then which of the
equation is appropriate to use?
Ans. Hagen Poisenille Equation
# Which is true about Hagen Williams equation?
Ans. It is primarily use for water
# For highly turbulent flow, what causes shear stress?
Ans. Momentum Effect
# In a circular pipe laminar flow momentum flux is:
Ans. Maximum at the pipe wall
# If the head pressure is too high
Ans. The high pressure cut-out switch should operate
# In a combustion engine cycle, what is the ratio of the net output power to the input?
Ans. Thermal efficiency
# In a reciprocating engines, what is the ratio of actual to ideal volumes of entering
gases?
Ans. Volumetric efficiency
# In a combustion engine, what is the ratio of the actual power developed to the ideal
power
developed?
Ans. Mechanical efficiency
# The equivalent of ratio of emissive power to absorptivity for both thermal equilibrium is
described
by:
Ans. Kirchoffs law
# The ideal cycle based on the concept that the combination process in both diesel and
gasoline in
the combination of heat transfer process that is constant pressure and constant volume.
Ans. Dual cycle
# The ratio of the volume at the end of heat addition to the volume at the start of heat
addition is called:
Ans. Cut-off ratio
# A theoretical body which when heated to incandescence would emit continuous light
ray spectrum.
Ans. Blackbody
# Which of the following is the reason for insulating the pipes?
Ans. Heat loss from the surface is minimize
# Heat transfer due to density differential
Ans. Convection
# A process involving a gas that cannot, even in principle be reserved with being done.
Ans. An isobaric compression
# The maximum amount of heat that can be converted into mechanical energy
Ans. Depends on the intake and exhaust temperature
# The maximum amount of mechanical energy converted into heat
Ans. 100%
# The work output of energy heat engine.
Ans. Equals the difference between its heat intake and heat exhaust
# To increase the output of a centrifugal pump, you must
Ans. Speed of rotation
# By superheating the refrigerant in vapor compression cycle with useful cooling, which
of the following will increase? (Use per unit mass analysis)
Ans. Heat rejected from condenser
# By superheating the refrigerant in vapor compression cycle with useful cooling, which
of the following will decrease? (Use per unit mass analysis)
Ans. Mass flow rate
# By superheating the refrigerant in vapor compression cycle with useful cooling, which
of the following will decrease? (Use per unit mass analysis)
Ans. COP
# By superheating the refrigerant in vapor compression cycle with useful cooling, which
of the following will increase? (Use per unit mass analysis)
Ans. Compressor power
# By superheating the refrigerant in vapor compression cycle with useful cooling, which
of the following will not be affected? (Use per unit mass analysis)
Ans. Compressor power
# By sub-cooling the refrigerant in 110apour compression cycle at condenser exit, which
of the following will increase? (Use per unit mass analysis)
Ans. Refrigerating effect
# By sub-cooling the refrigerant in 110apour compression cycle at condenser exit, which
of the following will decrease? (Use per unit mass analysis)
Ans. Mass flow rate
# By increasing the vaporizing temperature in vapor compression cycle, which of the
following will increase? (Use per unit mass analysis)
Ans. COP
# By increasing the vaporizing temperature in 110apour compression cycle, which of the
following will decrease? (Use per unit mass analysis)
Ans. temperature difference between evaporator and compressor
# By increasing the condenser pressure in 110apour compression cycle, which of the
following will increase? (Use per unit mass analysis)
Ans. Compressor power
# By increasing the vaporizing temperature in 111apour compression cycle, which of the
following will decrease? (Use per unit mass analysis)
Ans. moisture content after expansion
# If the pressure drop in the condenser increases in a vapor compression cycle, which
of the following will increase? (Use per unit mass analysis)
Ans. heat rejected in the condenser
# If the pressure drop in the condenser increases in a 111apour compression cycle,
which of the following will decrease? (Use per unit mass analysis)
Ans. mass flow rate
# If the pressure drop in the condenser increases in a 111apour compression cycle,
which of the following will not be affected? (Use per unit mass analysis)
Ans. compressor power
# If the pressure drop in the evaporator increases in a 111apour compression cycle,
which of the following will increase? (Use per unit mass analysis)
Ans. heat rejected in the condenser
# If the pressure drop in the condenser increases in a 111apour compression cycle,
which of the following will decrease? (Use per unit mass analysis)
Ans. COP
# By lowering the condenser pressure in Rankine cycle, which of the following will
decrease? (Use per unit mass analysis)
Ans. heat rejected
# By increasing the boiler pressure in Rankine cycle, which of the following will
decrease? (Use per unit mass analysis)
Ans. heat rejected
# By superheating the steam to a higher temperature in Rankine cycle, which of the
following will decrease? (Use per unit mass analysis)
Ans. moisture content at the turbine exhaust
# By superheating the steam to a higher temperature in Rankine cycle, which of the
following will increase? (Use per unit mass analysis)
Ans. cycle efficiency
# By superheating the steam to a higher temperature in Rankine cycle, which of the
following will decrease? (Use per unit mass analysis)
Ans. pump work
# What type of boiler incorporates furnace water cooling in the circulatory system?
Ans. Integral-furnace boiler
#The main components of a combined cycle power plant are:
Ans. Gas turbine and waste heat boiler
# Which of the following indicators is used to determine the anti-knocking characteristics
of gasoline?
Ans. Octane number
# Indicate the false statement
Ans. The heat transfer cannot exceed the work done.
# In fluid flow, linear momentum is:
Ans. A vector quantity equal to the product of mass and velocity
# A fact that a fluids velocity increases as the cross-sectional area of the pipe through
which it flow decreases due to:
Ans. The continuity equation
# The coefficient of contraction is the ratio of:
Ans. Area of vena contracta to the orifice area
# The coefficient of discharge is the ratio of:
Ans. Actual discharge to the theoretical discharge
# The coefficient of velocity is the ratio of:
Ans. Actual velocity divided the theoretical velocity
# Flow measuring devices include all of the following except:
Ans. Magnetic dynamometers
# In the series pipe systems, all of the following parameters vary from section to section
except
Ans. Mass flow
# The coefficient of velocity accounts for the:
Ans. Small effect of friction and turbulence of the orifice
# Expansion factors take into account the
Ans. Effects of compressibility
# The water hammer phenomenon is primarily what kind of fluid mechanics?
Ans. Dynamic (a time-dependent phenomena)
# All of the following are forms of drug on a body moving through a fluid except:
Ans. DAlemberts paradox drug
# The function of a turbine is to:
Ans. Extra energy from the flow
# The fact that there is no bodies moving through an ideal fluids is known as:
Ans. DAlemberts paradox
# Liquids and gases takes the following characteristics of their contents.
Ans. Shapes
# All of the following dimensionless parameters are applicable to fluid flow problem
excepts
Ans. Bolt number
# All of the following can be characteristics of fluids except:
Ans. Hysteresis
# The most common method for calculating frictional energy loss for laminar flowing
fluid is non- circular pipes is:
Ans. The Darcy equation
# For computation convenience, fluids are usually classed as
Ans. Real and ideal
# Which of the following is not a characteristics of real fluids?
Ans. Experience of eddy currents and turbulence
# Property of a fluid whereby its own molecules are attracted is known as:
Ans. Cohesion
# The term subsonic flow refers to a flowing gas with a speed.
Ans. Less than the local speed of sound
# The difference between stagnation pressure and total pressure
Ans. None of the terms are interchangeable
# The presence of friction in the hydraulic grade line will always cause the line to slope.
Ans. Down in the direction of the flow
#The presence of minor loss in the energy grade line will cause the line to slope
Ans. Down in the direction of the flow
# If the Mach number is greater than 1 but less than 5
Ans. Supersonic
# The flow is called sonic when mach number is
Ans. Equal to 1
# The flow is sub-sonic when mach number is
Ans. Less than 1
# To check water level in the gauge glass of a steam boiler
Ans. Use the
# The flame failure control is tested by?
Ans. Shutting off the fuel supply to the burner
# During purge cycle what is the minimum amount air volume changes required?
Ans. Four (4)
# The purge cycle time for gas compared to oil is:
Ans . The same
#A high (CO) Carbon monoxide reading, indicates:
Ans. None of these
# The induced draft fan is located:
Ans. In the breaching
# A balanced draft boiler required:
Ans. Both forced and induced draft
# What percent would expect in a well maintain boiler burning number 6 oil?
Ans. 15%
# Laminar friction factor of fluid is flowing through a pipe is a function of all the following
except:
Ans. Pipe roughness
# The stream function is a useful parameter in describing:
Ans. Conservation of mass
# The study of the practical law of fluid and the resistance of open pipes and channels is
the _____
Ans. Hydraulics
# The most common methods of calculating frictional energy loss for laminar flow fluids
in non circular pipes:
Ans. Equation
# The parameter in the expression for head loss is:
Ans. Darcy friction factor
# The characteristic length of Reynolds number use to calculate the friction in non-
circular full running pipes is based on the:
Ans. Hydraulic diameter
# The hydraulic radius of a non-circular pipe is:
Ans. The ratio of flow area to wetted parameter:
# An electromagnetic radiation wavelength is in the 0.1 to 10.000 mm range:
Ans. Thermal radiation
# What characteristics makes the difference between a Newtonian and non-newtonian
fluids?
Ans. Their viscous behaviour
# What instrument use to measure salt solution:
Ans. Salimeter
# A measure of fluid resistance to flow?
Ans. Viscosity
# What is another for absolute viscosity?
Ans. Dynamic
# What is the reciprocal of viscosity?
Ans. Fluidity
# The density of fresh water is a ship will float.
Ans. Lower in fresh water than in sea warer
# Bernoullis equation is based on?
Ans. Conservation of energy
# An express train goes past a station flatform at high speed a person standing in the
edge of the flatforms tend to be:
Ans. Attracted to the train
# The volume of fluid flowing per second out of an orifice at the bottom. A tank does not
depend on:
Ans. Density of fluid
# Addition of detergent to water?
Ans. Decreases its surface tension
# Water neither rise or falls in silver capillary. This suggest that the contact angle
between the water and silver is:
Ans. 90 degrees
# Most pressure in the Freon system have two dials or graduation on 1 gage what does
the two dial represent:
Ans. Temperature and pressure
# The flame detector is directed:
Ans. Into the furnace area
#Which of the following statement is true?
Ans. All of the above
#Which of the following is a pseudo plastic fluid?
Ans. All of the choices
# A type of fluids which are capable of indefinitely resisting a small shear stress but
move easily when the stress becomes large:
Ans. Bingham fluids
# Which of the following is an example of Bingham fluids?
Ans. All of the above
# The Bingham fluids will become pseudo plastic when:
Ans. The stress increases
# A type of fluids which exhibits viscosities that increases with an increasing velocity
gradient.
Ans. Dilatant fluid
# The viscosity decreases with time the fluids is said to be:
Ans. Thixotropic fluid
# Viscosity increases with time the fluids is said to be:
Ans. Rneopectic fluids
# What is the dominating cause of viscosity in fluids:
Ans. Molecular cohesion
# In a compressor, the piston is said to be at _____ when it is at its most petracted
position in the stroke,
Ans. Bottom dead center
# Reciprocating compressors are characterized by their
Ans. Clearance volume
# The gases remaining in the clearance volume after the discharge in the dead center
are known as the:
Ans. Residual gases
# The ratio of the clearance volume to the swept volume is known as:
Ans. Percent clearance or just clearance
# The work done in an adiabatic (isentropic, process is _____ the work done by the
isothermal process;
Ans. Less than
# The work done in an isentropic compressor is ____ the work done by the isothermal
compressor;
Ans. Greater than
# The term used to describe a partially compressed gases withdrawn cooled and
compressed further;
Ans. Intercooling
# A perfect intercooling refers to the case where the gases cooled to:
Ans. The original inlet temperature
# Multi stage blowers can reach pressure up to ____ of water;
Ans. 100 inches
# What represents the ratio of the coefficient of the friction at the expanded in pumping
a fluid;
Ans. Fried heat transfer efficiency factor
# What is the proper oil storage temp for No. 6 oil?
Ans. 20 F above pour point
# To prime a pump
Ans. Fill casing with water
# Balance draft means;
Ans. Almost atmosphere pressure in the furnace
# A gallon of No. 6 oil contains:
Ans. 152,000 Btu
# What is the efficiency of a steam plant with a heat rate of 12,000 Btu/KW?
Ans. 28.4%
# What type of fuel must be preheated to burn properly?
Ans. Heavy oil
# The fuel oil for mechanical atomizing burners is usually heated
Ans. 150 F
# Fuel suppliers specify a minimum flash point for their oil because a low flash point oil
is;
Ans. Dangerous
# How many cubic feet of furnace volumes are needed to burn one gallon of No. 6 oil?
Ans. 10
# A fuel oil is heated its viscosity
Ans. Decreases
# Only ____ of the turbine work output is required to operate the pump.
Ans. 0.04%
# Superheating the steam to higher temperature decreases the moisture content of the
steam at the __
Ans. Turbine exit
# Regeneration also provides a convenient means of dearating the feedwater to
prevent.
Ans. Boiler corrosion
# Can be apply in Steam turbine cycle (Rankine), Gas turbine cycle (Brayton) and
Combined cycle.
Ans. Cogeneration plant
# In a Rankine cycle with fixed turbine inlet conditions. What is the effect of lowering the
condenser pressure, the heat rejected will;
Ans. Decrease
# In an ideal Rankine cycle with fixed boiler and condenser pressures. What is the effect
of superheating the steam to a higher temperature, the pump work input will;
Ans. Remains the same
# How do the following quantities change when the simple ideal Rankine cycle is
modified with regeneration? The heat rejected will;
Ans. Decreases
# During a combustion process, the components which exist before the reaction are
called
Ans. Reactants
# Is an obvious reason for incomplete combustion.
Ans. Insufficient oxygen
# Higher heating value when H2O in the product of combustion is in
Ans. Liquid form
# Device which transfer heat from low temperature medium to a high temperature one is
a
Ans. Heat pump
# A rule of thumb is that the COP improves by _____ for each C the evaporating
temperature is raised or the condensing temperature is lowered.
Ans. 2 to 4%
# Are generally more expensive to purchase and install than other heating systems, but
they save money in the long run,
Ans. Heat pumps
# The most widely used absorption system is the ammonia-water system, where
ammonia serves as refrigerant and H2O as the
Ans. Transport medium
# Known as drum less boiler.
Ans. Once-through boiler
# Reduces the steam temperature by spraying low temperature water from boiler drum.
Ans. Desuperheater
# Carbon dioxide can be removed by;
Ans. Aeration
# Is often used to absorb silica from water.
Ans. Magnesium hydroxide
# Presence of excess hydrogen ions makes the water
Ans. Acidic
# PH of water varies with
Ans. Temperature
# PH value of ____ is usually maintained for boiler water to minimize corrosion.
Ans. 10.5
# What type of turbine that has a degree of reaction of 1/2?
Ans. Rarsons turbine
# The cooling water is made to fall in series of baffles to expose large surface area for
steam fed from below to come in direct contact.
Ans. Barometric condenser
# Show the variatiom of river flow (discharge) with time.
Ans. Hydrograph
# The pressure at the bottom of a vessel filled with liquids does not depend on the;
Ans. Area of the liquid surface
# A person stands on a very sensitive scale, inhales deeply the reading on the scale:
Ans. Depends on the expansion of the persons chest, relative to the volume
inhaled.
# Buoyancy occurs because, with increasing depth in a fluid:
Ans. The pressure increases
# In order for an object to sink when placed in water its average specific gravity must
be;
Ans. More than 1
# The average solar energy heating the outer edge atmosphere is approximately 442
Btu per hour-foot squared of 1.41KW per meter squared is known as:
Ans. Solar constant
# The ratio of total dissolve solids in the recirculating water to the total solids in the
make-up water.
Ans. Cycles of concentration
# What is the another term for Bryton cycle:
Ans. Joule cycle
# The process of transferring heat from low temperature area to high temperature area;
Ans. Refrigeration
#The device to remove heat from water:
Ans. Chiller
# What device which transfer heat from low temperature area to operate on refrigeration
system:
Ans. Heat pump
# Refers to the process of one substance mixing with another substance:
Ans. Diffusion
# Which of the following statement about Newtonian fluid is most accurate:
Ans. Shea stress is proportional to the rate of strain
# Which is not characteristic of fluids?
Ans. Hysteresis
# The relationship between pressure and altitude in the atmosphere:
Ans. Barometric height relationship
# Flash point means:
Ans. Ignition temperature
# Fire point means:
Ans. None of the above
# Bernoullis equation is a
Ans. Conservation of energy equation
# The pressure at given depth due to several immiscible liquid is:
Ans. The sum of individual pressure
# The relationship between pressure and altitude in the atmosphere is given by the
Ans. Barometric height relationship
# The fact that the buoyant force on a floating object is equal to the weight of the water
displaced
Ans. Archimedes principle
# Which of the following terms does not appear in the steady flow energy equation in
the extended Bernoullis equation
Ans. Hysteresis losses
# The pitot tube can be use to measure fluid velocity as described by the Bernouilli
equation and the relationship between;
Ans. Kinetic energy and static pressure
# The difference between stagnation pressure and total pressure is;
Ans. None. The terms are interchangeable
# Fully turbulent flow in a pipe is characterized by all the following except:
Ans. Parabolic velocity profile
# What is the bulk modulus of the water is;
Ans. 300,000 psi
# Atmospheric pressure does not correspond to approximately;
Ans. 98 N/m2
# Hydraulic press is able to produce a mechanical advantage because;
Ans. An external pressure extended on a fluid is transmitted uniformly through its
volume
# In the operation of a hydraulic press, it is impossible for the output piston to exceed
the input piston
Ans. Work
# If one of the management legs is inclined it is known as:
Ans. Draft gauge
# Why are manometer tube generally large in diameter?
Ans. To avoid significant capillary effect
# Bernoullis equation is an energy conservation based on several reasonable
assumptions:
Ans. All of the above
# Body that emits a constant emissivity, regardless of the wavelength;
Ans. Gray body
# What gives the total emissive power from a black body
Ans. Stefan-Boltzmann law of the fourth power
# What accounts for the spatial arrangement of the body and their emissivity;
Ans. Emissivity factor
# The gray body shape factor is the product of the
Ans. Black body shape factor and emissivity factor
# The product of the area and the shape factor is known as;
Ans. Geometric flux
# Surfaces that re-radiates absorb thermal radiation are known as;
Ans. Refractor materials of refractories
# Pitot tube used to measure
Ans. Velocity
# Liquid gases take the following characteristics of the containers:
Ans. Shape
# For computational convenience usually classed as:
Ans. Real and ideal
# Which of the following statement about Newtonian fluid is more accurate?
Ans. Shear stress is proportional to the rate of strain
# Which of the following is not a characteristic of a real of fluid?
Ans. Experience of eddy current and turbulence
# One could expect the possibility of Reynolds number similarity in all of the following
cases except
Ans. Closed pipe turbulent flow
# One could expect the possibility of Froude number similarity in all of the following
cases except
Ans. Subsonic airflows
# The absolute viscosity of a fluid varies in pressure and temperature as a function of;
Ans. Shear and angular deformation rate
# Turbulent flow of a fluid in a pipe. All of the following except;
Ans. Reynolds number will be less than 2300
# Flow measuring devices include all of the following except;
Ans. Magnetic dynamometer
# Flow measuring devices include all of the following except;
Ans. Mercury barometer
# Flow measuring devices include all of the following except;
Ans. Precision tachometer
# The following are all examples of indirect (secondary) measurements to measure flow
rates using obstruction meters except;
Ans. Volume tanks
# The following are all examples of indirect (secondary) measurements to measure flow
rates using velocity meters except;
Ans. Weight and mass scales
# The following are all examples of indirect (secondary) measurements to measure flow
rates using flow meters except;
Ans. Positive displacement meters
# In series pipe systems which of the following parameters vary from section to section
except;
Ans. Mass flow
# Venturimeter, pitot static gauges, flow nozzles, manometer vary with;
Ans. Flow velocity and pressure
# Other name for R 729?
Ans. Air
# In an indirect refrigeration system, which of the following is used to measure the
density of brine?
Ans. Hydrometer
# Which of the following is the result of opening a hand expansion valve too much in a
refrigeration system?
Ans. Evaporator will freeze back to compressor
If one of the manometer legs is inclined, it is known as:
Ans. Draft gauge
Why are monometer tubes generally large in diameter?
Ans. To avoid significant capillary effect
Bernoullis equation is an energy conservation based on several reasonable
assumptions:
Ans. All of the above
Body that emits a constant emissivity, regardless of the wavelength:
Ans. Gray body
What gives the total emissive from a black body:
Ans. Stefan-boltzman law of the fourth power
What accounts for the spatial arrangements of the body and their emissivity?
Ans. Emissivity factor
The gray body shape factor is the product of the
Ans. Black body shape factor and emissivity factor
The product of the area and the shape factor is known as:
Ans. Geometric flux
Surfaces that re-radiates absorb thermal radiation are known as:
Ans. Refractory materials of refractories
Pito- tube used to measure
Ans. Velocity
Liquid gases take the following characteristic of the containers:
Ans. Shape
For computation convenience usually classed as:
Ans. Real and ideal
Which of the following statement about Newtonian fluid is more accurate?
Ans. Shear stress is proportional to the rate of strain
Which of the following is not a characteristic of a real fluid?
Ans. Experience of eddy current and turbulence
One could expect the possibility of Reynolds number similarity in all of the following
cases except
Ans. Closed pipe turbulent flow
One could expect the possibility of Froude number similarity in all of the following cases
except
Ans. Subsonic airflows
Difference between energy grade line (friction) and the energy line friction loss is:
Ans. Friction and minor losses
The absolute viscosity of a fluid varies in pressure and temperature as a function of:
Ans. Shear and angular deformation rate
Turbulent flow of a fluid in a pipe. All of the following are true except:
Ans. Reynolds number will be less than 2300
Flow measuring devices include all of the following except:
Ans. Mercury barometer
Flow measuring devices include all of the following except:
Ans. Precision tachometer
The following are all examples of indirect (secondary) measurements to measure flow
rates using obstruction meters except:
Ans. Volume tank
The following are all examples of indirect (secondary) measurements to measure flow
rates using velocity meters except:
Ans. Weight and mass scales
The following are all examples of indirect (secondary) measurements to measure flow
rates using flow meters except:
Ans. Positive displacement meter
In series pipe systems which of the following parameters vary from section to section
except:
Ans. Mass flow
Venturimeter, pitot static gauges, flow nozzles, manometers vary with:
Ans. Flow velocity and pressure
Other name for R 729?
Ans. Air
In an indirect refrigeration system, which of the following is used to measure the density
of the brine?
Ans. Hydrometer
Which of the following is the result of opening a hand expansion valve too much in a
refrigeration system?
Ans. Evaporator will freeze back to compressor
Types of turbine used up to 300m head.
Ans. Deriaz turbine
A turbine that has a diagonal flow.
Ans. Deriaz turbine
Oil is atomized either by air blast or pressure jet at about
Ans. 70 bar
Type of solid injection that use single pump supplies fuel under high pressure to a fuel
header.
Ans. Common rail injection
Water flow in diesel engine that is caused by density differential.
Ans. Thermosiphon cooling
Type of lubrication system in diesel engine in which oil from pump is carried to a
separate storage tank outside the engine cylinder and used for high capacity engine.
Ans. Dry sump lubricating system
Produces extreme pressure differentials and violent gas vibration
Ans. Detonation
In a spark ignition engine, the detonation occurs near the______________.
Ans. End of combustion
In a compression ignition engine, the detonation occurs near the _____________.
Ans. Beginning of combustion
Morse test is use to measure the _________ of multi-cylinder engine.
Ans. Indicated power
Ignition delay can be minimized by adding ______ to decrease engine knocking.
Ans. Ethyl nitrate
The work done in throttling valve is
Ans. Zero
Stagnation enthalpy represents the enthalpy of a fluid when it is brought to rest ______.
Ans. Adiabatically
Represents the temperature an ideal gas attains when it is brought to rest adiabatically.
Ans. Stagnation temperature
After passing through a nozzle the density of fluid decreases as the fluid velocity.
Ans. Increases
The volume flow passes through a venture meter will
Ans. Constant
A converging-diverging nozzle is the standard equipment in
Ans. Supersonic aircraft
Nozzles efficiencies range from
Ans. 90%-99%
By reheating the steam before entering the second stage in Rankine cycle, which of the
following will increase?
Ans. Heat rejected
Contains 90% gasoline and 10% ethanol
Ans. Gasohol
In an evaporator section in refrigerating unit, which of the following is the function of
evaporator?
Ans. Absorption of latent heat of vaporization
Which of the following may not be the type of bulb in R-12 refrigeration system?
Ans. Duplex
Which of the following shuts down the compressor motor when discharge pressure rises
to a predetermined point, it cuts in motor when pressure drops to predetermined
pressure:
Ans. High pressure cut-off switch
Which of the following shuts down the compressor motor when discharge pressure rises
to a predetermined pressure and shuts down the compressor motor when there is
decrease in pressure in the suction line.
Ans. Low pressure cut-off switch
The P.O. service pump must have:
Ans. A means of shutting down from outside the machinery space
The relief valve on the P.O. transfer pump discharge
Ans. Back to the transfer pump discharge
When heating hot water, feed pump should at least be how many mm to prevent vapor
lock? Ans.1220 mm
The water column should be provided with a valve drain at least:
Ans. 15 mm pipe size
Lubricating oils have flash points from
Ans. 375-800 degree Fahrenheit
A device receives information in one form of instrument signal and transmits an output
in another form;
Ans. converter
Refinement petroleum Naphtha which by its composition is suitable for use in internal
combustion engine;
Ans. Gasoline
It is possible for water to get into the fuel oil by:
Ans. All of the above
Air chambers are attached to the:
Ans. Discharge side of the reciprocating pumps
On automatic combustion control:
Ans. Reduce the number of burners when the oil pressure approaches minimum
specified
The volumetric efficiency of the compressor increases as the suction pressure:
Ans. Increase
As the discharge pressure increases, the volumetric efficiency:
Ans. Increases
When the suction And discharge pressure are varied in such a direction that the
compression ratio is increased, the volumetric efficiency of the compressor:
Ans. Decrease
For a compressor of any given clearance, the volumetric efficiency
Ans. Varies with compression ratio
The useful energy transfer in Btu/hr divided by input power in Watts;
Ans. Energy efficiency ratio
If air is humidified by injecting steam of by pressuring the air through a hot water spray,
the dry bulb temperature and enthalpy of air
Ans. Increase
Air passing through a solid or absorbed bed, such as silica gel or activated alumina will:
Ans. Decrease in humidity
A device that passes air through dense spray of recirculating water;
Ans. Air washer
The mass of water sprayed to the mass of air passing through the washer per unit time.
Ans. Spray ratio
When the cooling tower is used to provide cold water for the condense of a refrigerator
system the water circulation system will be approximately:
Ans. 3 gal/min-ton
Water lost in small droplets and carried away by the air flow:
Ans. Drift
The ratio of total dissolved solids in the circulating water to the dissolved solids in the
make-up water
Ans. Ratio of concentration
Through windage removes some solids, most must be removed by bleeding some of
the water.
Ans. Bleed-off
A dry cooling tower where stream travels through large diameter trunks to a cross-flow
heat exchanger where it is condensed and cooled by air;
Ans. Direct condensing tower
An economizer is used to:
Ans. Heat the feedwater before it enters the boiler
A manometer measures:
Ans. Air pressure
A pyrometer measures:
Ans. Stack pressure
An attemperator is another name for a
Ans. Desuperheater
Waterwalls are used to:
Ans. Carry of excess heat from the furnace walls
The studs on waterwalls tubes are to:
Ans. Give added surface area to tubes
The heats remove from the cold area
Ans. Use for energy transfer
On the upstroke of the piston, the low pressure vapor is first compressed and then
discharged as high pressure vapor through the discharged valves into the:
Ans. Head of the compressor
The vapor that remains the clearance space at the end discharge stroke is called
Ans. Clearance vapor
Which of the following steps would you take if you found an accumulation of oil on the
furnace floor?
Ans. Open all registers
Which of the following method can be used to clean up the inside of the boiler?
Ans. All of the above
Another term for the oil discharge strainer is the:
Ans. Hot strainer
In a controlled feed water system the power water level maintained by:
Ans. The fluctuation water level in the boiler
The difference between the enthalpy of the product at a specific state and the enthalpy
of reactants at the same state for a complete reaction:
Ans. Enthalpy of reaction
The amount of heat released during a steady flow combustion process when I kg of fuel
burned completely at a specific temperature and pressure.
Ans. Enthalpy of combustion
The enthalpy of a substance at a specific state due to its chemical composition
Ans. Enthalpy of formation
The inlet side of condenser tube are rolled and flared to:
Ans. Allows a smoother entrance flow of circulating water
The purpose of inspection plates on the main condenser is to:
Ans. Check tubes without removing condenser heads
A pump that has one steam cylinder and two liquid cylinders is called a:
Ans. Tandem pump
A mixture of dry air and superheated water vapor
Ans. Unsaturated air
When the air is unsaturated, what is the relation between the dry bulb temperature and
wet bulb temperature?
Ans. Less than
The difference between the dry bulb temperature and wet bulb temperature
Ans. Wet bulb depression
The relative ratio of water vapor to dry air is:
Ans. Humidity ratio
Also known as humidity ratio:
Ans. Specific humidity
The ratio of the actual humidity ratio to the saturated humidity ratio:
Ans. Degree of saturation
Also known as saturation ration:
Ans. Percentage humidity
The partial pressure of the water vapor divided by the saturation pressure:
Ans. Relative humidity
Rapidly increasing the temperature through the air of a chord will cause the water in the
gauge to
Ans. Evaporate
Since the increase in the water vapor a latent heat contents equals the decrease in the
airs sensible heat, the total enthalpies before and after adiabatic saturation is:
Ans. Equal
An adiabatic saturation process follows a line of constant
Ans. Dew point temperature
Which of the following used with adiabatic saturation process?
Ans. Saturation efficiency
The saturation efficiency of a large commercial air washer is typically
Ans. 80%-90%
The presence of oil or scale on the boiler tubes causes:
Ans. Overheating
What would you do before giving a boiler bottom blow off?
Ans. Raise the water level
When installing a new gauge glass in water gauge, you should secure the bolts from?
Ans. Center, alternately, toward each end
One of the main purpose of refractories in the boiler furnace is to:
Ans. Prevent excessive furnace heat losses
A fuel oil service pump steam valves are fitted with:
Ans. Each rod leading to a location outside the fire room
Water tube boilers have
Ans. No fusible plugs
The valve that prevents water from backing out of the boiler in the feed water line:
Ans. Feed check valve
The fuel oil heater is located
Ans. On the discharge side of the service pump
In the forced draft system:
Ans. One fan supplies air to all furnaces
In the closed fire room system
Ans. The fire room is supplied with air from one fan
The air cock on a boiler is located at the:
Ans. Highest point of the steam and water drum
Soot blowers should be used in proper sequence so that
Ans. The soot will be upright toward the uptake
What is the first thing you would check on taking over a watch?
Ans. The water level
A boiler with a fan/blower located in the uptake is operating on
Ans. Induced draft
A fire room that is isolated(closed) operates on:
Ans. Forced draft
How many feed water lines are connected to the boiler?
Ans. Two
If the water in the gauge glass has not been blown for a period of time, the level of
water in the glass will be:
Ans. less than that in the steam and water drum
The enthalpy of fluid when it is brought to rest adiabatically
Ans. Stagnation enthalpy
In the absence of any heat and work and any changes in potential energy, the
stagnation enthalpy of a fluid during a steady flow process:
Ans. remains constant
During the stagnation process, which of the following is converted to enthalpy?
Ans. Kinetic energy
The cross section of a nozzle at the smallest flow area is called:
Ans. Throat
The properties of a fluid at a location where the Mach number is unity are called:
Ans. Critical properties
What is the average fuel oil temperature range of the oil in the discharge line?
Ans. 180-200 degree F
Which of the following is a common type of oil burner?
Ans. All of the above
The boiler gauge glass should be blown down.
Ans. At the beginning of every watch
Gauge pressure of 200 pounds is equivalent to what absolute pressure?
Ans. 215
In a fuel oil service system, the quick closing fuel oil shut-off valve is located between
the:
Ans. Master fuel oil shutoff valve and the boiler
A fuel oil meter placed between the fuel oil service pumps and the fuel oil heaters.
Ans. Cold-type meter
The presence of oil or scale on boiler tubes causes:
Ans. Overheating
What would you do before giving a boiler bottom blow off:
Ans. raises the water level
When installing a new gauge glass in a water gauge you should secure the bolts from:
Ans. Center, alternately towards each ends
One of the main purpose of refractories in the boiler furnace is to:
Ans. Prevent excessive furnace heat losses
A fuel oil service pump steam valves are fitted with:
Ans. Each rod leading to a location outside the fire room
Water tube boilers have
Ans. No fusible plugs
The valve that prevents water from backing out of the boiler in the feed water line:
Ans. Feed check valve
The fuel oil heater is located
Ans. On the discharge side of the service pump
In the forced draft system:
Ans. One fan supplies air to all furnaces
In the closed fire room system
Ans. The fire room is supplied with air from one fan
The air cock on a boiler is located at the:
Ans. Highest point of the steam and water drum
Soot blowers should be used in proper sequence so that
Ans. The soot will be upright toward the uptake
What is the first thing you would check on taking over a watch?
Ans. The water level
A boiler with a fan blower located in the uptake is operating on
Ans. Induced draft
A fire room that is isolated (closed) operates on:
Ans. Forced draft
How many feed water lines are connected to the boiler?
Ans. Two
If the water in the gauge glass has not been blown down for a period, the level of water
in the glass will be:
Ans. Less than that in the steam and water
The enthalpy of fluid when it is brought to rest adiabatically.
Ans. Stagnation enthalpy
In the absence of any heat and work and any changes in potential energy, the
stagnation enthalpy of a fluid during a steady flow process;
Ans. Remains constant
During the stagnation process, which of the following is converted to enthalpy?
Ans. Kinetic energy
The cross section of a nozzle at the smallest flow area is called:
Ans. Throat
The properties of a fluid at a location where a Mach number is unity are called:
Ans. Critical properties
What is the average fuel oil temperature range of the oil in the discharge line?
Ans. 180-200 degree Fahrenheit
Which of the following is a common type of burner?
Ans. All of the above
The boiler gauge glass should be blown down.
Ans. At the beginning of every watch
Gauge pressure of 200 pounds is equivalent to what absolute pressure?
Ans. 215
In a fuel oil service system, the quick closing fuel oil shut-off valve is located between
the:
Ans. Master fuel oil shutoff
A fuel oil meter placed between the fuel oil service pump and fuel oil heater
Ans. Cold-type meter
When securing a scotch boiler
Ans. The belly plug must be removed
If the cooling coils temperature is between the airs dew point, the moisture will
Ans. Condense
What occurs when the air passes through a water spray in an air washer?
Ans. Evaporative cooling process
To prevent ice buildup, the cooled air temperature should be kept from dropping
Ans. Below the freezing point of water
What precaution must be taken before using steam soot blowers?
Ans. Drain thoroughly
Which of the following would you do before blowing the tubes with steam soot blower?
Ans. All of the above
Which of the following would cause a flareback?
Ans. Trying to relight from hot
Sputtering oil burners might indicate
Ans. Water in the fuel oil
The boiler feed water in the feed water heater is heated by:
Ans. Auxiliary exhaust steam

If the boiler is smoking white smoke, a possible cause would be:


Ans. Too much air
The Edwards-type air pump has:
Ans. One set of valve
A scoop condenser is a:
Ans. Single pass condenser
The purpose of the steam to baffle in a condenser
Ans. Prevent the steam from hitting directly on the cooler tubes
The condenser vacuum feed valve is open and the water in the reserve tank is dry. The
result is:
Ans. A loss of vacuum
The division plate in a two-pass-condenser
Ans. Prevents the circulating water from passing directly to overload discharge
The holes in Kinghorn-valve disks
Ans. B
The equilibrium temperature that a regular thermometer measure if exposed to
atmospheric air:
Ans. Dry bulb temperature
The temperature of air that has gone through an adiabatic saturation process
Ans. Wet bulb temperature
If the vapor pressure equals the saturation process, the air is said to be:
Ans. Saturated
When the air is saturated, the dry bulb, wet bulb and the dew point temperature are
Ans. Equal
On taking over a watch, one should check
Ans. All of the above
When you are cleaning fuel oil burner tips, use a
Ans. Brass knife
The amount of steam generated by a boiler is dependent upon
Ans. All of the above
A boiler operated at a pressure not exceeding 1.055 kg/cm 2 gauge steam of
temperature not exceeding 120oC.
Ans. Low pressure heating boiler
No part of the steam generator should be closer than how many from the wall?
Ans. 1 m
Steam generator should be mounted over a suitable foundation or concrete not less
than 305 mm thick and with sufficient area at base to be supported the bearing capacity
of the soil with a factor of safety of
Ans. Not less than 4
No smoke stacks should be closer than how many millimeters from the exposed
woodwork or framing
Ans. 305 mm
The air on an extinguisher fire should be
Ans. Kept closed
Too low F.O. temperature will cause
Ans. Poor combustion and smoky fires
-when securing a scotch boiler
Ans. The belly plug must be remove
-if the cooling coils temperature is between the airs dew point, the moisture will
Ans. Condense
-what occurs when the air passes through a water spray in an air washer?
Ans. Evaporative cooling process
-to prevent ice build up, the cooled air temperature should be kept from dropping
Ans. Below freezing point of water
-What precaution must be taken before using steam soot blowers?
Ans. All of the above
-Which of the following would cause a flareback?
Ans. Trying to relight from hot brickwork
-Sputtering oil burners might indicate
Ans. Water in the fuel oil
-The boiler feed water in the feed water heater is heated by:
Ans. Auxiliary exhaust steam
-If a boiler is smoking white smoke, a possible cause could be:
Ans. Too much air
-The Edwards-type air pump has;
Ans. One set of valve
-A scoop condenser is a:
Ans. Single pass condenser
- The purpose of the steam baffle in a condenser is to:
Ans. Prevent the steam from hitting directly on the cooler tubes
-The condenser vacuum feed valve is open and the water in the reserve tank is dry.
Ans. A loss of vacuum
-The division plate in a two-pass-condenser
Ans. Prevents the circulating water from passing directly to overload discharge
-The holes in kinghorn-valve discs
Ans. B
-The equilibrium temperature that a regular thermometer measure if exposed to
atmospheric air;
Ans. Dry bulb temperature
-The temperature of air that has gone through an adiabatic saturation process
Ans. Wet bulb temperature
-if the vapor pressure equals the saturation process the air is said to be
Ans. Saturated
-When the air is saturated, the dry bulb, wet bulb and dew point temperature are
Ans. Equal
-On taking over a watch, one should check
Ans. All of the above
-When you are cleaning fuel oil burner tips, use a
Ans. Brass knife
-The amount of steam generated by a boiler is dependent upon
Ans. All of the above
-A boiler operated at pressure not exceeding 1.055 gauge steam of steam temperature
not exceeding 120oC.
Ans. Low pressure heating boiler
-No part of the steam generator should be closer than how many from the wall?
Ans. 1m
-Steam generator should be mounted over a suitable foundation or concrete of not less
than 305 mm thick and with sufficient area at base to be supported the bearing capacity
of the soil a factor of safety of
Ans. Not less than 4
-No smoke stacks should be closer than how many millimeters from exposed wood
work or framing?
Ans. 305 mm
-The air register on an extinguisher fire should be
Ans. Kept closed
-Too low F.O. temperature will cause
Ans. Poor combustion and smoky fires
-When the fuel-oil temperature is too high it causes
Ans. Carbon deposits on the fuel-oil heater coils
-A dry cooling tower where steam is condensed by cold water jets
Ans. Indirect condensing dry cooling tower
-The hot condensate is pumped to cross heat exchangers whose it is cooled by.
Ans. Air
-which of the following is the refrigerant of choice in entering air conditioning
Ans. R-22
-In new equipment, which of the following replaces R-11
Ans. R-12
-The super heater is used to
Ans. Remove moisture from steam
-Water tube boiler are
Ans. Rolled in
-The recirculating valve is used to
Ans. Recirculate and heat the fuel oil prior to lighting off
-The fusible plug in a scotch boiler are found in the
Ans. Fire tubes
-Which of the following valves are arrange in a Y-branch fitting?
Ans. Boiler-drum safety valves
-The coils in the feedwater heater are secured by.
Ans. Union fitting
-Natural draft towers can cool the water to within.
Ans. 10 to 12oF
Force draft towers can cool the water to within
Ans. 5 to 8oF
-When you light a fire in the burner, you must always
Ans. Use a torch
-The recirculating line on the burner valves returns the oil to the
Ans. Suction side of service pump
-The valve which shuts off liquid line with the least amount of resistance to flow.
Ans. Gate valve
-The process applied to iron pipe which retards corrosion is called
Ans. Galvanizing
-Listed in sequence, iron pipe series are;
Ans. ,3/8, ,
-The two main purpose of the main condenser are;
Ans. Convert exhaust steam to water
-Pyrometer is a
Ans. High temperature thermometer
-The temperature and humidity of the air to be used for comfort cooling shall be
maintained at effective temperature at air movement within the range from
Ans. 0.0762 to 0.127m/s
-As the air enters the living zone the air motion in such occupied space and which the
only source of contaminants shall have a velocity of not more than
Ans. 0.294m/s
-In air conditioning and ventilation standards, as the air enters into living zone, the
distance above the floor is
Ans. 1.603mm
-Carbon dioxide concentration in air when measured 910 mm above the floor shall be
estimated
Ans. 1000ppm
-The useful refrigerating effect equals to 211kj/mm is
Ans. Tons of refrigeration
-Another name for the fuel oil suction strainer;
Ans. Cold strainer
-In a manually controlled feed water heater system the proper water level is maintained
by
Ans. Operating the feed check valve
-The actual refrigerating capacity of the compressor
Ans. Always less than the theoretical capacity
-The ratio of the actual displacement of the compressor to its piston displacement
Ans. Total or real volumetric efficiency
-Cyclone super heater are located in the
Ans. Steam and water drum
-The purpose of the blow valve is
Ans. Removes oil and scum from surface of water
-The purpose of the salinometer cock is to
Ans. Obtain a sample of boiler water for testing
-Salt in boiler water usually contains from
Ans. Leaky condenser tubes
-Before blowing down a boiler always
Ans. Open skin valves
-The mass flow rate produced by the compressor is
Ans. Equal to the mass of the suction vapor that the compressor takes in the entire
suction inlet unit time
-In order to increase back pressure, one must
Ans. Close in on the back pressure valve
-The actual steam that condenses in the feedwater heater
Ans. Discharge to the hotwell
-The excess steam pressure in the back pressure system exhausts to the
Ans. Atmosphere through a relief valve
-The air pressure supplied to the boiler is measure in
Ans. Inches of water
-The condensate from the coils in the fuel oil heater to the fuel oil heaters goes to the
Ans. Observation tank
The valve between the fuel oil heaters and the burner valve is called the
Ans. Root valve
-The safety device located in the crown sheet of a scotch boiler is called a
Ans. Fusible plug
-Tube retarders in scotch boilers are used to
Ans. Slow down the combustion gases
-In Order for oil to burn properly it must
Ans. Al of the above
-The steam that is discharge from safety valves goes to the
Ans. Atmospheric line
-The polytrophic exponent approaches one with the change in the
process____approaches zero
Ans. Temperature
-The change in kinetic energy of a process ___ as the change in temperature of the
process path increases
Ans. Increases
-The final temperature of an isobaric process if the system work in a _____Quantity and
the Surrounding heat transfer to the system
Ans. Positive
-As the volume of an isothermal expansion process approaches its first value the rate of
change in the system pressure.
Ans. Decreases
-As a fluid flow through a pipe , their potential energy
Ans. Decreases
-The heat of compression___ as suction vapor becomes super heated
Ans. Increases
-The heat content of the refrigeration cycle____ when a liquid- suction line heat
exchanger is installed
Ans. Remains the same
-The COP of refrigeration cycle___ with subcooling
Ans. Increases

-Which defrost method is commonly used small commercial application where the forms
on the evaporator surfaces?
Ans. Hot gas
-Which of the following liquid-chilling evaporator types incorporates overflow heat
exchangers?
Ans. Baudelot
-Which of the following is not a unit of energy?
Ans. Watt
-The unit of special heat are:
Ans. Btu/lbm.oF
-Phase change process are constant________process.
Ans. None of the above
-Which of the following sequences accurately indicates the responses that occur when
heat is transferred from a gas?
Ans. Sensible heat of vapors, latent heat of vaporization, sensible heat of liquids, latent
heat of fusion, sensible heat of fluids
-Btu/lbm are units of
Ans. Specific enthalpy
-As a liquid changes phase to vapor, its enthalpy
Ans. Increases
- As a liquid changes phase to a vapor , its entropy
Ans. Decreases
-The vaporization process that occurs at temperatures below the triple point of a
substance is_____
Ans. Sublimation
-The Vaporization process that occurs when the vapor pressure of a substance is equal
to the atmospheric pressure is:
Ans. Boiling
-As the pressure of a vapor increases, the amount of work increases and its enthalpy__
Ans. Increases
-The entropy of R-134a in a saturated liquid phase at 40 psi is approximately
Ans.21 BTU/lb-oF
-The latent heat of vaporization of R-134a in its saturated vapor phase at 0 degree C is
approximately.
Ans.196.7Kj/Kg
-The condenser of a commercial display cause that is located within the unit is called
Ans. B and C
-Which of the following characteristics of early refrigeration systems which applies to
todays units.
Ans. Relative expensive
-Mechanical refrigeration system make it possible in.
Ans. All of the above
-_______is method used to reduce the dehydration of sensitive fruits and vegetable
during the chilling process
Ans. Water chilling
-Which of the following is not a step in preparing vegetables for frozen storage?
Ans. Coating with syrup
-Loss of food juices by osmosis is a consequence of the ______ freezing process.
Ans. Immersion
-Heat transfer that occurs primarily by conduction is used for ____ freezing process.
Ans. All of the above
-A disadvantage of sharp freezing is the
Ans. All of the above
-Frozen storage chamber temperature set points are usually.
Ans. -20.5 degree C
-Which of the following is not a unit of density?
Ans. Lb/ft3
-Wrapping and refrigerating food products extends the storage life of
Ans. Meat
-The approximate life of strawberries is.
Ans. None of the above
-Which of the following is not category of refrigeration application?
Ans. Transportation air conditioning
-Which of the following is not an industrial application
Ans. Meat display cases
-Which of the following variable is not most often maintain by a commercial air
conditioning unit?
Ans. Temperature
-Providing clean, filtered air for trouble free operation of equipment and instrumentation
is a function of?
Ans. Industrial air conditioning process
-______ is one of the most common application of mechanical refrigeration.
Ans. Preservation of perishable commodities.
-The air conditioning systems for cabins on a luxury ocean liner belongs to the ____
category.
Ans. Commercial air conditioning
Mechanical refrigeration system makes it possible to;
Ans. All of the above
-As the liquid changes to a vapor. The enthalpy
Ans. Increases
-Which of the following systems has the most complicated oil return system?
Ans. Direct staged
-Which of the following process does not after the kinetic energy level of a substance?
Ans. Fusion
-200 degrees F is equal to
Ans. None of the above
-An evaporator in a refrigeration unit makes use of which heat transfer mode?
Ans. All of the above
-When a service technician places his/her hand on a suction line to check the operation
of a system, he/she is using which heat transfer modes?
Ans. Conduction
-Energy added to a vapor is known as Ans. B and C
-The rate of changes velocity is called:
Ans. Acceleration
-Which of the following devices should be used to measure a pressure of 90kpa?
Ans. All of the above
-The height of the mercury column in a barometer placed on a mountain with a local
pressure of 12.5 psi will be?
Ans. All of the above.
-A compound gauge measuring a pressure of 22.44 inches of mercury is equivalent to
an absolute pressure of
Ans. 11.0 Psia
-The electric meter on a home or building measure the amount of that was consumed
over the billing period.
Ans. Energy
-Which of the following is not a method of food preservation?
Ans. Grilling
-Which of the following is not plant, animal or fungi?
Ans. Enzyme
-Which of the following refrigerant group denote a more toxic vapor?
Ans. B3
-What is the molecular weight of helium in 3600 Btu lb K?
Ans. 4.0 lb mol
-In an isobaric process changes in pressure can be caused by changes in.
Ans. None of the above
-Boyles law states that pressure and volume changes in gas process is ___related.
Ans. Inversely
-As a gas is heated in an isothermal gases the volume
Ans. Increases
-As a gas is heated in an isobaric gases the volume
Ans. Decreases
-As a gas is heated in an isometric gases the volume
Ans. Remains constant
-As the volume of a gas decreases, its specific gas constant
Ans. Remains constant
-Vapor do not behave as ideal gases because they experience
Ans. Friction
-The unit of latent heat of vaporization are:
Ans. None of the above
-The unit of latent heat of fusion are:
Ans. Btu-lbm
-A smaller temperature difference between the refrigerant in the liquid line and that in
the evaporator___ the mass flow rate of the refrigerant needed per ton of refrigeration
effect.
Ans. Decreases
-Oil separator are used in system where:
Ans. All of the above
-Which evaporator design incorporates secondary surface heat transfer?
Ans. Finned-tube
-Eutectic filled plate evaporator have a higher_____ than evacuated plate evaporators:
Ans. All of the above
-Which evaporator circuit configuration is less effective when used in expansion load
applications.
Ans. Cross-flow
-Which of the following evaporators feed methods produces the greatest surface without
the used of a liquid pump.
Ans. Flooded
-What type of force convection unit cooler is used in vegetable storage application that
require a velocity of approximately 120m/min?
Ans. Medium velocity
-Which of the following is not a hydrocarbon refrigerants
Ans. inane
-Which of the following refrigerants is most hazardous?
Ans. R-717
-A mixture of two or more refrigerants is called a___
Ans. Zoetrope
-Dessicants are used to remove___ from refrigerants
Ans. Moisture
-As an oil-miscible refrigerant mixes with oil, the viscosity of the oil___
Ans. Decreases
-The society that sponsors research on refrigerants is called:
Ans. ASHRAE
-The scale factor for hard water used in a condenser is:
Ans. 0.002
-As the altitude of a forced air-condenser increases, the surface are of the coil or
volume flow rateof the fan must:
Ans. Increase
-As the water temperature entering a water-cooled condenser decrease, the power
drawn by the compressor.
Ans. Decrease
-Which of the does not increase the volumetric efficiency of a compressor?
Ans. Decreasing the discharge pressure
-What is the compressor operating with a 30 psig suction pressure and a discharge
pressure?
Ans. 3.13
-As the length of the tubes in a chiller barrel increases, the pressure drop across the
inlet and outlet of the barrel:
Ans. Increases
-_____ decreases the volumetric efficiency of a compressor
Ans. Decreasing the suction pressure
-The mechanical efficiency of a compressor_____ as the compressor load increases.
Ans. Remain the same
-The saturated suction temperature used in the selection of a compressor corresponds
to the pressure of the vapor at the
Ans. Inlet of the compressor
-A smaller temperature difference between the refrigerant in the liquid line and that in
the evaporator ____ the mass flow rate of the refrigerant needed per ton of refrigerating
effect.
Ans. Decreases
-The difference between the saturation pressure in the evaporator and that in the
condenser___ as the suction temperature decreases.
Ans. Increases
-The COP and the efficiency of a refrigeration cycle____ as the vaporizing temperature
increases
Ans. Increases
-The volume of vapor that the compressor must handle per minute per ton____ as the
vaporizing temperature increases.
Ans. Decreases
-The quantity of heat rejected at the condenser per unit capacity per minute____ as the
vaporizing temperature decreases.
Ans. Decreases
-As the amount of scale on the refrigerant tubes of an evaporative condenser increases,
the scale formation of hot surfaces
Ans. Decreases
-As the cycles of concentration in a tower or evaporative increases, the scale formation
of hot surfaces:
Ans. Increases
-The____ can only be effectively employed in industrial process that has constant
refrigeration load.
Ans. Hand expansion valve
-The device used to transfer forces across a sealed boundary is called:
Ans. Diaphragm
-The device used to measure the effects of a large pressure drop across the evaporator
is called a
Ans. External equalizer
-The mass flow rate of refrigerant per unit capacity____ as the condensing pressure
decreases.
Ans. Decreases
-It consists of a tapered glass tube set vertically in the fluid or gaseous piping system
with its large end at the top and a metering float free to move vertically in the tube
Ans. Rotameter
-Also termed as monitor light, it indicates which among a number of normal conditions
of a system or device exist.
Ans. Pilot light
-Which of the following is not a caliper?
Ans. Feeler gage
-An underground formation that contains sufficient saturated permeable material to yield
significant quantities of water.
Ans. Aquifer
-Chemical used to speed up sewage sedimentation:
Ans. Lime
-Power expenditure when a current of the one ampere flow across two points having a
voltage drop of one volt.
Ans. Watt
-Prandtl number for air is generally in the order of
Ans. 0.70
Relative humidity of air to be used in standard air cooling: Ans. 55%-60%
A refrigerating system in which the pressure-imposing element is mechanically
operated: Ans. Compression
A cushioning device all the end of a trolley, bridge or other moving parts of a crane
operating on rails to minimize shock
in the event of collision: Ans. Buffer

A boom type mobile crane mounted on endless track or treated belts: Ans. Crawler
crane
An apparatus for raising or lowering a load by the application of a pulling force but does
not include a car or platform riding in guides: Ans. Hoist

Most widely used industrial pressure gage applied to both pressure and vacuum: Ans.
Bourdon tube gage

Hydrocarbons found in liquefied petroleum gas: Ans. All of the above

Which of the following is not a solid fuel: Ans. Tar

Amount of cooling produced by 2000 lbs of ice in melting over a period of 24 hrs.: Ans.
B and C

It smoothens the flow due to the nature of flow of the liquid from a reciprocating pump:
Ans. Air Chamber

Science of force exerted by water in motion: Ans. Hydrodynamics

Lowest permissible water level of a boiler without internal furnace: Ans. 1/3 height of
shell

In pumps, it transmits power from the driver to the impeller: Ans. Shaft

Cavitation occurs when the pressure at any point inside a pump drops below the vapor
pressure corresponding to the temperature of liquid. Its effect include: Ans. All of the
above

Positive displacement pump consisting of a fixed casing containing gears, cams,


screws, vanes, plungers or similar elements actuated by rotation of the drive shaft. Ans.
Rotary pumps

Branch of which is larger than the run. Ans. Bull head tee

A larger pipe or drum into which each group of boiler is connected: Ans. Header

In pipe identification, the color fro pipe used for electricity: Ans. Light orange

Has the same equipment as the refrigerating circuits each with a condenser,
evaporator, and a pressure imposing elements, where the evaporator of one circuit
cools the condenser of the other circuit. Ans. Cascade

The temperature of air to be used for comfort cooling shall be in the range of: Ans. 20
deg to 23.3 deg

A valve held closed by a spring or other means and designed automatically relieve
pressure in excess of its settling. Ans. Pressure relieve device.
A refrigerants which should not be used in public assembly occupancies. Ans. Group 3
refrigerants

Fire involving flammable liquids and gases. Ans. Class B fires

Mechanics of water or other liquids whether at rest or motion. Ans. Hydraulics

Measure the pressure of water discharging from a nozzle by having its open end on the
water and the end connected to a manometer. Ans. Pitot Tube

A vertical turbine pump with the pump and the motor closed coupled and designed to be
installed underground. Ans. Submersible pump

Which of the following is a type of deep well pump. Ans. All of the above.

Steel pipe coated with zinc to resist corrosion. Ans. Galvanized pipe

A fitting with a number of branches in the connecting the smaller pipes. Ans. Manifold

The ratio of peak load to the Average load is termed as _____ in variable load
nomenclature. Ans. Load factor

A heat exchange device used to provide heat transfer between exhaust gases and the
air prior to its entrance to the combustor. Ans. Regenerator

In a sensible heating process, the moisture content: Ans. Remains constant

Flow on both sides on a normal shockwave: Ans. Supersonic on one side, Subsonic on
the other

A change of phase directly from vapor to solid, without passing through the liquid state:
Ans. Deposition

What reaction occurs when the enthalpy of the product is less than the enthalpy of the
reactants? Ans. Exothermic

A well designed engine has a volumetric efficiency within the ranged: Ans. 75%-90%

It is referred to as the maximum continuous power available from a hydroelectric power


plant even under the most adverse hydraulic condition: Ans. Firm power

There are how many feed water lines connected to the boiler? Ans. Two

It is the difference in pressure as measured above or below the atmospheric pressure.


Ans. Draft
The compression ratio of a gas turbine is in the range of 5-8. Ans. 5 to 8

The average pH of a normal rainfall is generally: Ans. Slightly less than 7

Another term used for liquid valve. Ans. King valve


The coefficient of performance of a domestic (local) refrigerator is always: Ans. Greater
than one

A belt-Coleman cycle is a reversed : Ans. Joule cycle

In an air compressor system, the function of a receiver is to: Ans. Collects water and
grease suspended in air

General layout plan for each floor drawn to scale should not be less than scale of: Ans.
1:200

Highest vapor drop in a vapor compression refrigeration cycle occurs in the: Ans.
Expansion valve

Which of the following pairs represent the two broad classification of lubricating oils?
Ans. Straight and Additives

A form of misalignment between the pump and the driver shaft wherein the shafts are
concentric but not parallel. Ans. Angular Misalignment

In the field of metal corrosion, it is the process wherein it exhibits in the quality
deterioration of metals: Ans. Passivation

It is the ratio of the density of liquid substance to the density of water at standard
conditions: Ans. Specific gravity

Term associated with an increase on pressure on a pipe caused by sudden velocity


decrease. Ans. Water hammer

A process which takes place without change in volume. Ans. Isochoric

Weir refers to an opening: Ans. Having partial full flow

The Westphal balance is a laboratory instrument used to: Ans. Specific gravity

Generally, permissible velocity of water flowing through concrete tunnel is: Ans. 4-5 m/s

Refrigerant used in passenger aircraft bins. Ans. Air


It refers to atoms of the same atomic number but differ in atomic masses and molecular
weights. Ans. Isotopes

Piston rings are normally made of: Ans. Cast Iron

Two isothermal and two reversible adiabatic process comprise a: Ans. Carnot cycle

In terms of viscosity, density can be expressed as: Ans. Dynamic viscosity/ kinematic
viscosity

Compressor often used in supercharging Diesel engine: Ans. Root blower type

In treating a person whose eyes accidentally got in contact with Freon use: Ans. Sterile
mineral oil

A device in vapor compression refrigeration system whose primary function is to meter


the flow of
Refrigerant to the evaporator: Ans. Thermostatic expansion valve

From the mathematical perspective, a thermodynamic property is: Ans. A point function

In the production of beer, a raw materials called yeast is added in the: Ans. Starting tubs

Purpose is to keep the moisture from entering the system. Ans. Dehydrator

Medium pressure as applied to valves and fittings implies that a working pressure of
_____ is suitable enough. Ans. 862-1200psi

A liquid-vapor mixture with a dryness factor is allowed to absorb heat. Which of the
following is likely to occur? Ans. Increases

Work rooms referring to the maintenance shop and machine rooms shall be _____ in
the height from floor to the ceiling? Ans. 3,000mm

Defined as a passageway made of sheet metal or other suitable material not


necessarily leak tight, used for conveying air or other gases at low temperature. Ans.
Duct

As a good practical rule, the foundation depth may be taken as _____ times the engine
stroke, the lower factor for well-balanced multi-cylinder engine and the higher factor for
engines with fewer cylinder s or on less firm soil: Ans. 3.2 to 4.2

What is the suggested maximum permissible dos (MDP) of a gamma ray exposed for
individuals now working in a nuclear environment in rem year? Ans.
Most commercially petroleum lubricating oil deteriorates starting from temperature of:
Ans. 200 F

States that the external pressure applied to a confined liquid increases the pressure of
every point in the fluid by an amount equal to the external pressure: Ans. Pascals Law

The average fuel-oil temperature range of the oil in the discharge line to the boilers.
Ans. 180-200 F

Boiler gage glasses should be blown down: Ans. At the start of every watch

The lowest portion of the storage basin where the water is not drawn: Ans. Dead
storage

Which among the following do not measure relative humidity? Ans. Piezometer

Water behind the dam of a hydro electric power plant? Ans. Pondage

Converts energy of water to mechanical energy: Ans. Turbine

Ratio of average load to the rating of the equipment supplying the load. Ans. Plant
factor

Ratio of actual maximum demand load to the connected load. Ans. Demand factor

A device which preheats feed water by utilizing the heat of the flue gases. Ans.
Economizer
Study of the proportion of water vapor content of air. Ans. Air conditioning

Actual temperature of air. Ans. Dry bulb temperature

For a machine foundation with class A mixture what is the proportion of cement, sand
and gravel? Ans. 1:2:4

Component included in the proximate analysis of fuel. Ans. Ash, moisture, Volatile
matter, fixed carbon

The prime mover must have its main steam line: Ans. In loops

Intercooler are primarily used with: Ans. Gas compressors

In the psychrometric chart, the diagonal lines represents: Ans. Wet bulb temperature

Moderator in certain types of nuclear reactors. Ans. Heavy water


The main advantage of turbulous boilers. Ans. Steam pressure can be raised in short
time

The law which states that the entropy of all perfect crystalline solids is zero at absolute
zero temperature. Ans. 3rd law of thermodynamics

Operates between two constant temperature reservoir. Ans. Carnot engine

Specific measurements of moisture content of air. Ans. Degree of saturation

Simultaneous on site generation of electric energy and steam from the same plant
output. Ans. Co-generation

Instrument used for pressure readings. Ans. Manometer

Instrument used to analyze gases. Ans. Orsat Apparatus

The chemical formula of R-12 or dichlorofluoromethane. Ans. CCLF

Which of the following is not desirable property of a refrigerant? Ans. Low thermal
conductivity

Refrigerant widely used in room air conditions. Ans. R-22

What occurs when pumps are connected in parallel? Ans. Increase discharge, same
head

Which of the following is not a cause of cavitation? Ans. Low velocity

Pipe attached to the penstock to be able to let the water be at atmospheric pressure.
Ans. Surge chamber

Which of the following is not considered as gaseous fuel? Ans. Bunker

Ratio of the density of a liquid to the density of water. Ans. Specific Gravity
Measurement of randomness or disorder. Ans. Entropy

Energy cannot be created nor destroyed but can only be transformed from one form to
another. Ans. First law of thermodynamics

In standard Otto cycle when the compression ratio is increased then the thermal
efficiency will: Ans. Increase

Heat energy produced by the movement of molecule within a substance caused by its
temperature. Ans. Internal energy
The law which states that one cannot operate a 100% efficient machine. Ans. 2 nd law of
thermodynamics

In relation to brake power the instrument used to measure torque: Ans. Dynamometer

Ratio of the radiation of an actual body to the radiation of the black body. Ans.
Emittance

Refrigerant used in ice plant. Ans. Ammonia

Measures the pressure of water discharging from the nozzle by having its open end in
the water and the other end connected to a manometer. Ans. Pitot tube

Defined as a wall designed to prevent the spread of fire having a fir resistance for four
hours. Ans. Fire wall

The work done per unit charge when the charge is moved from one point to another?
Ans. Potential at a point

In the relation of PV is constant, what value makes the process isobaric? Ans. Zero

In actual gas behavior, molecular collisions are: Ans. Inelastic

An ideal refrigeration should have: Ans. Low freezing point

The immediate undesirable products from the petroleum based lubricating oil subjected
to high pressure and temperature is referred as: Ans. Carbon Dioxide

A rotary dynamic pump, fluid is at no time confined by moving boundaries in its passage
through the pump. Ans. Non-positive displacement pump

Also known as tangential or Pelton wheel. Ans. Impulse turbine

Term given to a chimney made of steel. Ans. Stack

An integral part of reaction turbine used to recover energy head. Ans. Draft tube

Based on good practice, the vertical distance from the floor soil level to the top edge of
the foundation must be around _____ as minimum distance in machiner foundation.
Ans. 6 inches

The geometric view factor for a black body is always. Ans. One
For every 1000ft ascent, there is a corresponding pressure decrease approximately.
Ans. 1 in of Hg.

Used for impounding water storage and for creating head for the power plant. Ans. Dam
The temperature at which the phase change takes place at a given pressure. Ans.
Saturation pressure

Distinguishes the system from its surroundings may be at rest or in motion. Ans.
Boundary

Quality is a measure of: Ans. Dryness

The highest pressure under which distinguished liquid vapor phase can exist in
equilibrium: Ans. Critical pressure

In a thermodynamic gaseous substance, the relationship between P-V-T given by the:


Ans. Equation of state

A correction factor for gas approaching real gas behavior: Ans. Compressibility factor

Furl production process wherein heavy oil is changed into gasoline by means of a high
pressure, high temperature and longer exposure time. Ans. Thermal cracking

A shut off valve for controlling the flow of refrigerant. Ans. Stop valve

Measured by the amount of its pressure below the prevailing atmospheric pressure.
Ans. Partial vaccum

A relief valve that can positively lift the disc from its seat at least 1.5 mm when there is
no pressure in the boiler. Ans. Water relief valve

Vapor produced in the vaporization of a small amount of liquid refrigerant downstream


of the expansion valve. Ans. Flash gas

Product of the rate of evaporation and the factor of evaporation. Ans. Equivalent
evaporation

Difference between the indicated power and the brake power. Ans. Friction power

In Diesel power plant, its purpose is to reduce the weight-to-power ratio: Ans.
Supercharging

Change of phase from solid to gas. Ans. Sublimation

The memory lost when operating power is removed. Ans. Volatile memory

A refrigeration system using direct method in which refrigerant is delivered to two or


more evaporators in space room or in refrigerators. Ans. Multi-pressure
The carbon dioxide concentration of air in all rooms when measured 910mm above the
floor space not exceed: Ans. 100ppm

Consists of helmets shall be removed immediately after having been used or the seal
broken, must be removed atleast every: Ans. 2 years

Pump whose purpose is to increase the effective water pressure by sucking water from
public service main or private use water system: Ans. Booster pump

A small fitting with a double offset, or shaped like the letter C with the ends turned out.
Ans. Cross-cover

Material interposed between two relatively moving machine elements to reduce wear
and tear: Ans. Lubricant

Measure temperature by electromotive force: Ans. Thermocouple

According to SAE which of the following is a type of lubricating oil? Ans. All of the above

A device that information in one form of an instrument signal and transmits an output
signal on another form: Ans. Digital

A device whose function to pass on information in an unchanged form or in some


modified term. Ans. Switch

A device that receives information on one form of one or more physical quantities
modifies the information and/or its form and produces a resultant output signal. Ans.
Transducer

Which of the following is not a requirement in designing pipe installations? Ans.


Galvanized pipe should be used for steam

Part of a valve used to guide and support valve stem: Ans. Bonnet

In piping installations, the color of pipe tube used for water is: Ans. Green

The color code for pipes used in communication. Ans. White

Which of the method is used for classifying coal. Ans. All of the above

Alcohol frequently considered as fuel for internal combustion engine. Ans. Ethyl alcohol

Measure of resistance to flow. Ans. Viscosity

Converts Solar energy to electrical energy. Ans. Photovoltaic cell


Which of the following is not an instrument used to measure flow rates? Ans. Velometer

Refrigeration follows what cycle? Ans. Reversed Carnot cycle

An evaporator constructed on pipe tubing. Ans. Expansion coil

Reinstalled or second hand boilers shall have a minimum factor of safety of? Ans. 6
A line that shows the rotation of the consumption and the load a steam turbine
generator. Ans. Willans Line

Instrument that indicate the percentage of carbon dioxide in flue gases in a power plant.
Ans. Ranarex indicator

For real process, the net entropy change in the universe is: Ans. Positive

The sum of the internal energy and the product of the pressure and specific volume.
Ans. Enthalpy

Ratio of the maximum demand of the system to the rated capacity of the system. Ans.
Utilization factor

Source of thermal energy. Ans. All of the above

Cycle that has two isentropic and two isometric process. Ans. Otto cycle

Cycle that has two isentropic and two isobaric process. Ans. Brayton cycle

# Two kilogram of gas is confined in a 1 m3 tank at 200 kpa and 88C. What type of gas
is in the tank?
Solution:
PV=mRT
200(1) = 2(8.314/M) (88+273)
M = 30
Therefore: gas is Ethane

# Find the enthalpy of helium if its internal energy is 200 KJ/kg.


Solution:
R = 8.314/4 = 2.0785
K = 1.667 for helium
Cp = k R/(k-1) = 2.0785/(1.667 1) = 3.116 KJ/kg-K
h/ = Cp/Cv
/200 = 5.195/3.116
= ______________ANS

# Compute the mass of a 2 m3 propane at 280 kpa and 40C.


Solution:
Propane is C3 H8 --------- M = 12(3) + 8(1) = 44
PV = mRT
280(2) = m(8.314/44)(40+273)
m = ____________ ANS

# A thermal power plant has a heat rate of a 11,363 Btu/kw-hr. Find the thermal
efficiency of the plant.
Solutionj:
eth = 3412/Heat rate = 3412/11,363 = _________ANS.

# What is the hydraulic gradient of a 1 mile, 17 inches inside diameter pipe when 3300
gal/min of water flow with f = 0.03.
Solution:
V = (3300/7.481)/(/4)(17/12)2(60) = 4.66 ft/s
L = 1 mile = 5280 ft
hL = fLv2/2gD = 0.03(5280)(4.66)2/2(32.2)(17/12) = 37.7 ft
Hydraulic gradient = 37.7/5280 = ___________ANS.

# Find the loss of head in the pipe entrance if speed of flow is 10 m/s.
Solution:
Loss at entrance = 0.5(v2/2g) = 0.5[102 / 2(9.81)] = ___________ANS.

Wet material, containing 220% moisture (dry basis) is to be dried at the rate of 1.5 kg/s
in a continuous dryer to give a product containing 10% (dry basis). Find the moisture
removed, kg/hr.
Solution:
Solid in wet feed = solid in dried product
[1/(1 + 2.2)](1.5) = [1/(1 + 0.1)](x)
x = 0,5156 kg/s (total dried product)
Moisture removed = 1.5 0.5156 = 0.984 kg/s = ___________ANS.

# Copra enters a dryer contacting 70% moisture and leaves at 7% moisture. Find the
moisture removed on each pound of solid in final product.
Solution:
Solid in wet feed = solid in dried product
0.3x = 1
x = 3.333 lbs
1= 0.93y
Y = 1.07527 lb
Moisture removed = x y = 3.333 1.07527 = _________ ANS.

# A 1 m x 1.5 m cylindrical tank is full of oil with SG = 0.92. Find the force acting at the
bottom of the tank in dynes.
Solution:
P = w h = (0.92 x 9.81) (1.5) = 13.5378 kpa
F = PA = 13.5378(/4 x 12) = 10,632 KN = 10,632.56 N x 10,000 dynes/N
F = ____________ANS.

# Find the pressure at the 100 fathom depth of water in kpag


Solution:
H = 100 fathom x 6 = 600 ft
P = w h = (600/3,281)(9.81) = ______________ANS.

# Find the depth in furlong of the ocean (SG = 1.03) if the pressure at the sea bed is
2,032.56 kpag.
Solution:
P=wh
2,032.56 = (1.03 x 9.81) h
H = 201.158 m x 3.281ft/m x 1 yd/3ft x 1 furlong/220 yd = 1 furlong

# Find the mass of 10 quartz of water.


Solution:
V = 10 quartz x 1gal/4quartz x 3.785li/gal x 1m3/1000li
V= 0.0094625 x 10-3 m3
W = m/V
1000 = m/0/0094625 x 10-3
m = __________ ANS.

# Find the mass of carbon dioxide having a pressure of 20 psia at 200F with 10 ft3
volume.
Solution:
PV = m R T
(20 x 144)(10) = m (1545/44)(200 + 460)
m = ___________ANS.

# Find the heat needed to raise the temperature of water from 30C to 100C with 60%
quality. Consider and atmospheric pressure of 101.325 kpa. Use the approximate
enthalpy formula of liquid.
Solution:
At 100C
hf = Cp t = 4.187(100) 418.7 KJ/kg
hfg = 2257 KJ.kg
h2 = hf + xhfg = 418.7 + 0.60(2257) = 1,772.9 KJ/kg
Q = 1(4.187)(100-30) + 1(1772.9 418.7) = _________ANS.

# Find the enthalpy of water at 212F and 14.7 psi if the dryness factor is 30%. Use the
approximate enthalpy formula of liquid.
Solution:
hf = (F 32) = (212 32) = 180 Btu/lb
hfg = 970 Btu/lb
h = hf + xhfg
h = 180 + 0.3(970) = ________ANS.
# An air compressor consumed 1200 kw-hr per day of energy. The electric motor driving
the compressor has an efficiency of 80%. If indicated power of the compressor is 34 kw,
find the mechanical efficiency of the compressor.
Solution:
Pim = 1200kw-hr/24 hrs = 50 kw
BP = 50(0.80) = 40 kw
em = 34/40 = _______ANS.

# A refrigeration system consumed 28,800 kw-hr per month of energy. There are 20%
of energy is lost due to cooling system of compressor and motor efficiency is 90%. If
COP of the sustem is 6, find the tons of refrigeration of the system.
Solution:
Pim = 28,800/(24 x 30) = 40 kw
BP = 40 (0.90) = 36 kw
Wc = 36(1-0.20) = 28.80 kw
COP = RE/Wc
6 = RE/28.80
RE = 172.8/3.516 = ______________ANS.

# A 23 tons refrigeration system has a heat rejected of 100 kw. Find the energy
efficiency ratio of the system.
Solution:
QR = RE + Wc
100 = 23(3.516) + Wc
Wc = 19.132 kw
COP = RE/Wc = (23 x 3.516) / 19.132 = 4.23
EER = 3.412 COP = 3.412(4.23) = ____________ANS.

# A 200 mm x 250 mm, 8-cylinder, 4-stroke diesel engine has a break power of 150 kw.
The mechanical efficiency is 80%. If two of the cylinders were accidentally cut off, what
will be the new friction power?
Solution:
em = BP/IP
0.8 = 150/IP
IP = 187.5 kw
FP1 = IP BP = 187.5 150 = 37.50 kw
FP1 = FP2 = ____________ANS.

# If the energy efficiency ratio of the refrigeration system is 12.6, what is the COP of the
system?
Solution:
EER = 3.412 COP
12.6 = 3.412 COP
COP = ____________ANS.
# An air compressor has a power of 40 kw at 4% clearance. If clearance will increase to
7%, what is the new power?
Solution:
The power of compressor will not be affected with the changes in clearance. Therefore
the power will still be 40 kw.

# What is the approximate value of temperature of water having enthalpy of 208 Btu/lb?
Solution:
h = F 32
208 = F -32
F = _________ANS.

# An Otto cycle has a compression ratio of 8. Find the pressure ration during
compression.
Solution:
P1V1k = P2V2k
(V1/V2)k = (P2/P1)
rK = rp
rp = (8)1.4 = ___________ANS.

# A diesel cycle has a cut off ratio of 2.5 and expansion ratio of 4. Find the clearance of
the cycle.

# Determine the atmospheric pressure at a location where barometric reading is 740


mm Hg and gravitational acceleration is g = 9.7 m/s2. Assume the temperature of
mercury to be 10C, at which the density is 13,570 kg/m3.
Solution:
P = (wg)h = (13,570 x 9.7)(9.74) (1Kpa/1000 N/m2) = ___________ANS.

# The barometer of a mountain hiker reads 930 mbars at the beginning of a hiking trip
and 780 mbars at the end. Neglecting the effect of altitude on local gravitational
acceleration, determine the vertical distance climb. Assume g = 9.7 m/s 2.
Solution:
P1 P2 = w h
(0.93 0.78)(100 kpa/bar) = (1.2 x 0.00981) h
h = 1274.21 m
h = 1274.21 (9.81/9.7) = ___________ANS.

# The lower half of a 10 m high cylindrical container is filled with water and the upper
half with oil that has SG = 8.85. Determine the pressure difference between the top and
bottom of the cylinder.
Solution:
P = Pw + Po = 9.81(5) + (0.85 x 9.81)(15) = ________ANS.

# An ideal gas at 0.80 atmospheres and 87C occupies 0.450 liter. How many moles
are in the sample? (R = 0.0821 liter-atm/mole K)
Solution:
PV = nRT
(0.80 atm)(0.450 li) = n (0.0821 )(87+273)K
n = ___________ANS.

# A certain gas at 101.325 Kpa and 10C whose volume is 2.83 m 3 are compressed into
a storage vessel of 0.31 m3 capacity. Before admission, the storage vessel contained
the gas at a pressure and temperature of 173.8 Kpa and 26C, after admission the
pressure has increased to 1171.8 Kpa. What should be the final temperature of the gas
in the vessel in Kelvin?
Solution:
Solving for the mass of gas which is to be compressed:
PV = mRT
101.325(2.83) = m1R(10 + 273)
m1 = 1.01325/R
Solving for the mass of gas initially contained in the vessel:
PV = mRT
137.8(0.31) = m2R(26 + 273)
m2 = 0.14286/R
Solving for the final temperature:
m3 = m1 + m2
m3 = 1.01325/R + 0.14286/R = 1.156/R
1171.8(0.31) = (1.156/R)RT3
T3 = ____________ANS.

# A perfect gas has a value of R = 58.8 ft-lb/lb-R and k = 1.23. if 20 Btu are added to 10
lbs of this gas at constant volume when initial temperature is 90F, find the final
temperature.
Solution
Q = m cv (t2 t1)
cv = R / (k -1) = = 0.29086 Btu/lb-F
20 = 10(0.29086)(t2 90)
t2 = ___________ANS.

# Ammonia weighing 22kgs is confirmed inside a cylinder equipped with a piston


has an initial pressure of 413KPa at 38C. If 3200KJ of heat is added to the ammonia
until its final pressure and temperature are 413KPa and 100oC respectively, what is the
amount of work done by the fluid in KJ?
SOLUTION
Since the molecular weight of ammonia is 17. Then
R = 8.3143/M = 8.3143/17 = 0.489 KJ/kgoK
P1V1 = mRT1
413V1 = 22(0.489)(38 + 273)
V1 = 8.101 m3
P2V2 = mRT2
413V2 = 22(0.489)(100 + 273)
V2 = 9.716 m3
W = P(V1-V2) = 413(9.716 8.101) = ANS.
# A tank contains 90ft3 of air at a pressure of 350 psig; if the air is cooled until its
pressure and temperature decreases to 200 psig and 70oF respectively. What is the
decrease in internal energy?
SOLUTION
m = PV/RT = (200 + 14.7)(90)(144)/(53.342)(70 + 460) = 98.50 lbs
for constant volume process:
P1/T1 = P2/T2
T2 = 70 + 460 = 530oR
=
T1 = 900oR
U = mcv(T2 T1) = 98.50(0.171)(530 900) = ANS.
# A large mining company was provided with a 3m3 of compressed air tank. Air
pressure in the tank drops from 700KPa while the temperature remains constant at
28oC. What percentage has the mass of air in the tank been reduced?
SOLUTION
Percent mass reduced = (700 150)/700 = ANS.
# A 4m3/hr pump delivers water to a pressure tank. At the start, the gauge reads
138KPa until it reads 276KPa and then the pump was shut off. The volume of the tank
is 180liters. At 276KPa the water occupied 2/3 of the tank volume. Determine the
volume of the water that can be taken out until the gauge reads 138KPa.
SOLUTION
Consider the air pressure:
V2 = 1/3 (180) = 60liters
P1V1 = P2V2
(138 + 101.325)(V1) = (276 + 101.325)(60)
V1 = 94.59 liters
Amount of water to be removed = 2/3 (180) (180 94.59) = ANS.
# A refrigeration plant is rated at 15tons capacity. How many pounds of air per hour
will it cool from 70 to 90oF at constant pressure.
SOLUTION
Tons of refrigeration = mcp(t2 t1)/12000

m= ANS.
# An air standard engine has a compression ratio of 18 and a cut off ratio of 4. If
the intake pressure and temperature are 100KPa and 27oC, find the work in KJ per kg.
SOLUTION

T1 = 27 + 273 = 300oK
Process 1 to 2 is isentropic procees:
T2 = T1(V1/V2)k-1 = 300(18)1.4-1 = 953.301oK
Process 2 to 3 is constant pressure process:
T3/T2 = V2/V3 = rc
T3 = 953.301 (4) = 3813.205oK
QA = mcp(T3 T2) = 1(3813.205 -953.301) = 2860 KJ/kg
W = (.5531)(2860) = ANS.
# Determine the air standard efficiency of an engine operating on the diesel cycle
with clearance of 6% when the suction pressure is 99.97KPa and the fuel injected for
7% of the stroke. Assume k = 1.4.
SOLUTION
V3 V2 = 0.07VD
V2 = 0.06VD
V3 - 0.06VD = 0.07VD
V3 = 0.13VD
rc = V3/ V2 = 0.13VD/0.06VD = 2.167
rk = (1 + 0.06)/0.06 = 17.667
= ANS.
# Steam at 2MPa and 250oC in a rigid cylinder is cooled until the quality is 30%.
Find the heat rejected from the cylinder.
At 2MPa and 250oC: v = 0.11144m3/kg u = 2679.6 KJ/kg
At 2MPa, (saturated): vf = 0.0011767m3/kg, vg = 0.09963m3/kg uf = 906.44
KJ/kg,
ufg = 1693.8KJ/kg
SOLUTION
Q = U2 U1
U1 = 2679.6 KJ/kg
U2 = Uf + xUfg = 906.44 + 0.30(1693.8) = 1414.58 KJ/kg
Q = 1414.58 2679.6 = ANS.
# At 1.3MPa, mixture steam and water has an entropy of 3KJ/kg - oK. Find the
enthalpy of the mixture.
At 1.3MPa: sf = 2.2515, sg = 6.4953, hf = 814.93, hfg = 1972.7
SOLUTION
s = sf + xsfg
3 = 2.2515 + x(6.4953)
x = 0.17614
h = hf + xhfg = 814.93 + 0.17614(1972.7) = ANS.
# Mixture with 70% quality at 500KPa is heated isothermally until its pressure is
300KPa. Find the heat added during the process.
At 500KPa: sf = 1.8607, sfg = 4.9606
o
At 300KPa and 151.86 C s = 7.088 KJ/kg
SOLUTION
s1 = sf + xsfg = 1.8607 + 0.70(4.9606) = 5.333 KJ/kg
s2 = 7.088 KJ/kg
Q = T(s2 - s1) = (151.86 + 273)(7.088 5.333) = ANS.
# A tank contains exactly one kilogram of water consisting of liquid and vapor
equilibrium at 1MPa. If the liquid contains one third and the remaining is vapor of the
volume of the tank, what is the enthalpy of the contents of the tank?
At 1MPa: vf = 0.0011273 vfg = 0.19444
hf = 762.81 hfg = 2015.3
SOLUTION
Let V = total volume of the tank
T2 = T1(V1/V2)k-1 = 300(18)1.4 1 = 953.301oK
Process 2 to 3 is constant process:

T3 = 953.301(4) = 3813.205oK
QA = mcp(T3 T2) = 1(3813.205 - 953.301) = 2850 KJ/kg
W = (0.5531)( 2850) = ANS.
# Determine the air standard efficiency of an engine operating on the diesel cycle
with clearance of 6% when the suction pressure is 99.97KPa and the fuel is injected for
7% of the stroke. Assume k = 1.4.
SOLUTION
V3 - V2 = 0.07VD
V2 = 0.06VD
V3 = 0.07VD + 0.06VD
V3 = 0.13VD
rc = V3/ V2 = 0.13VD /0.06VD = 2.167
rk = (1 + 0.06)/(0.060) = 17.667
e= = ANS.
# A steam at 2MPa and 250oC in a right rigid cylinder is cooled until the quality is
30%. Find the heat rejected from the cylinder.
At 2MPa and 250oC: v = 0.11144 m3/kg u = 2679.6 KJ/kg
At 2MPa, (saturated): vf = 0.0011767 m3/kg, vg = 0.09963
3
m /kg
uf = 906.44 KJ/kg ufg = 1693.8 KJ/kg
SOLUTION
Q = (u2 u1)
u1 = 2679.6KJ/kg
u2 = uf + x ufg = 906.44 + 0.3(1693.8) = 1414.58KJ/kg
Q = (1414.58 2679.6) = ANS
# At 1.3MPa, mixture steam and water has an entropy of 3KJ/kg-oK. Find the
enthalpy of the mixture.
At 1.3MPa: sf = 2.2515, sg = 6.4953, hf = 814.93, hfg = 1972.7
SOLUTION
s = sf + x(sg sf)
3 = 2.2515 + x(6.4953 2.2515)
x = 0.17637
h = hf + xhfg = 814.93 + 0.17637(1972.7) = ANS.
# Mixture with 70% quality at 500KPa is heated isothermally until its pressure is
300KPa. Find the heat added during the process.
At 500KPa: sf = 1.8607, sfg = 4.9606
At 300KPa and 151.86oC: s = 7.0888
SOLUTION
For isothermal process, t1 = t2
s1 = sf + xsfg = 1.8607 + 0.7(4.9606) = 5.333
s2 = 7.0888
Q = T(s2 s1) = (151.86 + 273)(7.0888 5.333) = ANS.
# A tank contains exactly one kilogram of water consisting of liquid and vapor in
equilibrium at 1MPa. If the liquid contains one third and the remaining is vapor of the
volume of the tank, what is the enthalpy of the contents of the tank?
At 1MPa: vf = 0.0011273 vfg = 0.19444
hf = 762.81 hfg = 2015.3
SOLUTION
Let V = total volume of tank
mL = VL/vL = (V/3) / 0.0011273 = 295.69 V
mv = Vv / vv = (2V/3) / 0.1944 = 3.429V
x= = ANS.
# Water substance at 70 bar and 65oC enters a boiler tube of constant inside
diameter of 35mm. The water leaves the boiler tube at 50 bar and 700oK at velocity of
100m/s. Calculate the inlet volume flow(li/sec).
SOLUTION
From steam tables:
At 70bar(7MPa) and 65oC
v1 = 0.001017m3/kg
at 50bar(5MPa) and 700oK(427oC)
v2 = 0.06081m3/kg
m1 = m2
Q1/ v1 = Q2/ v2

V1 = 1.672m/sec
Q1 = A x v = (/4)(0.035)2(1.672) = ANS.
# Steam leaves an industrial boiler at 827.4KPa and 171.6 oC. A portion of the
steam is passed through a throttling calorimeter and is exhausted to the atmosphere
when the calorimeter pressure is 101.4KPa. How much moisture does the steam
leaving the boiler contain if the temperature of the steam at the calorimeter is 115.6 oC?
At 827KPa(171.6oC): hf = 727.25 KJ/kg, hfg = 2043.2 KJ/kg
From table 3: At 101.4KPa and 115.6oC: h2 = 2707.6 KJ/kg
SOLUTION
Let x = quality of steam entering the throttling calorimeter
h1 = h 2
hfg + xhfg = h2
727.25 + x(2043.2) = 2707.6
x = 0.9692; y = 1 - 0.9692 = ANS.
# A throttling calorimeter is connected to the desuperheated steam line supplying
steam to the auxiliary feed pump on a ship. The line pressure measures 2.5MPa. The
calorimeter pressure is 110KPa and 150oC. Determine the entropy of the steam line.
At 110KPa and 150.C: h2 = 2775.6KJ.kg
At 2.5MPa: hf = 962.11KJ/kg, hfg = 1841 KJ/kg, sf = 2.5547, sfg = 3.7028
SOLUTION
For throttling process: (h1 = h2)
h1 = h2 = hf + x hfg
2775.6 = 962.11 + x(1841)
x = 98.5%
s1 = sf + x sfg = 2.5447 + 0.985(3.7028) = ANS.
# o
Atmospheric pressure boils at 212 F. At the vacuum pressure at 24 in Hg, the
temperature is 142oF. Find the boiling temperature when the pressure is increased by
40psia from the atmospheric.
SOLUTION
P2 = 14.7 + 40 = 54.7 psia
P1 = -24(14.7/29.92) + 14.7 = 2.908 psia
By interpolation:

t2 = ANS.
# A certain coal has the following ultimate analysis:
C = 69% N2 = 5% H2 = 2.5% S = 7%
Determine the amount of oxygen of the heating value of fuel is 26 961.45 KJ/kg.
SOLUTION
Qh = 33 820C + 144 212 (H - ) + 9304S
26961.45 = 33820(.69) + 144212 (0.025 O/8) + 9304(0.07)
O= ANS.
# A diesel engine consumed 945 liters of fuel per day at 35oC. If the fuel was
purchased at 15.5oC and 30oAPI at P29.00/li, determine the cost of fuel to operate the
engine per day.
SOLUTION
16

Cost = P29.00/li(935.44li) = ANS.


# A cylindrical tank 4m long and 3m diameter is used for oil storage. How many
days can the tank supply the engine having 27oAPI with fuel consumption of 60kg.hr?
SOLUTION
V = /4 D2 h = /4 (3)2(4) = 28.274 m3

Density of fuel = 0.89274(1000kg/m3) = 892.74kg/m3


w = m/V
V = 60/892.74 = 0.0672 m3/hr
Number of days = 28.274/0.0672 = 420.75hrs = ANS.
# A logging firm in Isabella operates a Diesel Electric Plant to supply its electric
energy requirements. During a 24 period, the plant consumed 250 gallons of fuel at
80oF and produced 2900KW-hrs. Industrial fuel used is 30oAPI and was purchased at
P30.00/li at 60oF. Determine the overall thermal efficiency of the plant.
SOLUTION
Qh = 41130 + 139.6xoAPI = 41130 + 139.6(30) = 45318 KJ/kg
60oF = 15.6oC
80oF = 26.6oC
At 26.6oC
mf = 250gal/24hrs x 3.785li/gal x 0.869kg/li x 1hr/3600sec
mf = 0.00952 kg/sec
load = 2900/24 = 120.833KW
overall efficiency = = ANS.
# The dry exhaust gas from oil engine has the following gravimetric analysis:
CO2 = 21.6% O2 = 4.2% N2 = 74.2%
Specific heats at constant pressure for each component of the exhaust gas in Kcal/kgoC
are:
CO2 = 0.203 O2 = 0.219 N2 = 0.248
Calculate the specific gravity if the molecular weight of air is 28.97 kg/kg-mol.
SOULTION
Converting the gravimetric analysis to volumetric:
CO2 = 0.219/44 = 0.004909
O2 of nitrogen in lb/= 0.042/32 = 0.001312
N2 = 0.742/28 = 0.026500
0.032721 mols/kg-mol
Molecular weight = 1/0.032721 = 30.56kg/kg-mol
SG = 30.56/28.97 = ANS.
# A bituminous coal has the following composition:
C = 71.5% H = 5.0% O = 7.0% N = 1.3% S = 3% Ash =
7.6%
W = 3.4%
Determine the theoretical weight of nitrogen in lb/lb of coal.
SOLUTION
Theo. A/F = 11.5C + 34.5(H O/8) + 4.3S
= 11.5(0.715) + 34.5 (0.05 0.07/8) + 4.3(0.03)
= 9.8 lb air / lb coal
N2 in air by weight = 76.8%
therefore:
theoretical weight of N2 = 0.768(9.8) = ANS.
# A gaseous fuel mixture has a molal analysis:
H2 = 14% CH4 = 3% CO = 27%
O2 = 0.6% CO2 = 4.5% N2 = 50.9%
Determine the air fuel ratio for complete combustion of molal basis.
SOLUTION
Chemical reaction with oxygen:
0.14H2 + 0.070O2 = 0.14H2O
0.03CH4 + 0.060O2 = 0.03CO2 + 0.06H2O
0.27Co + 0.135O2 = 0.27CO2
0.265O2
Actual O2 in product = 0.265 O2 0.006 O2 = 0.259 O2
Molal A/F = 0.259 + 0.259(3.76) = ANS.
# A volumetric analysis of a gas mixture is as follows:
CO2: 12% N2: 80%
O2: 4% CO: 4%
What is the percentage of CO2 on a mass basis?
SOLUTION
Converting to mass basis:
CO2 = 0.12 x 44 = 5.28
O2 = 0.014x32 = 1.28
N2 = 0.82x28 = 22.96
CO = 0.02x28 = 0.56
Total mass of product = 5.28 + 1.28 + 22.96 + 0.56 = 30.08 kg
% mass of CO2 = 5.28/30.08 = ANS.
# The following coal has the following ultimate analysis by weight:
C = 70.5% H2 = 4.5% O2 = 6.0% N2 = 1.0%
S = 3.0% ash = 11% moisture = 4%
A stocker fired boiler of 195000kg/hr steaming capacity uses this coal as fuel. Calculate
volume of air in m3/kg with air at 60oF and 14.7 psia pressure of boiler efficiency is 70%
and FE = 1.10.
SOLUTION
Theo. A/F = 11.5C + 34.5(H O/8) + 4.3S
= 11.5(0.705) + 34.5(0.045 0.06/8) + 4.3(0.03)
= 9.53
Actual A/F = 9.53(1.3) = 12.389 kg air/kg fuel

; mf = 23107.56 (12.389) = 286279.57 kg/hr

101.325(V) = 286279.57(0.287)(15.6 + 273)


V= ANS.
# 23.5 kg of steam per second at 5MPa and 400oC is produced by a steam
generator. The feedwater enters the economizer at 145 oC and leaves at 205oC. The
steam leaves the boiler drum with a quality of 98%. The unit consumes 3kg of coal per
second as received having a heating value of 25102 KJ/kg. What would be the overall
efficiency of the unit in percent?
Steam properties:
At 5MPa and 400oC: h = 3195.7KJ/kg At 5MPa: hf = 1154.23, hg
=1640.1
At 205oC: hf = 875.04 At 145oC: hf = 610.63
SOLUTION
= ANS.
# In a Rankine cycle steam enters the turbine at 2.5MPa (enthalpies and entropies
given) and condenser of 50KPa (properties given), what is the thermal efficiency of the
cycle?
At 2.5MPa: hg = 2803.1 sg = 6.2575
At 50KPa: sf = 1.0910 sfg = 6.5029 hf = 340.49 hfg = 2305.4 vf = 0.0010300
SOLUTION
h1 = 2803.1 KJ/kg
solving for h2:
s = sf + xsfg
6.2575 = 1.0910 + x(6.5029)
x = 0.7945
h2 = hf + xhfg = 340.49 + 0.7945(2305.4) = 2172.13 KJ/kg
h3 = 340.49 KJ/kg
h4 = hf + vf (P2 P1) = 340.49 + 0.00103(2500 - 50) = 342.98 KJ/kg

Efficiency = ANS.

# A two-stage air compressor air at 100 Kpa and 22C discharges to 750Kpa. If
intercooler intake is 105C, determine the value of n.
Solution:
Px = = 273.86 Kpa

1.281 =
n = _____________ ANS.
# A single acting air compressor has a volumetric efficiency of 89%, operates at 500
rpm. It takes in air at 100kpa and 30C and discharges it at 600 Kpa. The air handled is
8 m3/min measured at discharge condition. If compression is isentropic, find mean
effective pressure in Kpa
SOLUTION:
P1V1k = P2V2k
100(V11.4) = 600(8)1.4
V1 = 28.37687 m3/min
VD = 28.768/0.89 = 32.32 m3/min
W= =
W = Pm x V D
7562.19 = Pm x 32.32
Pm = ________ ANS.
# A water-jacketed air compressor handles 0.343 m3/s of air at 96.5 kpa and 21C and
leaving at 480 kpa and 132C; 10.9 kg/h of cooling water enters the jacket at 15C and
leaves at 21C. Determine the compressor break power.
SOLUTION:

n = 1.249
W = = 62.57 KW
Q = heat loss = mCp(t2 t1) = (10.9/3600)(4.187)(21 15) = 0.076 KW
Brake power = W + Q = 62.57 + 0.076 = _______ ANS.
# A double suction centrifugal pump delivers 20 ft3/sec of water at a head of 12 m and
running at 650 rpm. What is the specific speed of the pump?
SOLUTION:
Ns =
Q=
h = 12 x 3.281 = 39.37 ft
Ns = __________ANS.
# Determine the number of stages needed for a centrifugal pump if it is used to deliver
400 gal/min of water and pump power of 15 Hp. Each impeller develops a head of 30 ft.
SOLUTION:
Wp = w Q h
15 x 0.746 = 9.81(400 gal/min x 0.003785 m3/gal x 1/60) h
h =45.20 m x 3.281 ft/m = 148.317 ft
Number of stages = 148.317/30 = 4.94 stages
# The suction pressure of a pump reads 3 in. of mercury vacuum and discharge
pressure reads 140psi is use to deliver 120 gpm of water with specific volume of 0.0163
ft3/lb. Determine the pump work.
SOLUTION:
P1 = -3 in Hg x 101.325/29.92 = -10.15 Kpa
P2 = 140 psi x 101.325/14.7 = 965 Kpa
W = 1/v = 1/.0163 = 61.35 lb/ft3 x 9.81/62.4 = 9.645 KN/m3
h = = = 101.105 m
Q = 120 gal/min x 3.785 li/1gal x 1m3/1000li x 1/60 = 0.00757 m 3/sec
P = w Q h = 9.645(0.00757)(101.105) = ____________ ANS.
# A submersible pump delivers 350 gpm of water to a height of 5 ft from the ground.
The pump was installed 150 ft below the ground level and a draw down of 8 ft during the
operation. If water level is 25 ft above the pump, determine the pump power.
SOLUTION:
h = 5+ 150 (25 8) = 138/3.281 = 42.06 m
Q = 350 gal/min x 0.003785 m3/gal x 1min/60sec = 0.02246 m3/sec
Wp = w Q h = 9.819.02246)(42.06) = __________ ANS.
# A vacuum pump is used to drain a flooded mine shaft of 20C water. The pump
pressure of water at this temperature is 2.34 Kpa. The pump is incapable of lifting the
water higher than 16 m. What is the atmospheric pressure?
SOLUTION:

P1 = ___________ ANS.
# A submersible, multi-stage, centrifugal deep well pump 260 gpm capacity is installed
in a well 27 feet below the static water level and running at 3000 rpm. Drawdown when
pumping at rated capacity is 10 ft. the pump delivers the water into a 25,000 gallons
capacity overhead storage tank. Total discharge head developed by pump, including
friction in piping is 243 ft. Calculate the diameter of the impeller of this pump in inches if
each impeller diameter developed a head of 38 ft.
SOLUTION:
V=DN
V=
D (3000/60) =
D = 0.315 ft = _________ ANS.
# A fan draws 1.42 m3/sec of air at a static pressure of 2.54 cm of water through a duct
300 mm diameter and discharges it through a duct of 275 mm diameter. Determine the
static fan efficiency if total fan mechanical is 75% and air is measured at 25C and 760
mmHg.
SOLUTION:
wa =
# A total head of fan is 187 m and has a static pressure of 210 mm of water gage, what
is the velocity of air flowing if density of air is 1.15 kg/m3?
SOLUTION:
hs = 0.21(1000-1.15) = 182.61 m
h = hs + hv
187 = 182.61 + hv
hv = 4.39 m
4.39 = v2 /2(9.81)
v = _________ANS.
# A fan delivers 5.7 m3/sec at a static pressure of 5.08 cm of water when operating at a
speed of 400 rpm. The power input required is 2.963 KW. If 7.05 m 3/sec are desired in
the same fan and installation, find the pressure in cm of water.
SOLUTION:

N2 = 494.74 rpm

h2 = __________ANS.
# A rigid container is closed at one end and measures 8 in diameter by 12 in long. The
container is held vertically and is slowly moved downward until the pressure in the
container is 17 psia. What will be the depth of the top of the container from the free
water surface?
SOLUTION:
Patm = Pgage + Patm
17 = Pgage + 14.7
Pgage = 2.30 psi
Pgage = w h
2.30(144) = 62.4 (h)
h = 5.3077ft x 12 =__________ans.
# An empty, open can is 30 cm high with a 15 cm diameter. The can, with the open
end down, is pushed under water with a density of 1000 kg/m 3. Find the water level in
the can when the top of the can is 50 cm below the surface.
SOLUTION:
Consider the water pressure
Pw = wh + 101.325 = (0.8 - x)(9.81) + 101.325 = 109.173 9.81x
Consider the air pressure:
P1V1 = P2V2
101.325(A x 0.3) = P2[A(0.3 x)]
P2 =
Pw = P 2
109.173 9.81x =
9.81x2 112.116x + 2.3705 = 0
By quadratic formula:
X = 0.02118 m = _____________ANS
# A cylindrical pipe with water flowing downward at 0.03 m3/s having top diameter of
0.08, bottom diameter of 0.04 m and height of 1.5 m. Find the pressure between the
pipe.
SOLUTION:

Z1 Z2 = 1.5 m
Z2 Z1 = -1.5 m
V1 =
V2 =
+ (-1.5)
P1 P2 = __________ANS.
# Determine the size of pipe which will deliver 8 liters of medium oil (v = 6.10 x 10-6
m2/s) assuming laminar flow conditions.
V=
Re =
For laminar flow, Re = 2000
2000 =
d = 0.835 m = ________ANS.
# The type of flow occupying in a 1 cm diameter pipe which water flows at a velocity of
2.50 m/s. Use v=1.13x10-6 m2/s for water.
SOLUTION:
Re =dV/ v
Re =
Since it is greater than 2000, then it is _________ANS
# An insulated rigid tank initially contains 1.5 lb of helium at 80F and 50 psia. Apaddle
wheel with power rating of 0.02 hp is operated within the tank for 30 min. Determine the
final temperature.
SOLUTION:
W= U = mcv(T2 T1)
0.02hp (0.50 hr)(2545 Btu/hr/hp) = 1.5 (0.171)(T2 80)
T2 = __________ANS.
# A 4 m2 asphalt pavement with emissivity of 0.85 has a surface temperature of 50C.
Find the maximum rate of radiation that can be emmited from the surface.
SOLUTION:
Qr = e ksvA
Ksv = 5.67 x 10-8 (Stefan Boltzman constant)
Qr = 0.85(5.67 x 108)(4)(50 +273)4 = ____________ANS.
# Air at 10C and 80 kpa enters a diffuser of a jet engine steadily with a velocity of 200
m/s. The inlet area of diffuser is 0.40 m2. Determine the mass flow rate of air.
SOLUTION:
w = P/RT = 80/0.287(10+273) = 0.985 kg/m3
m = wvA = 0.985(200)(0.40) = ___________ANS.
# Consider a refrigeration whose 40watts light bulb remains on continuously as a result
of a malfunction of the switch. If the refrigerator has a COP of 1.3 and the cost of
electricity is 8 cents per kw-hr, determine the increase in the energy consumption of the
refrigeration and its cost per year if the switch is not fixed.
SOLUTION:
COP = Re / W ref
1.3 = 40 / W ref
Wref = W b + W ref = 40 + 30.769 = 70.77 watts
W = 0.07077 KW
Cost = 0.07077(8760)(P0.08) = ___________ANS.
# A 75 hp motor that has an efficiency of 91% is worn out and is replaced by a high-
efficiency motor that has an efficiency motor that has an efficiency of 95.4%. Determine
the reduction in heat gain of the room due to higher efficiency under full-load conditions.
SOLUTION:
P01 = (75x0.746)(0.91) = 50.91 KW
P02 = (75x0.746)(0.954) = 53.376 KW
Qreduced = 53.376 50.91 = _________ANS.
# A household refrigerator that has a power input of 450 watts and a COP of 2.5 is to
cool five large watermelons, 10 kg each, to 8C. If the watermelons are initially at 20C,
determine how long will it take for the refrigerator to cool them. The watermelons can be
treated as water whose specific heat is 4.2 KJ/kg-K.
SOLUTION:
COP = Re/Wc
2.5 = Re/450
Re = 1,125 watts
Re = mcp(t2 t1)
450t = (10 x 5)(4.2)(20 8)
t = ____________ANS.
# When a man returns to his wall-sealed house on a summer day, he finds that the
house is at 32C. He returns on the air conditioner which cools the entire house to
20C in 15 minutes. If COP is 2.5, determine the power drawn by the air conditioner.
Assume the entire mass within the house is 800 kg of ait which cv= 0.72 KJ/kg-K, cp=1.0
KJ/kg-K.
SOLUTION:
Re = mcv(t2 t1) = (800/15x60)(0.72)(32 20)
Re = 7.68 KW
Wc = 7.68 / 2.5 = ___________ANS.

A heat source at 800K losses 2000 KJ of heat to a sink at 500K. Determine the
entropy generated during the process.
Solution:
Ssource = -2000/800 = -2.5
Ssink = 2000/500 = 4
Sgen = -2.5/4 = _____________ANS
Helium gas in compressed in an adiabatic compressor from an initial state of 14 psia
and 50F to a final temperature of 320F in a reversible manner. Determine the exit
pressure of Helium.
Solution:
T2/T1 = (P2/P1)n-1/n
(320+460)(50+460) = (P2/14)1.667-1/1.667
P2 = _____________ANS
Air pass thru a nozzle with efficiency of 90%. The velocity of air at the exit is 600 m/s.
Find the actual Velocity at the exit.
Solution:
e = (Va/V3)2
0.9 = (Va/600)2
Va = _____________ANS
A 50 kg block of iron casting at 500K is thrown into a large lake that is at a temperature
of 285K. The iron block eventually reaches the thermal equilibrium with the lake water.
Assuming average specific heat of 0.45 KJ/kg-K for the iron, determine the entropy
generated during the process.
Solution:
Siron = m c ln(T2/T1) = 50(0.45)ln (285/500) = -12.65 KJ/K
Slake = Q/T = [50(0.45)(500-285)] / 285 = 16.97 KJ/K
Sgen = -12.65 + 16.97 = _____________ANS
A windmill with a 12 m diameter rotor is to be installed at a location where the wind is
blowing at an average velocity of 10 m/s. Using standard condition of air (1 atm, 25C),
determine the maximum power that can be generated by the windmill.
Solution:
w = P/RT = 101.325 / (0.287)(25+273) = 1.1847 kg/m 3
m = w A v = 1.1847(/4 x 122)(10) = 1,339.895 kg/s
KE = v2/2000 = 102/2000= 0.05 KJ/kg
Power = m KE = 1,339.895(0.05) = _____________ANS
Consider a large furnace that can supply heat at a temperature of 2000R at a steady
rate of 3000 Btu/s. Determine the exergy of this energy. Assume an environment
temperature of 77F.
Solution:

W = e Q = 0.7315 (3000) = 2194.5 Btu/s = _____________ANS


A heat engine receives hat from a source at 1200K at a rate of 500KJ/s and rejects the
waste heat to a medium at 300K. The power output of the heat engine is 180 KW.
Determine the irreversibility rate for this process.
Solution:
e = (1200-300) / 1200 = 0.75
W= 0.75(500) = 375 KW
Irreversibilities = 375-180 = _____________ANS
A dealer advertises that he has just received a shipment of electric resistance heaters
for residential buildings that have an efficiency of 100 percent. Assuming an indoor
temperature of 21C and outdoor temperature of 10C, determine the second law
efficiency of these heaters.
Solution:
COP1 = 100 efficient = 1
COP2 = (21+273) / (21-10) = 26.72
esl = COP1 / COP2 = 1 / 26.72 = _____________ANS
A thermal power plant has a heat rate of 11,363 Btu/KW-hr. Find the thermal efficiency
of the plant.
Solution:
e = 3412 / Heat rate = 3412 / 11363 = _____________ANS
A rigid tank contains 2 kmol of N2 and 6 kmol of CO2 gases at 300K and 115 Mpa. Find
the tank volume using ideal gas equation
Solution:
Pm Vm = Nm Ru Tm
15,000 Vm = (6+2)(8.314)(300)
Vm = _____________ANS
A spherical balloon with a diameter of 6 m is filled with helium at 20C and 200 kpa.
Determine the mole number.
Solution:
PV=NRT
(200) = N (8.314) (20+273)
N = _____________ANS
The air is an automobile tire with a volume of 0.53 ft3 is at 90Fand 20 psig. Determine
the amount of air that must be added to raise the pressure to the recommended value of
30 psig. Assume the atmospheric pressure to be 14.7 psia and the temperature and the
volume to remain constant.
Solution:
PV=mRT
(20+14.7)(144)(0.53) = m1 (53.3)(90+460)
m1 = 0.09034 lb
(30+14.7)(144)(0.53) = m2 (53.3)(90+460)
m2 = 0.11634 lb
madded = m2 m1 = 0.11634 - 0.09034 = _____________ANS
A rigid tank contains 20 lbm of air at 20 psia and 70F. More air is added to the tank
until the pressure and temperature rise to 35 psia and 90F, respectively. Determine the
amount of air added to the tank.
Solution:
P1 V1 = m1 R1 T1
(20 x 144) (V1) = 20(53.3)(70+460)
V = 196.17 ft3
P2 V2 = m2 R2 T2
(35 x 144)(196.17) = m2(53.3)(90+460)
m2 = 33.73 lbs
madded = m2 m1 = 33.73 20 = _____________ANS
A rigid tank contains 5 kg of an ideal gas at 4 atm and 40C. Now a valve is opened and
half of mass of the gas is allowed to escape. If the final pressure in the tank is 1.5 atm,
the final temperature in the tank is:
Solution:
PV=mRT
(4 x 9.81)(V) = 5(0.287)(40+273)
V = 11.446 m3
PV=mRT
(1.5 x 9.81)(11.446) = (5/2)(0.287)(T)
T = _____________ANS
The pressure of an automobile tire is measured to be 200 kpa(gage) before the trip and
220 kpa(gage) after the tip at a location where the atmospheric pressure is 90 kpa. If
the temperature of the air in the tire before the trip is 25C, the air temperature after the
trip is:
Solution:
T2/T1 = P2/P1
T2 / (25+273) = (220+90) / (200+90)
T2 = 318.55K
t2 = _____________ANS
Water is boiling at 1 atm pressure in a stainless steel pan on an electric range. It is
observed that 2 kg of liquid water evaporates in 30 min. The rate of heat transfer to the
water is:
Solution:
Q = mL = = _____________ANS
Consider a person standing in a breezy room at 20C. Determine the total rate of heat
transfer from this person if the exposed surface area and the average outer surface
temperature of the person are 1.6 m2 and 29C, respectively, and the convection heat
transfer coefficient is 6W/ m2 with emissivity factor of 0.95.
Solution:
Qc = h A (t2-t1) = (6)(1.6)(29.20) = 86.40 watts
Qf = (0.95)(5.67 x 10-8)(1.6)[(29+273)4-(20+273)4] = 81.7 watts
Q = Qc + Qf = 86.40 + 81.7 = _____________ANS
Water is boiled in a pan on a stove at sea level. During 10 minutes of boiling, it is
observed that 200 grams of water has evaporated. Then the rate of heat transfer to the
water is:
Solution:
Q = mL = (0.2/10) (2257) = _____________ANS
An aluminum pan whose thermal conductivity is 237 W/m-C has a flat bottom whose
diameter is 20 cm and thickness 0.4 cm. Heat is transferred steadily to boiling water in
the pan through its bottom at a rate of 500 watts. If the inner surface of the bottom of the
pan is 105C, determine the temperature of the surface of the bottom of the pan.
Solution:
A = /4 (0.20)2 = 0.0314m2

t2 = _____________ANS
For a heat transfer purposes, a standing man can be modeled as a 30 cm diameter, 170
cm long vertical cylinder with both the top and bottom surfaces insulated and with the
side surface at an average temperature of 34C. For a convection heat transfer
coefficient of 15 W/m2-C, determine the rate of heat loss from this man by convection
in an environment at 20C.
Solution:
Qc = k A (t2-t1) = 15( x 0.30 x 1.7) (34-20) = _____________ANS
A 5 cm diameter spherical ball whose surface is maintained at a temperature of 70C is
suspended in the middle of a room at 20C. If the convection heat transfer coefficient is
15 W/m2-C and the emissivity of the surface is 0.8, determine the total heat transfer
from the ball.
Solution:
A = 4 r2 = 4 (0.05)2 = 0.0314 m2
Qc = h A (t2-t1) = 15(0.0314) (70-20) = 23.56 watts
Qr = (0.80) (5.67 x 10-8) (0.0314) [(70+273)4-(50+273)4] = 9.22 watts
Q = Qc + Qr = 23.56 + 9.22 = _____________ANS
A frictionless piston-cylinder device and a rigid tank contain 1.2 kmol of an ideal gas at
the same temperature, pressure, and volume. Now heat is transferred, and the
temperature of both system is raised by 15C. The amount of extra heat that must be
supplied to the gas in the cylinder that is maintained at constant pressure.
Solution:
Q = m Cp (t2-t1) = (1.2 x 8.314)(1)(15) = _____________ANS
A supply of 50 kg of chicken at 6C contained in a box to be frozen to -18C in a freezer.
Determine the amount of heat that needs to be removed. The latent heat of the chicken
is 247 KJ/kg, and its specific heat is 3.32 KJ/kg-C above freezing and 1.77 KJ/kg-C
below freezing. The container box is 1.5 kg, and the specific heat of the box material is
1.4 KJ/kg-C. Also the freezing temperature of chicken is -2.8C.
Solution:
Qchicken = 50[3.32(6+2.8) + 247 + 1.77(-2.8+18)] = 15,156 KJ
Qbox = 1.5(1.4)(6+18) = 50.4 KJ
Q = 15,156 + 50.4 = _____________ANS
Water is being heated in a closed pan on top of a range while being stirred by a paddle
wheel. During the process, 30 KJ of heat is transferred to the water and 5 KJ of heat is
lost to the surrounding air. The paddle-wheel work amounts to 500 N-m. Determine the
final energy of the system if its initial energy is 10 KJ.
Solution:
Final energy = QA + U Qlose + W = 30 + 10 5 + 0.50 = _____________ANS
A classroom that normally contains 40 people is to be air-conditioned with window air-
conditioning units of 5 KW cooling capacity. A person at rest may be assumed to
dissipate heat at rate of about 360 KJ/hr. There are 10 light bulbs in the room, each with
a rating of 100 watts. The rate of heat transfer to the classroom through the walls and
the windows is estimated to be 15,000 KJ/hr. If the room to be maintained at a constant
temperature of 21C, determine the number of window air-conditioning units required.
Solution:
Q = total heat load = 40(360/3600) + 10(0.100) +15,000/3600 = 9.167 KW
No. of air conditioning = 9.167/5 = 1.833 = _____________ANS
A 4m x 5m x 6m room is to be heated by a baseboard resistance heater. It is desired
that the resistance heater be able to raise the air temperature in the room from 7 to
23C within 15 minutes. Assuming no heat losses from the room and an atmospheric
pressure of 100 kpa, determine the required power of the resistance heater. Assume
constant specific heats at room temperature.
Solution:
w = P/RT = 100 / (0.287)(7+273) = 1.244 kg/m3
m = 1.244(4 x 5 x 6) = 149.28 kg
Q= m Cv (t2-t1) = 149.28(0.7186)(23-7) = 1,716.36 KJ
Power = 1,716.36 / (15 x 60) = _____________ANS
A student living in a 4m x 6m x 6m dormitory room turns on her 150 watts fan before
she leaves the room on a summer day, hoping that the room will be cooler when she
comes back in the evening. Assuming all the doors and windows are tightly closed and
disregarding any heat transfer through walls and the windows, determine the
temperature in the room when she comes back 10 hours later. Use specific heat values
at room temperature, and assume the room to be at 100 kpa and 15C in the morning
when she leaves.
Solution:
w = P/RT = 100 / (0.287)(15+273) = 1.2098 kg/m3
m = 1.2098(4 x 6 x 6) = 174.216 kg
Q= m Cv (t2-t1)
0.15(10 x 3600) = 174.216(0.7186)(t2 - 15)
t2 = _____________ANS
A piston-cylinder device whose piston is resting on top of a set of stops initially contains
0.50 kg of helium gas at 100 kpa and 25C. The mass of the piston is such that 500 kpa
of pressure is required to raise it. How much heat must be transferred to the helium
before the piston starts rising?
Solution:
For helium: Cv = R/(k-1) = (8.314/4)(1.667-1) = 3.116 KJ/kg-K
T2 = (25+273)(500/100) = 1,490K
T1 = 25+273 = 298K
Q= m Cv (T2-T1) = 0.50(3.116)(1490-298) = _____________ANS
In order to cool 1 ton (1000 kg) of water at 20C in an insulated tank, a person pours 80
kg of ice at -5C into the water. Determine the final equilibrium temperature in the tank.
The melting temperature and the heat of fusion of ice at atmospheric pressure are 0C
and 333.7 KJ/kg, respectively.
Solution:
Qwater = Qice
1000(4.187)(20-tB) = 80(2.09)(0+5) + 80(333.7) + 80(4.187)(tB-0)
tB = _____________ANS
A fan is powerd by a 0.5 hp motor delivers air at a rate of of 85 m 3/min. Determine the
highest value for the average velocity of air mobilized by the fan. Take the density of air
to be 1.18 kg/m3.
Solution:
P=wQh
0.50(0.746) = (1.18 x 0.00981)(85/80)(h)
= _____________ANS
An Ocean-Thermal Energy Conversion power plant generates 10,000 KW using a warm
surface water inlet temperature of 28C and a cold deep-water temperature of 15C. On
the basis of a 3C drop in the temperature of the warm water and a 3C rise in the
temperature of the cold water due to removal and addition of heat, calculate the power
required in KW to pump the cold-deep water to the surface and through the system heat
exchanger if the required pumping pressure increase is 12 kpa.Assume a Carnot cycle
efficiency and density of cold water to be 1000 kg/m3.
Solution:
e = (TH-TL)/TH = [(28+273)-(15+273)(28+273)] = 0.03679
e = W/QA
0.03679 = 10,000/QA
QA = 271,812.99 KW
QR = QA W = 271,812.99 -10,000 = 261,813 KW
QR = m Cp (t)
261,813 = m (3)(4.187)
m = 20,843.32 kg/s
20,843.32 kg/s or 20,843.32 lit/s = 20.843 m3/s
h = P/w = 12/9.81 = 1.223 m
Wc = w Q h = 9.81(20.843)(1.223) = _____________ANS
A plane-type of solar energy collector with an absorbing surface covered by a glass
plate is to receive an incident radiation of 800 W/m2. The glass plate has a reflectivity of
0.12 and a transmissivity of 0.85. The absorbing surface has an absorptivity of 0.90.
The area of the collector is 5 m2. How much solar energy in watts is absorbed by the
collector?
Solution:
Q = heat absorbed from the sun
Q = 16.3(/4 x 0.008)[15-(-190)] = _____________ANS
An elastic sphere containing gas at 120 kPa has a diameter of 1.0 m. Heating the
sphere causes it to expand to a diameter of 1.3 m. During the process the pressure is
proportional to the sphere diameter. Calculate the work done by the gas in KJ.
Solution:
PD
P = kD
120 = k(1)
k = 120
P = 120 D
V = 4/3 (10/2)3
dV = 12/24 D2 dD

W = _____________ANS

# An ideal gas with a molecular weight of 7.1 kg/kg mol is compressed from 600 kPa
and 280 K to a final specific volume of 0.5 m3/kg. During the process the pressure
varies according to p = 620 + 150v + 95v2 where p is in kPa and v in m3/kg. Calculate
the work of compression in KJ/kg.
Solution:
V1 = RT/P = (8.314/7.1)(280)/(600) = 0.546 m3/kg
W = __________________ANS.
# A one cubic meter container contains a mixture of gases composed of 0.02 kg-mol of
oxygen and 0.04 kg-mol of helium gas at a pressure of 220 kPa. What is the
temperature of this ideal gas mixture in degrees Kelvin?
Solution:
V = V1 + V2
VT = m1R1T1/P1 + m2R2T2/P2
1 = (0.02x32)(8.314/32)(T)/220 + (0.04x32)(8.314/4)(T)/220
T = __________ANS.

# Methyl alcohol (CH3OH) is burned with 25% excess air. How much unburned oxygen
in kg-mol-oxygen/kg-mol-fuel will there be in the products if the combustion is
complete?
Solution:
CH3OH + O2 +3.76N2 = CO2 + H2O + 3.76N2
CH3OH + 1.5O2 + (1.5) (3.76) N2 = 1CO2 + H2O + 1.5(3.76) N2
Consider 25% excess air:
CH3OH + 1.5O2 + 1.25(1.5) (3.76) N2 = 1CO2 + H2O + 1.25(1.5) (3.76) N2 + 0.25(1.5)
O2
Unburned O2 = 0.25(1.5) = _________ANS.

# A 12 DC electrical motor draws a current of 15 amps. How much work in KJ does this
motor produce over a 10-minute period of operation?
Solution:
W=E=QV
W = (15 x 10 x 60)(12) = 108,000 J = _____________ANS.

# A 4liter (2-liter per revolution at standard pressure and temperature) spark ignition
engine has a compression ratio of 8 and 2200 KJ/kg heat addition by the fluid
combustion. Considering a cold air-standard Otto cycle model, how much power will the
engine produce when operating at 2500 rpm?
Solution:
w= 1.2 kg/m3 (standard density of air)
m = 2 li/rev x 2500 rev/min / 1.2 kg/m3 x 1 m3/1000 li x 1 min/60 sec = 0.10 kg/s
e = W/QA
e = 1 1/81.4-1 = 0.5647
0.5647 = W/2200
W = 1,242.34 KJ/kg (0.10 kg/s) = 124.23 KW = _____________ANS.

# A simple Rankine cycle produces a 40 MW of power, 50 MW of process heated and


rejects 50MW of heat to the surroundings. What is the utilization factor of this
cogeneration cycle neglecting the pump work?
Solution:
QA = WT + W process W P = 40 + 50 + 50 = 140 KW
UF = (Qprocess + W T)/QA = (50 +40)/140 = _______________ANS.
# The rate of heat transfer to the surroundings from a person at rest is about 400 KJ/hr.
Suppose that the ventilation system fails in an auditorium containing 120 people and
assuming that the energy goes into the air of volume 1500 m3 initially at 300K and 101
kPa, calculate the rate in C/min of air temperature change.
Solution:
Q = m CV
PV = mRT
101(1500) = m(0.287)(300)
m = 1,759.58 kg
Q = m CV
120(400/60) = 1759.58(0.7186)
= __________ANS.

# An insulated box containing helium gas falls from a balloon 4.5 km above the earths
surface. Calculate the temperature rise in C of the helium when box hits the ground.
Solution:
Cv of helium = 3118.9 J/kg-C
m g h = m Cv
m (9.81) (4500) = m (3118.9)
= ___________ANS.

# Consider two Carnot heat engines operating in series. The first engine receives heat
from the reservoir at 2400 K and rejects the waste heat to another reservoir at
temperature T. The second engine receives heat by the first one, convert some of it to
work, and rejects the rest to a reservoir at 300K. If thermal efficiencies of both engines
are the same, determine the temperature T.
Solution:
e1 = e 2

T = ___________ANS.

# An ideal gas mixtrure consists of 2 kmol of N2 and 6 mol of CO2. The mass fraction of
CO2 is:
Solution:
M = (2/8)(28) + (6/8)(44) = 40
R = 8.314/M = 8.314/40 = __________ANS.

# A Carnot cycle operates between the temperature limits of 300K and 1500K, and
produces 600 KW of net power. The rate of entropy change of the working fluid during
the head addition process is:
Solution:
W=
600 =
= ___________ANS.
# Air in an ideal Diesel cycle is compressed from 3 L to 0.15 L and then it expands
during the constant pressure heat addition process to 0.3 L. Under cold air standard
conditions, the thermal efficiency of this cycle is:
Solution:
rK = 3/0.15 = 20
rC = 0.3 / 0.15 = 2
e = 1 (1/rKk-1)[(rCK 1)/k(rC 1)] = 0.6467 = ________ANS.

# Helium gas in an ideal Otto cycle is compressed from 20C and 2 L to 0.25 L and its
temperature increases by an additional 800C during the head addition process. The
temperature of helium before the expansion process is:
Solution:
rk = 2/0.25 = 8
T2 = (20 + 273)(8)1.667-1 = 1,172 K
T3 = T2 + 800 = 1172 + 800 = 1972K = ______________ANS.

# In an ideal Otto cycle, air is compressed from 1.20 kg/m3 and 2.2 L to 0.26 L and the
net work output of the cycle is 440 KJ/kg. The mean effective pressure for the cycle is:
Solution:
VD = V2 V1 = 2.2 x 10-3 m3 0.26 x 10-3 m3 = 1.94 x 10-3 m3
W = 440 KJ/kg (1.2 kg/m3 x 2.2 x 10-3 m3) = 1.1616 KJ
Pm = W/VD = 1.1616 / 1.94 x 10-3 = _____________ANS.

# An ideal Brayton cycle has a net work output of 150 KJ/kg and backwork ratio of 0.4.
Of both the turbine and the compressor had an isentropic efficiency of 80%, the net
work output of the cycle would be?
Solution:
Backwork Ratio = W C / W T
WC = 0.40 W T
Wnet = W T - W C
Wnet = 250 KJ/kg
Wnet = 250(0.80) 0.40(200)/0.80 = ___________ANS.

# Air enters a turbojet engine at 200 m/s at a rate of 20 kg/s, and exits at 800 m/s
relative to the aircraft. The thrust developed by the engine is:
Solution:
Thrust developed = m (v2 v1) = 20(800-200) = ____________ANS.

# A thermal power plant has a net power 10 MW. The backwork ratio of the plant is
0.005. Determine the compressor work.
Solution:
Wnet = W T + WP
BW = W P / WT
WP = 0.005W T
10,000 = W T 0.005WT
WT = 10,050.25 KW
W C = 0.005(10,050.25) = ____________ANS.
# A heat engine receives heat from a source at 1200K at a rate of 500 KJ/s and rejects
the waste heat to a sink at 300K. If the power output of the engine is 200 KW, the
second law efficiency of this heat engine is?
Solution:
ea = 200/500 = 0.40
et = (TH - TL)/TH = (1200-300)/1200 = 0.75
es = 0.40/0.75 = _________ANS.

# A water reservoir contains 100,000 kg of water at an average elevation of 60m. The


maximum amount of electric power that can be generated from this water is:
Solution:
P = mh = (100,000x0.00981)(60) = 58,860 KJ
P = 58,860 KJ x KWh/3600KJ = ____________ANS.

# A house is maintained at 22C in winter by electric resistance heaters. If the outdoor


temperature is 5C, the second law efficiency of the resistance heaters is:
Solution:
ea = 100% resistance heaters
et = (22-15)/(22+273) = 5.8%
es = 5.8/100 = _________ANS.

# A thermoelectric refrigerator that resembles a small ice chest is powered by a car


battery, and has a COP of 0.10. If the refrigerator cools a 0.350 L canned drink from
20C to 4C in 30 min, determine the average electric power consumed by the
thermoelectric refrigerator.
Solution:
Q = m cp (t2 t1) =[(1 x 0.35)/(30 x 60)] (4.187) (20-4) = 13 watts
COP = RE / W C
0.10 = 13/0.10 = __________ANS.

MACHINE DESIGN ELEMENTS

C # What is the polar section modulus of a 4-inch solid shaft?


A. 25.13 in3 B. 12.57 in4 C. 12.57 in3 D. 25.13 in4
A # A hollow shaft carries a torque 3.4 KN-m at a shearing stress if 55 Mpa. The outside
diameter is 1.25 times that of the inside diameter. Find the inside diameter in mm.
A. 64.67 B. 46.67 C. 84.67 D. 74.84
A # It is specified that the angular deformation in a shaft should not exceed 1 in length
of 1.8m. The allowable shearing stress is 53 Mpa. Determine the diameter of the shaft.
The shaft material has G = 77 x 106 Mpa.
A. 222.34 mm B. 234.22 mm C. 23.42 cm D.
24.22 cm
B # What modulus of elasticity in tension is required to obtain a unit deformation of
0.00105 from a load producing a unit stress of 45,000 psi?
A. 40 x 106 psi B. 43 x 106 psi C. 45 x 106 psi
D. 46 x 106
A # A thrust washer has an inside diameter of 0.5 inch and an outside diameter of 3
inches. For an allowable bearing pressure of 90 psi, how much axial load can it sustain?
A. 618.5 lb B. 537.2 lb C. 702.2 lb D. 871.2 lb
D # An air cylinder has a bore of 25 mm and is operated with shop air at a pressure of
90 ps. Find the push force extended by the piston rod in N.
A. 127 B. 70 C. 402 D. 305
B # A line shaft is to transmit 200 Hp at 900 rpm. Find the diameter of the shaft.
A. 2.18 inches B. 2.26 inches C. 3.18 inches
D. 3.26 inches
B # A main transmitting shaft transmits 350 KW to drive a generator at 2500 rpm, what
is the required diameter of the shaft?
A. 58.5 mm B. 82.7 mm C. 85.3 mm D. 56.2 mm
D # A round steel shaft rotates at 200 rpm and is subjected to a torque of 275 N-m and
a bending moment of 415 N-m. Determine the equivalent twisting moment.
A. 597.84 N-m B. 456.42 N-m C. 546.43 N-m D. 497.85 N-m
A # A 4 inches shaft using a flat key, whose width is 1 inch, is transmitting a torque of
53,000 in-lb. If the design shearing stress is 5000 psi, determine the safe length of key.
A. 6.3 inches B. 5.3 inches C. 4.3 inches D. 7.0 inches
A # a 75 mm diameter shaft is transmitting 300 KW at 600 rpm. A flange coupling is
used and has 6 bolts, each 18 mm in diameter. Find the required diameter if the bolt
circle based on an average shearing stress of 27.5 Mpa.
A. 227.4 mm B. 477.2 mm C. 274.7 mm D. 247.7 mm
C # A heavy duty shaft coupling is to be secured woth 25 mm bolts at a distance of 150
mm from the shaft center. The shaft transmits 4330 KW of power at a speed of 1200 rp,.
If the allowable shearing stress for bolts is 100 Mpa, how many bolts are required?
A. 3 B. 6 C. 5 D. 4
B # A 1.75-inch diameter shaft is supported by two sleeve bearings. The total load on
the wo bearings is 2900 lb. Find the friction power loss, in Hp, if the coefficient of friction
between shaft and bearing is 0.10 and the shaft rotates 200 rpm.
A. 0.88 Hp B. 0.78 Hp C. 0.98 Hp D. 0.68 Hp
B # Find the horsepower lost when a collar is loaded with 1000 lb, rotates at 25 rpm,
and a coefficient of friction at 0.15. The outside diameter of the collar is 4 inches and the
inside diameter is 2 inches.
A. 0.0629 Hp B. 0.0925 Hp C. 0.0269 Hp D. 0.0692 Hp
C # A sleeve bearing has an outside diameter of 38.1 mm and a length of 50.1 mm, the
wall thickness is 3/16 inch. The bearing is subjected to radial load of 450 lb, determine
the bearing pressure.
A. 100 psi B. 150 psi C. 200 psi D. 250 psi
A # A vertical steel cylinder water tank is 30 m in diameter and 45 m high. The allowable
stress of the steel plate is 120 Mpa. Without reinforcing angle bars and rods, determine
the thickness of the steel plate.
A. 55.2 mm B. 56.2 mm C. 65.2 mm D. 52.6 mm
B # Determine the bursting steam pressure of a hemispherical steel shell with a
diameter of 100 inches and made of 0.0635 m thick steel plate. The joint efficiency is
70% and the tensile strength is 60,000 psi.
A. 4020 psi B.4200 psi C. 2500 psi D. 2040 psi
A # A cylinder having an internal diameter of 508 mm and external diameter of 814.4
mm is subjected to an internal pressure of 69 Mpa and an external of 14 Mpa.
Determine the hoop stress at the inner surface of the cylinder
90.11 Mpa B. 91.10 Mpa C. 911.0 Mpa D. 19.10 Mpa
A # The root diameter of a double square thread is 0.55 inch. The screw has a pitch of
0.2 inch. Find the outside diameter and the number of threads per inch.
A. 0.75 inch and 5 threads/inch C. 0.50 inch and 5 threads/inch
B. 0.75 inch and 4 threads/inch D. 0.50 inch and 4 threads/inch
B # Two shafts 3.6 m between centers carry pulleys 1.2 m in diameter and 0.91 m in
diameter respectively, connected by a crossed belt. It is desired to put the belt on as an
open belt. How long a piece must be cut of it?
300 mm B. 350 mm C. 400 mm D. 250 mm
B # A flat belt is 6 inches wide and 1/3 inch thick and transmits 15 Hp. The center
distance is 8 ft. The driving pulley is 6 inches in diameter and rotates at 2000 rpm such
that the loose side of the belt is on top. The driven pulley is 18 inches in diameter. The
belt material is 0.035 lb/in3 and the coefficient of friction is 0.30. Determine the net belt
tension.
167.56 lb B. 157.56 lb C. 156.75 lb D. 175.56 lb
A # Two pulleys, 80 cm apart, carry a belt in an open connection. If the diameter of the
pulleys are 40 cm and 15 cm, what is the length of the belt needed?
248.35 cm B. 348.35 cm C. 265.45 cm D. 305.35 cm
A # A pulley 600 mm in diameter transmits 40 KW at 500 rpm. The arc of contact
between the belt and pulley is 144o , the coefficient of friction between belt and pulley is
0.35 and the safe working stress of the belt is 2.1 Mpa. Determine the belt tension ratio,
neglecting the effect of centrifugal force.
2.41 B. 2.14 C. 1.24 D. 4.12
B # A roller chain and sprocket is to drive vertical discharge bucket elevator. The pitch
of chain connecting sprockets is 1.75. The sprocket is rotating at 120 rpm and has 11
teeth while the driven sprocket is rotating at 38 rpm. Determine the number of teeth of
driven sprocket.
33 teeth B. 35 teeth C. 30 teeth D. 37 teeth
C # A disc clutch has 6 pairs of contacting friction surfaces with an outside diameter of
200 mm and an inside diameter of 100 mm. The coefficient of friction of the clutch
materials is 0.4 and the axial force is 1500 N. The shaft speed is 1200 rpm. Find the Hp
that can be transmitted by the clutch assuming uniform pressure.
35.2 Hp B. 23.5 Hp C. 47.2 Hp D. 27.4 Hp
A # Determine the power capacity of a cone clutch under uniform pressure and
assuming the following pressure and assuming the following conditions; major diameter
= 250 mm, minor diameter = 200 mm, length of conical elements in contact-125 mm,
rotating speed = 870 rpm, coefficient of friction = 0.30, and allowable pressure = 70 kpa.
19.2 KW B. 21.9 KW C. 29.1 KW D. 12.9 KW
D # A flywheel has a mean diameter of 4ft and is required to handle 2250 ft-lb of kinetic
energy. It has a width of 8 inches, mean operating speed is 300 rpm and the coefficient
of fluctuation is to be 0.05. Find the weight of rim, assuming that the arms and hub are
equivalent to 10% of the total rim weight. The flywheel is made up of cast iron with
specific weight of 0.26 lb per cubic inch.
334 lb B. 434 lb C. 433 lb D. 343 lb
A # A 20o involute spur gear has a tooth whole depth of 16.95 mm, a tooth thickness of
13.2 mm, and a pitch of 3. Determine the circular pitch of the gear.
26.6 mm B. 16.6 mm C. 25.6 mm D. 24.6 mm
C # A parallel helical gear-set consists of a 19-tooth pinion driving a 57-tooth gear. The
pinion has a left-hand helix of 20o, a normal pressure angle of 14 1/2 deg . and a normal
diametral pitch of 10 tooth/inch. If the pinion is to transmit 50 Hp at a speed of 1750
rpm, determine the center distance of the two gears.
2.02 inch B. 6.06 inch C. 4.04 inch D. 2.06 inch
A # A right-handed single-thread hardened steel worm has a catalog rating of 2.25 KW
at 650 rpm when meshed with a 48-tooth cast-steel gear. The axial pitch of the worm is
25 mm, normal pressure angle is 14.5o, and the pitch diameter of the worm is 100 mm.
The coefficient of friction is 0.085. Determine the shaft center distance.
241 mm B. 142 mm C. 412 mm D. 124 mm
o
A # A 20 straight-tooth bevel pinion having 14 teeth and a diametral pitch of 6
tooth/inch drives a 42-tooth gear. The two shafts are at right angles and in the same
plane. Find the pitch angle of the pinion.
18.4o B. 20o C. 14.5o D. 20.5o
A # A triple thread worm has a lead angle of 17o and a pitch diameter of 2.2802 inches.
Find the center distance when the worm is mated with a wheel of 48 teeth.
6.72 inches B. 7.26 inches C. 6.27 inches D. 7.62 inches
B # A double thread worm has a pitch diameter of 3 inches. The wheel has 20 teeth and
a pitch diameter of 5 inches. Find the gear helix angle.
4.69o B. 9.46o C. 6.49o D. 6.94o
C # What is the polar section modulus of a 4-inch solid shaft?
A. 25.13 in3 B. 12.57 in4 C. 12.57in3 D. 25.13in4
A # A hollow shaft carries a torque 3.4 KN-m at a shearing stress Mpa. The outside
diameter is 1.25 times that of the inside diameter. Find the inside diameter in mm.
64.87 B. 46.87 C. 84.67 D. 74.64
A # It is specified that the angular deformation in a shaft should not to exceed 1 in
length of 1.8 m. The allowable shearing stress is 83 Mpa. Determine the diameter of the
shaft. The shaft material has G = 77 x 108 Mpa.
222.34 mm B. 234.22 mm C. 23.42 mm D. 24.22 mm
B # What modulus of elasticity in tension is required to obtain a unit deformation of
0.00105 from a load producing a unit stress of 45,000 psi?
40 x 106 psi B. 43 x 106 psi C. 45 x 106 psi D. 46 x 106 psi
A # A thrust washer has an inside diameter of 0.5 inch and an outside diameter of 3
inches. For an allowable bearing pressure of 90 psi, how much axial load can it sustain?
618.5 lb B. 537.2 lb C. 702.2 lb D. 871.2 lb
D # An air cylinder has a bore of 25 mm and is operated with shop air at a pressure of
90 psi. Find the push force exerted by the piston rod in N.
127 B. 70 C. 402 D. 305
B # A line shaft is to transmit 200 Hp at 900 rpm. Find the diameter of the shaft.
2.18 inches B. 2.28 inches C. 3.18 inches D. 3.28 inches
B # A main transmitting shaft transmits 350 KW to drive a generator at 2500 rpm, what
is the required diameter of the shaft?
A.58.5 mm B. 62.7 mm C. 65.3 mm D. 56.2 mm
D # A round steel shaft rotates at 200 rpm and is subjected to a torque of 275 N-m and
a bending moment of 415 N-m. Determine the equivalent twisting moment.
597.84 N-m B. 456.42 N-m C. 546.43 N-m D. 497.85 N-m
A # A 4 inches shaft using a flat key, whose width is 1 inch, is transmitting a torque of
63,000 in-lb. If the design shearing stress is 5000 psi, determine the safe length of key.
6.3 inches B. 5.3 inches C. 4.3 inches D. 7.0 inches
A # A 1200 mm cast iron pulley is fastened to a 112.5 mm shaft by means of a 28.13
mm square key 175 mm long. The key and the shaft have a shearing stress of 14,000
psi. Determine the force acting at the pulley rim that will shear the key.
10015 lb B. 11005 lb C. 11050 lb D. 10501 lb
A # A 75 mm diameter shaft is transmitting 300 KW at 600 rpm. A flange coupling is
used and has 6 bolts, each 18 mm in diameter. Find the required diameter of the bolt
circle based on an average shearing stress of 27.5 Mpa.
227.4 mm B. 477.2 mm C. 274.7 mm D. 247.7 mm
C # A heavy duty shaft coupling is to be secured with 25 mm bolts at a distance of 150
mm from the shaft center. The shaft transmits 4330 KW of power at a speed of 1200
rpm. If the allowable shearing stress for bolts is 100 Mpa, how many bolts are required?
3 B. 6 C. 5 D. 4
B # A 1.75-inch diameter shaft is supported by two sleeve bearings. The total load on
the two bearings is 2800 lb. Find the friction power loss, in Hp, if the coefficient of
friction between shaft and bearing is 0.10 and the shaft rotates 200rpm.
0.88 Hp B. 0.78 Hp C. 0.98 Hp D. 0.68 Hp
B # Find the horsepower lost when a collar is loaded with 1000 lb, rotates at 25 rpm,
and a coefficient of friction at 0.15. The outside diameter of the collar is 4 inches and the
inside diameter is 2 inches.
0.0629 Hp B. 0.0926 Hp C. 0.0269 Hp D. 0.0692 Hp
C # A sleeve bearing has an outside diameter of 38.1 mm and a length of 50.1 mm, the
wall thickness is 3/16 inch. The bearing is subjected to radial load of 450 lb, determine
the bearing pressure.
100 psi B. 150 psi C. 200 psi D. 250 psi
A # A vertical steel cylinder water tank is 30 m in diameter and 45 m high. The allowable
stress of the steel plate is 120 Mpa. Without reinforcing angle bars and rods, determine
the thickness of the steel plate.
55.2 mm B. 56.2 mm C. 65.2 mm D. 52.6 mm
B # Determine the bursting steam pressure of a hemispherical steel shell with a
diameter of 100 inches and made of 0.0635 m thick steel plate. The joint efficiency is
70% and the tensile strength is 98 psi.
4020 psi B. 4200 psi C. 2500 psi D. 2040 psi
A # A cylinder having an internal diameter of 508 mm and external diameter of 914.4
mm is subjected to an internal pressure of 69 Mpa and an external of 14 Mpa.
Determine the hoop stress at the side surface of the cylinder.

A. 90.11 Mpa B. 91.10 Mpa C. 911.0 Mpa D. 19.10 Mpa


A # The root diameter of a double square thread is 0.55 inch. The screw has a pitch of
0.2 inch. Find the outside diameter and the number of threads per inch.
A. 0.75 inch and 5 threads/inch C. 0.50 inch and 5 threads/inch
A. 0.75 inch and 4 threads/inch D. 0.50 inch and 4 threads/inch
B # Two shafts 3.6 m between centers carry pulleys 1.2 m in diameter and 0.91 m in
diameter respectively, connected by a crossed belt. It is desired to put the belt on as an
open belt. How long a piece must be cut of it?
A. 300 mm B. 350 mm C. 400 mm D. 250 mm
B # A flat belt is 6 inches wide and 1/3 inch thick and transmits 15 Hp. The center
distance is 8 ft. The driving pulley is 6 inches in diameter and rotates at 2000 rpm such
that the loose side of the belt is on top. The driven pulley is 18 inches in diameter. The
belt material is 0.035 lb/in3 and the coefficient of friction is 0.30. Determine the net belt
tension.
A. 167.56 lb B. 157.56 lb C. 156.75 lb D. 175.56 lb
A # Two pulleys, 80 cm apart, carry a belt in an open connection. If the diameter of the
pulleys are 40 cm and 15 cm, what is the length of the belt needed?
A. 248.35 cm B. 348.35 cm C. 265.45 cm D. 305.35 cm
A # A pulley 600 mm in diameter transmits 40 KW at 500 rpm. The arc of contact
between the belt and pulley is 144o , the coefficient of friction between belt and pulley is
0.35 and the safe working stress of the belt is 2.1 Mpa. Determine the belt tension ratio,
neglecting the effect of centrifugal force.
A. 2.41 B. 2.14 C. 1.24 D. 4.12
B # A roller chain and sprocket is to drive vertical discharge bucket elevator. The pitch
of chain connecting sprockets is 1.75. The sprocket is rotating at 120 rpm and has 11
teeth while the driven sprocket is rotating at 38 rpm. Determine the number of teeth of
driven sprocket.
A. 33 teeth B. 35 teeth C. 30 teeth D. 37 teeth
C # A disc clutch has 6 pairs of contacting friction surfaces with an outside diameter of
200 mm and an inside diameter of 100 mm. The coefficient of friction of the clutch
materials is 0.4 and the axial force is 1500 N. The shaft speed is 1200 rpm. Find the Hp
that can be transmitted by the clutch assuming uniform pressure.
A. 35.2 Hp B. 23.5 Hp C. 47.2 Hp D. 27.4 Hp
A # Determine the power capacity of a cone clutch under uniform pressure and
assuming the following pressure and assuming the following conditions; major diameter
= 250 mm, minor diameter = 200 mm, length of conical elements in contact-125 mm,
rotating speed = 870 rpm, coefficient of friction = 0.30, and allowable pressure = 70 kpa.
A. 19.2 KW B. 21.9 KW C. 29.1 KW D. 12.9 KW
D # A flywheel has a mean diameter of 4ft and is required to handle 2250 ft-lb of kinetic
energy. It has a width of 8 inches, mean operating speed is 300 rpm and the coefficient
of fluctuation is to be 0.05. Find the weight of rim, assuming that the arms and hub are
equivalent to 10% of the total rim weight. The flywheel is made up of cast iron with
specific weight of 0.26 lb per cubic inch.
A. 334 lb B. 434 lb C. 433 lb D. 343 lb
o
A # A 20 involute spur gear has a tooth whole depth of 16.95 mm, a tooth thickness of
13.2 mm, and a pitch of 3. Determine the circular pitch of the gear.
A. 26.6 mm B. 16.6 mm C. 25.6 mm D. 24.6 mm
C # A parallel helical gear-set consists of a 19-tooth pinion driving a 57-tooth gear. The
pinion has a left-hand helix of 20o, a normal pressure angle of 14 1/2 deg . and a normal
diametral pitch of 10 tooth/inch. If the pinion is to transmit 50 Hp at a speed of 1750
rpm, determine the center distance of the two gears.
A. 2.02 inch B. 6.06 inch C. 4.04 inch D. 2.06 inch
A # A right-handed single-thread hardened steel worm has a catalog rating of 2.25 KW
at 650 rpm when meshed with a 48-tooth cast-steel gear. The axial pitch of the worm is
25 mm, normal pressure angle is 14.5o, and the pitch diameter of the worm is 100 mm.
The coefficient of friction is 0.085. Determine the shaft center distance.
A. 241 mm B. 142 mm C. 412 mm D. 124 mm
A # A 20o straight-tooth bevel pinion having 14 teeth and a diametral pitch of 6
tooth/inch drives a 42-tooth gear. The two shafts are at right angles and in the same
plane. Find the pitch angle of the pinion.
A. 18.4o B. 20o C. 14.5o D. 20.5o
A # A triple thread worm has a lead angle of 17o and a pitch diameter of 2.2802 inches.
Find the center distance when the worm is mated with a wheel of 48 teeth.
A. 6.72 inches B. 7.26 inches C. 6.27 inches D. 7.62 inches
B # A double thread worm has a pitch diameter of 3 inches. The wheel has 20 teeth and
a pitch diameter of 5 inches. Find the gear helix angle.
A. 4.69o B. 9.46o C. 6.49o D. 6.94o
C # What is the polar section modulus of a 4-inch solid shaft?
A. 25.13 in3 B. 12.57 in4 C. 12.57in3 D. 25.13in4
A # A hollow shaft carries a torque 3.4 KN-m at a shearing stress Mpa. The outside
diameter is 1.25 times that of the inside diameter. Find the inside diameter in mm.
A. 64.87 B. 46.87 C. 84.67 D. 74.64
A # It is specified that the angular deformation in a shaft should not to exceed 1 in
length of 1.8 m. The allowable shearing stress is 83 Mpa. Determine the diameter of the
shaft. The shaft material has G = 77 x 108 Mpa.
A. 222.34 mm B. 234.22 mm C. 23.42 mm D. 24.22 mm
B # What modulus of elasticity in tension is required to obtain a unit deformation of
0.00105 from a load producing a unit stress of 45,000 psi?
A. 40 x 106 psi B. 43 x 106 psi C. 45 x 106 psi D. 46 x 106 psi
A # A thrust washer has an inside diameter of 0.5 inch and an outside diameter of 3
inches. For an allowable bearing pressure of 90 psi, how much axial load can it sustain?
A. 618.5 lb B. 537.2 lb C. 702.2 lb D. 871.2 lb
D # An air cylinder has a bore of 25 mm and is operated with shop air at a pressure of
90 psi. Find the push force exerted by the piston rod in N.
A. 127 B. 70 C. 402 D. 305
B # A line shaft is to transmit 200 Hp at 900 rpm. Find the diameter of the shaft.
A. 2.18 inches B. 2.28 inches C. 3.18 inches D. 3.28 inches
B # A main transmitting shaft transmits 350 KW to drive a generator at 2500 rpm, what
is the required diameter of the shaft?
A. 58.5 mm B. 62.7 mm C. 65.3 mm D. 56.2 mm
D # A round steel shaft rotates at 200 rpm and is subjected to a torque of 275 N-m and
a bending moment of 415 N-m. Determine the equivalent twisting moment.
A. 597.84 N-m B. 456.42 N-m C. 546.43 N-m D. 497.85 N-m
A # A 4 inches shaft using a flat key, whose width is 1 inch, is transmitting a torque of
63,000 in-lb. If the design shearing stress is 5000 psi, determine the safe length of key.
A. 6.3 inches B. 5.3 inches C. 4.3 inches D. 7.0 inches
A # A 1200 mm cast iron pulley is fastened to a 112.5 mm shaft by means of a 28.13
mm square key 175 mm long. The key and the shaft have a shearing stress of 14,000
psi. Determine the force acting at the pulley rim that will shear the key.
A. 10015 lb B. 11005 lb C. 11050 lb D. 10501 lb
A # A 75 mm diameter shaft is transmitting 300 KW at 600 rpm. A flange coupling is
used and has 6 bolts, each 18 mm in diameter. Find the required diameter of the bolt
circle based on an average shearing stress of 27.5 Mpa.
A. 227.4 mm B. 477.2 mm C. 274.7 mm D. 247.7 mm
C # A heavy duty shaft coupling is to be secured with 25 mm bolts at a distance of 150
mm from the shaft center. The shaft transmits 4330 KW of power at a speed of 1200
rpm. If the allowable shearing stress for bolts is 100 Mpa, how many bolts are required?
A. 3 B. 6 C. 5 D. 4
B # A 1.75-inch diameter shaft is supported by two sleeve bearings. The total load on
the two bearings is 2800 lb. Find the friction power loss, in Hp, if the coefficient of
friction between shaft and bearing is 0.10 and the shaft rotates 200rpm.
A. 0.88 Hp B. 0.78 Hp C. 0.98 Hp D. 0.68 Hp
B # Find the horsepower lost when a collar is loaded with 1000 lb, rotates at 25 rpm,
and a coefficient of friction at 0.15. The outside diameter of the collar is 4 inches and the
inside diameter is 2 inches.
A. 0.0629 Hp B. 0.0926 Hp C. 0.0269 Hp D. 0.0692 Hp
C # A sleeve bearing has an outside diameter of 38.1 mm and a length of 50.1 mm, the
wall thickness is 3/16 inch. The bearing is subjected to radial load of 450 lb, determine
the bearing pressure.
A. 100 psi B. 150 psi C. 200 psi D. 250 psi
A # A vertical steel cylinder water tank is 30 m in diameter and 45 m high. The allowable
stress of the steel plate is 120 Mpa. Without reinforcing angle bars and rods, determine
the thickness of the steel plate.
A. 55.2 mm B. 56.2 mm C. 65.2 mm D. 52.6 mm
B # Determine the bursting steam pressure of a hemispherical steel shell with a
diameter of 100 inches and made of 0.0635 m thick steel plate. The joint efficiency is
70% and the tensile strength is 98 psi.
A. 4020 psi B. 4200 psi C. 2500 psi D. 2040 psi
A # A cylinder having an internal diameter of 508 mm and external diameter of 914.4
mm is subjected to an internal pressure of 69 Mpa and an external of 14 Mpa.
Determine the hoop stress at the side surface of the cylinder.
D # A 35-tooth pinion turning at 300 rpm drives a 120-tooth gear of 14.5 involute full
depth pressure angle. Determine the rpm of the driven gear.
A. 60 rpm B. 45 rpm C. 75 rpm D. 90 rpm
B # Two parallel shafts connected by cylinders in pure rolling contact and turning in the
same direction having a speed ratio of 2.75. Determine the center distance of the two
shafts if the diameter of the smaller cylinder is 22 cm.
A.18.25 B. 19.25 cm C. 20.25 cm D. 17.25 cm
A # Three extension springs are hooked in parallel that supports a single weight of 100
kg. The first spring is rated at 4 KN/m and the other two springs are rated at 6 KN/m
each. Determine the equivalent stiffness of the three springs.
A. 1.71 KN/m B. 5 KN/m C. 2.71 KN/m D. 3.71
KN/m
B # Three extension springs are hooked in parallel that support a single weight of 100
kg. The springs are rated 4 KN/m, 5 KN/m, and 6 KN/m, respectively. Determine the
equivalent spring constant of the three springs.
A. 10 KN/m B. 15 KN/m C. 9 KN/m D. 11 KN/m
A # Two extension coil springs are hooked in series that support a single weight of 100
kg. The first spring is rated at 4 KN/m and the other spring is rated at 6 KN/m.
Determine the total deflection of the springs.
A. 410 mm B. 310 mm C. 510 mm D. 210 mm
A # If the ultimate shear strength of a steel plate is 42,000 psi, what force is necessary
to punch inch diameter hole in a 5/8 inch thick plate/
A. 61,850 lb B. 65,810 lb C. 61,580 lb D. 60,185 lb
C # In a straight bevel gear, the angle between an element of the pitch cone and an
element of the face cone is called:
A. Face angle B. Pitch angle C. Addendum angle D.
Dedendum angle
D # A four bar mechanism in which one of the links can perform a full rotation relative to
the other three links.
A. Geneva mechanism C. Triple rocker mechanism
B. Crossover-position mechanism D. Grashof mechanism
A # A Grashof four-bar mechanism in which the shorted link is the frame or the fixed link
and the other two cranks completely rotate with their axes.
A. Drank link mechanism C. Double-rocker mechanism
B. Crank-rocker mechanism D. Triple-rocker mechanism
C # For planar four bar linkage, the sum of the shortest and the longest lengths cannot
be greater than the sum of the remaining two links lengths if there is to be a continuous
relative rotation between the two member The preceding statement is known as:
A. Grublers law B. Coriollis law C. Grashofs law D. Freudenteins
law
B # Which of the following is not true for an instant center or centro of planar linkages?
A. Centro is a point common to two bodies having the same velocity in each.
B. Centro is a point in one body about which another body does tend to rotate.
C. Centro is a point in one body about which another body actually turns.
D. Centro is a point in one body about which another body tends to turn.
A # The most common work holding devices of a shaper machine with the base
graduated in degrees that make it possible to swivel any angle.
A. Shaper vise B. Parallel bars and hold down bars C. Lathe
holder D. Swivel head
A # Shaper operation which is shaping the given stock and having the excess material
remain with a tolerable allowance for finishing.
A. Roughing B. Finishing C. Angular Cutting D. Contouring
C # A cutting tool that has two or more cutting edges as in drill presses and milling
machine cutters.
A. Grinder B. Single-point cutting tool C. Multi-point cutting tool D, Two-point
cutting tool
A # The trade name for a patented alloy made up chiefly of cobalt, chromium, and
tungsten varying proportions.
A. Stellite B. Carbology C. Stainless Steel D. Copper
A # The transformation of concepts and ideas into useful machinery is called as:
A. Design B.Synthesis C. Analysis D. Theorem
C # It is a combination of mechanisms and other components that transform, transmits,
or uses energy, load, or motion for a specific purpose.
A. Mechanism B. Engine C. Machine D. Linkage

B # It is defined as synergistic collection of machine elements; synergistic because as a


design it represents an idea or concept greater than the sum of the individual parts.
A. System of mechanism B. Mechanical system C. Design system D.
Expert system
C # It may be defined the displacement per length produced in a solid as the result of
stress.
A. Deformation B. Elongation C. Strain D. Stress
D # The combination of applied normal and shear stresses that produces maximum
principal normal stress, with a third principal stress between or equivalent to the
extremes.
A. Principal shear stress C. Maximum shear stress
B. Principal normal stress D, Bending and shear stress
C # It is a load applied transversely to longitudinal axis of member.
A. Combined loads B. Concentrated load C. Bending load
D. Distributed load
C # It is the intensity and direction of internal force acting at a given point on particular
plane.
A. Load B. Strain C. Stress D. Sustained load
A # It is the capacity of a material to absorb energy when it is deformed elastically and
then, upon unloading, to release this energy.
A. Resilience B. Toughness C. Rigidity D. Ductility
D # It is the strain energy per unit volume required to stress a material from an unloaded
state to the point of yielding.
A. Modulus of roughness C. Modulus of rigidity
B. Modulus of elasticity D. Modulus of resilience
A # The ability of the material to absorb energy up to fracture.
A. Toughness B. Rigidity C. Resilience D. Stiffness
B # The Maximum Shear Stress Theory, as a failure prediction theory, is also known as:
A. von Mises criterion B. Tresca yield criterion C. Coulomb-Mohr
Theory
D. Modified Mohr Theory
B # A failure prediction theory, which states that a part subjected to any combination of
loads, will fail (by yielding or fracturing) whenever the maximum shear stress exceeds a
critical value.
A. Distorsion-energy theory C. Internal friction theory
B. Maximim-shear-stress theory D, Modified Mohr theory
A # A theory in a cyclic and impact loading, which states that damage at any stress level
is proportional ro the number of cycles.
A, Miners rule B. Paris Power Law C. Goodman Rule D. Manson-Coffin
Relationship
B # A journal bearing where the radius of the journal is less than the radius of the
bushing or bearing.
A. Fitted journal bearing C. Partial journal bearing
B. Clearance journal bearing D. Full journal bearing
B # A lubrication where the load-carrying surfaces of the bearing are separated by a
relatively thick film of lubricant, so as to prevent metal to metal contact, and where the
stability of the film can be explained by the laws of fluid mechanics.
A. Hydrostatic lubrication C. Elastohydrodynamic lubrication
B. Hydrodynamic lubrication D. Boundary lubrication
A # A lubrication condition where non-conformal surfaces are completely separated by
lubricant film and no asperities are in contact.
A. Elastohydrodynamic lubrication C. Hydrodynamic lubrication
B. Boundary lubrication D. Hydrostatic lubrication
C # A speed at which rotating shaft becomes dynamically unstable.
A. Normal Speed B. Variable Speed C. Critical Speed D. Average Speed
B # A ball bearing with race contacting pronounced groove for rolling elements.
A. Crown bearing B. Conrad bearing C. Angular-contact bearing D. Cylindrical
bearing
A # A machining process for producing internal straight cylindrical surface or profiles,
with process characteristics and tooling similar to those for turning operations.
A. Boring B. Drilling C. Reaming D. Milling
B # A machining operation for all types of metallic and nonmetallic materials and is
capable of producing circular parts with straight of various profiles.
A. Boring B. Turning C. Drilling D. Milling
B # A set of specification for parts, materials, or processes intended to achieve
uniformity, efficiency, and a specified quality.
A. Code B. Standard C. Law D. Theorem
A # A set of specification for the analysis, design, manufacture, and construction of
something; the purpose of which is to achieve a specified degree of safety, efficiency,
and performance or quality.
A. Code B. Standard C. Law D. Theorem
# It is defined as synergistic collection of machine elements; synergistic because as a
design it represents an idea or concept greater than the sum of the individual parts.
A. System of mechanism B. Mechanical system C. Design system
D. Expert system

# It may be define the displacement per length produced in a solid as a result of stress.
A. Deformation B. Elongation C. Strain D. Stress

# The combination of applied normal and shear stresses that produces maximum
principal normal stress, with a third principal stress between or equivalent to the
extremes.
A. Principal shear stress C. Maximum shear stress
B. Principal normal stress D. Bending and shear stress

# It is a load applied transversely to longitudinal axis of member.


A. Combined load B. Concentrated load C. Bending load
D. Distributed load.

# It is the intensity and direction of internal force acting at given point on particular
plane.
A. Load B. Strain C. Stress D. Sustained load

# It is the capacity of a material to absorb energy when it is deformed elasticity and


then, upon unloading, to release this energy.
A. Resilience B. Toughness C. Rigidity D. Ductility

# It is the strain energy per unit volume required to stress a material from an unloaded
state to the point of yielding.
A. Modulus of roughness B. Modulus of elasticity C. Modulus of rigidity
D. Modulus of resilience

# The ability of the material to absorb energy up to the fracture.


A. Toughness B. Rigidity C. Resilience D. Stiffness

# The Maximum Shear Stress Theory, as a failure prediction theory, is also known as:
A. von Mises criterion B. Tresca yield criterion C. Coulomb-Mohr theory
D. Modified Mohr-theory

# A failure prediction theory, which states that a part subjected to any combination of
loads, will fail (by yielding and fracturing) whenever the maximum shear stress exceeds
a
critical value.
A. Distorsion-energy theory C. Internal friction theory
B. Maximum-shear-stress theory D. Modified Mohr theory

# A theory in cyclic and impact loading, which states that damage at any stress level is
proportional to the number of cycles.
A. Mines rule B. Paris Power Law C. Goodman Rule
D. Manson-coffin Relationship
# A journal bearing where the radius of the journal is less than the radius of the bushing
or bearing.
A. Fitted journal bearing C. Partial journal bearing
B. Clearance journal bearing D. Full journal bearing

# A lubrication where the load-carrying surfaces of the bearing are separated by a


relatively thick film of lubricant, so as to prevent metal to metal contact, and where the
stability of the film can be explained by the laws of fluid mechanics.
A. Hydrostatic lubrication C. Elastohydrodynamic lubrication
B. Hydrodynamic lubrication D. Boundary lubrication

# A lubrication condition where non-conformal surfaces are completely separated by


lubricant film and no asperities are in contact.
A. Elastohydrodynamic lubrication C. Hydrodynamic lubrication
B. Boundary lubrication D. Hydrostatic lubrication

# A speed at which rotating shaft becomes dynamic unstable.


A. Normal speed B. Variable speed C. Critical speed D. Average
speed

# A ball bearing with race containing pronounced groove for rolling elements.
A. Crown bearing B.Conrad bearing C.Angular-contact bearing
D.Cylindrical bearing

# A machining process for producing internal straight cylindrical surface or profiles, with
process characteristics and tooling similar to those for turning operations.
A.Boring B.Drilling C.Reaming D.Milling

# A machining operation for all types of metallic and nonmetallic materials and is
capable of producing circular parts with straight or various profiles.
A.Boring B.Turning C.Drilling D.Milling
# An American nonprofit society, founded in 1921, whose objectives are to improve and
advance the use of fabricated structural stress.
A. American Iron Steel Institute (AISI)
B. American Institute of Steel Construction (AISC)
C. American Society for Metals (ASM)
D. American Society of Testing and Materials (ASTM)

# A sketch of a machine, a machine element, or part of a machine element that shows


all acting forces, such as applied load and gravity forces, and all reactive forces.
A. Schematic diagram B. Free body diagram C. Moment diagram
D. Skeletal diagram

# The size to which a limit of deviations is assigned and is the same for both members
of the fit, it is the exact theoretical size.
A. Nominal stress B. Basic size C. Maximum size D. Minimum size

# The algebraic difference between a size and the corresponding basic size.
Tolerance B. Allowance C. Deviation D. Limit

# The algebraic difference between the maximum limit and the corresponding basic.
A. Fundamental deviation B. Upper Deviation C. Lower deviation D.
Tolerance

# The algebraic difference between the minimum limit and the corresponding basic.
A. Fundamental deviation B. Upper Deviation C. Lower deviation D.
Tolerance

# Either the upper of the lower deviation, depending on which is closer to the basic
size.
A. Fundamental deviation B. Upper Deviation C. Lower deviation D.
Tolerance

# The difference between the maximum and minimum size limits of a part.
A. Allowance B. Tolerance C. Deviation D. Basic size

# The stated maximum and minimum dimensions.


A. Tolerance B. Limits C. Nominal size D. Basic size

# A general term that refers to the mating of cylindrical parts such as bolt or a hole; it is
used only when the internal member is smaller that the external member.
A. Clearance B. Interference C. Allowance D. Tolerance

# The opposite of clearance, for mating cylindrical parts in which the internal member is
larger than the external member.
A. Clearance B. Allowance C. Tolerance D. Interference

# The minimum stated clearance or the maximum stated interference for mating parts.
A. Clearance B. Allowance C. Tolerance D. Interference

# The property of a material that measures the degree of plastic deformation sustained
at fracture.
A. Toughness B. Stiffness C. Ductility C. Brittleness

# Compounds of metallic elements, most frequently oxides, nitrides, and carbides.


A. Plastic B. Polymers C. Ceramics D. Alloy

# A material having different properties in all directions at point in solid.


A. Isotropic material B. Anisotropic material C. Orthropic material
D. Ceramic material
# A material having different properties in three mutually perpendicular directions at
point in solid and having three mutually perpendicular planes of material symmetry.
A. Orthotropic material B. Isotropic material C. Anistropic material
D. Thermoplastic material

# The combination of two or more materials, usually consisting of fiber and


thermosetting polymer.
A. Brittle materials B. Composite materials C. Polymers D.
Ceramics

# A theorem stating that when a body is elastically deformed by a system of loads, the
deflection at any point p in any direction a is equal to the partial derivatives of the strain
energy (with the system of loads acting) with respect to a load at p in the direction a.
A. Poissons Theorem B. Newtons Theorem C. Castiglianos Theorem
D. Mohrs Theorem

# A principal or method that a deflection at any point in bar is equal to sum f deflection
caused by each load acting separately.
A. Summation Method B. Method of balancing C. Method of superposition
D. Shear and Moment diagram method

# A failure prediction theory in which failure is caused by the elastic energy associated
with shear deformation.
A. Maximum-shear-stress theory C. Maximum-normal-stress theory
B. Distorsion-energy theory D. Internal friction theory

You might also like